Download as pdf or txt
Download as pdf or txt
You are on page 1of 166

Ancient History

Compiled by Sanjay Pahade


6th Standard -: Ch 11-67
11th Standard -: Ch 11-111

Telegram Channel -: Mpsc Super Batch 1 Compiled by Sanjay Pahade 1


1. The Indian Subcontinent and History

1.1 Geographical conditions and the


course of history
1.2 Geographical features of India
1.3 The Indian subcontinent

1.1 Geographical conditions and history


In the fifth standard, we saw at some
length that there is a close relationship
between man and his environment. We saw
how the changes in the lifestyle of the early
man and his technology were related to the
changes in his surroundings. We also
reviewed the history of human civilization
from the Stone Age to the agricultural
civilizations that flourished on the banks
of rivers.
Types of houses
History is a coherent account of the
significant past events in the progress of Our diet, clothing, housing, occupation,
human culture. Time, place, society and in fact, all human life in any region depends
individuals are the four major pillars of to a large extent on its geographical
history. We cannot write history without characteristics. For example, the life of the
them. Of these four components, ‘place’ is people in hilly regions is more strenuous
related to geography or geographical than that of the people on the plains. Not
conditions. In this sense, history and much fertile land is available in the hilly
regions, while in the plains, it is available
on a large scale. That is why, grains and
Let’s discuss. vegetables are scarce in hilly areas.
Ɣ What occupations are seen in In comparison, people on the plains get
your locality? them in a sufficient measure. Naturally,
Ɣ What crops are grown in your this has an impact on the diet of the people.
surroundings? In hilly regions, people depend more on
hunting and gathering for their food. We
geography are inseparable. History is also find other such differences in the
influenced by geographical conditions in lifestyle of the people of the hilly regions
many ways. and those of the plains.
1
Telegram Channel -: Mpsc Super Batch 2 Compiled by Sanjay Pahade 2
The climate, rainfall, agricultural
produce, flora and fauna of the region
where we live are the sources of our
livelihood. The lifestyle and culture of a
region develops with their support. Human
settlements have flourished wherever the
means of living are plentiful. Over a period
of time, these settlements develop further
into villages and towns. But sometimes
Himalaya
reasons like the degradation of the
1. Himalayas : The Hindukush and
environment, drought, invasions, etc. lead Himalaya ranges have created an
to the scarcity of those means. People are impenetrable wall on the northern side of
forced to leave their settlements. Villages the Indian subcontinent. This wall has
and towns become deserted. We see many separated the Indian subcontinent from the
such instances in history. Thus we see that deserts of Central Asia. However, there is a
there is a very close relation between history land route through the Khyber and Bolan
and geography. passes in the Hindukush mountains. This
route was connected to an ancient trade
1.2 Geographical features of India
route. The trade route from China passed
Our country India extends far and wide. through Central Asia and reached Arabia. It
At its north lie the Himalayas; to the east, is known as the ‘Silk Route’ or ‘Silk Road’,
the Bay of Bengal; to the west the Arabian because silk was the main commodity
Sea; and to the south, the Indian Ocean. exported to the western countries using this
Except for the islands of Andaman - road. The route through the passes was used
Nicobar and Lakshwadeep, the rest of the by many foreign invaders to enter ancient
India. Many foreign travellers also came to
country is contiguous.
India by this route.
We have to take into account this
region, henceforth referred to as ‘ancient
India’, when we study the ancient history of
India. Before 1947, today’s Pakistan and
Bangladesh were also a part of India.
The following regions are seen to be
important when we look at the course of
Indian history.
1. The Himalayas
2. The plains of Sindhu-Ganga- Khyber Pass
Brahmaputra rivers 2. The plains of the Sindhu-Ganga-
3. The Thar Desert Brahmaputra : This region consists of the
4. The Deccan Plateau basins of the three big rivers, Sindhu,
5. The coastal regions Ganga, Brahmaputra and their tributaries.
6. The islands in the seas This region extends from Sindh-Punjab in
2
Telegram Channel -: Mpsc Super Batch 3 Compiled by Sanjay Pahade 3
Arabian Sea to its west, the Indian Ocean
to its south, and the Bay of Bengal to its
east. A region thus bound by the sea on
three sides is called a peninsula. A major
part of the Indian peninsula is occupied by
the Deccan Plateau.
The mountain ranges of the Vindhya
and Satpuda are located to the north of the
Deccan Plateau. The Sahyadri mountain
River Ganga ranges are to its west. They are also known
the west to the present day Bangladesh in as the ‘Western Ghats’. To the west of the
the east. It was in this region that the earliest Sahyadris is the coastal region of Konkan
Indian civilization of Harappa and the later and Malabar . The mountains on the eastern
States and empires of ancient India side of the Deccan plateau are known as the
emerged. ‘Eastern Ghats’. Deccan Plateau has fertile
land where many post-Harappan agrarian
cultures flourished. Deccan Plateau was a
part of the Maurya empire, the largest in
ancient India. After the decline of the
Maurya empire, too, several other kingdoms
and smaller empires continued to flourish
in this region.
5. The coastal regions : From the time
Thar Desert of the Harappan civilization, ancient India
3. The Thar Desert : The Thar desert had trade relations with the western
spreads across Rajasthan, Haryana and countries. This trade was carried on by sea.
some parts of Gujarat. A part of the desert Therefore, India had developed contact and
lies in today’s Pakistan. The desert has the interaction with foreign cultures and people
Satluj river to its north, the Aravalli at the sea ports. Later on, land routes came
mountain range to its east, the Rann of to be used for trade and transport. But the
Kachchh to its south and the Indus (Sindhu) importance of sea routes did not diminish.
river to its west. The Ghaggar river that 6. The islands in the sea : Andaman
originates in Himachal Pradesh reaches the and Nicobar are the Indian islands in the
Thar desert. It is known as ‘Hakra’ in Bay of Bengal. Lakshadweep is a group of
Pakistan. Its course in Rajasthan and Indian islands in the Arabian Sea. The
Pakistan has now dried up. Many sites of location of these islands may have been
the Harappan civilization are situated along
important in ancient sea trade. The
the now dry course of the river.
manuscript ‘Periplus of the Erythrean Sea’
4. The Deccan Plateau : The region or ‘Handbook of the Red Sea’ makes a
between the east and the west coast of India mention of Indian islands. It has been
tapers off to the south. This region has the written by an unknown Greek sailor.

3
Telegram Channel -: Mpsc Super Batch 4 Compiled by Sanjay Pahade 4
You can do this.
Show the following on an outline
map of India.
1. The Himalaya mountain
2. The Thar Desert
3. East Coast

Andaman island

4
Telegram Channel -: Mpsc Super Batch 5 Compiled by Sanjay Pahade 5
1.3 The Indian subcontinent
The cities of Harappa and civilization had spread mainly in the
Mohen-jo-daro are in today’s Pakistan. northwestern part of the Indian subcontinent.
Afghanistan, Pakistan, Nepal, Bhutan, China and Myanmar, our neighbouring
Bangladesh, Sri Lanka and India together countries, are not a part of South Asia or
form the region known as South Asia. the Indian subcontinent. However, they had
Considering the expanse and significance cultural and trade relations with ancient
of India in this region, it is also known as India. They have an important place in the
the Indian subcontinent. The Harappan study of ancient Indian history.

Exercise

1. Answer the following in one sentence. 5. Observe the physical map of India and
(1) What is history ? answer the questions based on it.
(2) Where do human settlements flourish? (1) Which mountain ranges lie in the north
(3) What do the people in the hilly region of India ?
depend on for their food ? (2) Which are the routes to India from the
(4) Which is the earliest civilization in northeast ?
India ? (3) Where do the Ganga and Brahmaputra
meet ?
2. Answer the following questions in short.
(1) What does human life depend on ? (4) Name the islands to the east of the
Indian peninsula.
(2) What features of our surroundings
form the means of our livelihood ? (5) In which part of India do we see the
Thar Desert ?
(3) Which region is known as the Indian
Activity :
subcontinent ?
(1) Obtain information about the lakes or
3. Give reasons. reservoirs in your locality.
(1) There is a close relation between (2) Show the following on a world map :
history and geography. 1. The Himalaya Mountain
(2) People are forced to leave their 2. The Silk Road
settlements. 3. Arabia
4. Explain the difference in the human life ***
in the hilly regions and that on the plains.

Various costumes
5
Telegram Channel -: Mpsc Super Batch 6 Compiled by Sanjay Pahade 6
2. Sources of History

2.1 Material sources designs on ancient potsherds. The ornaments


2.2 Written sources and other artefacts throw light on social
2.3 Oral sources interaction. We get information about the
2.4 Sources of ancient Indian history diet of the people from the remains of
2.5 Precautions to be taken while foodgrains, seeds and bones of animals
writing history
found in excavations. Also, we find the
Try this. remains of houses and buildings built in
different historical periods. We also find
z Make a list of things in your house ancient coins and seals. All these give us
which belong to your grandparents’ information about human activities in the
time.
past. All such artefacts, articles,
z Collect information about an old
monuments or their ruins are called the
structure in your locality.
‘material sources’ of history.
A number of objects used by our
ancestors still exist. Some inscriptions from
the past have been recovered. Sources like Do you know ?
these help us to understand our history. Foodgrains do not last long.
History can also be learnt through customs They get easily infested and turn into
and traditions, folk arts, folk literature as powder.
well as historical documents. All these are In ancient times, people used to
known as ‘sources of history’. These are of roast foodgrain before grinding
three types : material sources, written them. If a few grains got charred
sources and oral sources. while roasting, they were thrown
away. Such charred grains can
Can you tell ? remain intact for a very long period.
During excavations, such charred
Structures such as forts, rock-cut grains can be recovered. When these
caves, stupa, etc. are known as are inspected in a laboratory, we
material sources of history. Try to come to know what grain it is.
guess what other structures can be
called material sources.

2.1 Material Sources


Man uses a variety of things in
his day-to-day life. They are known as
artefacts. Artefacts used by humans in the
past provide us valuable information about
life in the ancient times. We can decide the
period of ancient artefacts like earthern
pots by studying the shape, colour and Coins
6
Telegram Channel -: Mpsc Super Batch 7 Compiled by Sanjay Pahade 7
2.2 Written sources
The Stone Age people have recorded
many events and expressed their emotions
in paintings on rocks. It was only after
thousands of years that man learnt the art
Potsherd Ornaments of writing.
In the beginning, humans used symbols
and signs to record their thoughts and ideas.
However, it took thousands of years for a
script to develop from these.
Earthen Pots In the beginning, man wrote on
potsherds, unbaked bricks using pointed
Do you know ? objects. He also used birch (bhurja) tree
Written sources of history include bark to write on. As his experience and
inscriptions carved on temple walls, knowledge increased, he began to write in
walls of rock-cut caves and rocks; different ways. He began to record the
inscriptions on copper-plates, pots, events that took place around him. Accounts
bricks and on writing sheets made from of the proceedings of royal courts were also
palm leaves and birch bark, etc. written systematically. Many rulers had
their orders, judicial decisions, donations
etc inscribed on stone or copper-plates.
Over a period of time, many forms of
literature developed. They included books
on religious and social subjects, plays,
poetry, travelogues and scientific works.
All this literature helps us to understand the
history of the various historical periods.
These sources are known as the ‘written
sources of history’.

Do you know ?
A copper-plate Bhurjapatra means writing sheets
made from the bark of a birch tree.
Bhurja is the
Sanskrit name of the
birch tree. Birch
trees are found in
Kashmir.

Inscription on a rock Birch (Bhurja) tree Bhurjapatra

7
Telegram Channel -: Mpsc Super Batch 8 Compiled by Sanjay Pahade 8
Telegram Channel -: Mpsc Super Batch 9 Compiled by Sanjay Pahade 9
Do this. Do you know ?

z Visit a museum in your locality.


Write an essay describing the zAn owi
artefacts displayed there.
,>#ʼn2?,&>_ē§‹0%0>@.1>_
z Collect the traditional verses such as
‘owis’.
9>$7>2@1G4>_,A#?44>ž1>1>__
z Collect folk songs. Present one of
zLines from a folk song
them in the cultural programme of
your school. Ǹ0;>*2@ *@™  ™
(Ujjain)
4
,A™17>*(>*@
2.3 Oral sources
&G'G*>(&;K&>2>>
Owis, folk songs, folk tales and
similar literature is passed on from :A@;K&@Ð>
generation to generation by word of ?&ž;@4K½>>7>>
mouth. This literature is not in the written :> *@> É>0™2>>ǹ™(Vikramaditya)
form. Its authors are unknown. This type
of literature preserved by oral tradition
forms the ‘oral sources’ of history.

Do you know ?

Sources of Ancient Indian History

Material Sources Written Sources Oral Sources

Artefacts Structural remains zInscriptions on Harappan seals The Vedic, Buddhist


Rock paintings Caves zVedic literature and Jain literature of
ancient India was
Pottery Houses zMesopotamian clay tablets
preserved by oral
Terra cotta objects Stupas zManuscript copies of
tradition. It is now
Mahabharata and Ramayana
Beads Rock-cut caves available in the
zJain and Buddhist literature
Ornaments Temples written form. Even
zWritingsof Greek historions, so, the oral tradition
Sculptures Churches Travelogues is still alive. When
Metal objects Mosques zTravelogues by Chinese oral compositions
Coins Pillars travellers are used for history
Weapons zBooks on Grammar, writing, they are
Puranas and inscriptions included in the oral
sources.

8
Telegram Channel -: Mpsc Super Batch 10 Compiled by Sanjay Pahade 10
2.4 Sources of ancient Indian history sources like archaeological remains,
The ancient period of Indian history ancient monuments and coins found in
extends from the Stone Age to the eighth excavations.
century CE. We learn about the Stone Age 2.5 Precautions to be taken while writing
period of Indian history through history
archaeological excavations. Since scripts It is necessary to take precautions while
had not developed at that time, no written using these sources to write history. A
sources are available. However, information written document cannot be said to be
about the ancient period from 1500 BCE authentic just because it is old - it needs to
onwards can be derived from Vedic be examined critically as to who wrote it,
literature. In the beginning, the Vedas were why and when it was written. The
not in the written form. The ancient Indians conclusions drawn on the basis of various
had developed the technique of memorizing genuine documents have to be cross-
and reciting them. The Vedas were written checked and corroborated with one another.
down later on. The Vedas and the post- Such a critical analysis is very important in
Vedic literature form an important source the writing of history.
of ancient Indian history. It includes the
Brahmanas, the Upanishads, the epics What would you do ?
Ramayana and Mahabharata, the Jain and
Buddhist literature, plays, poems, stone l You have found an old coin.
and pillar inscriptions, travelogues of - Keep it to yourself.
foreign travellers etc. We also learn about - Give it to your parents.
ancient Indian history through material - Give it to a museum.

Exercise
1. Answer in one sentence. 4. Observe any coin and note the following
(1) In the past, what materials were used things.
for writing ? Inscription on the coin ...............
(2) What information is obtained from Metal used ...............
Vedic literature ? Year of the coin ...............
(3) Which literature in preserved by oral Symbol on the coin ...............
tradition ? Picture, language, shape and denomination
of the coin ...............
2. Classify the following sources as
material, written and oral sources. 5. Do you know a few things by heart ?
Present them in your group.
Copper-plate, folk tales, pottery, beads,
For example : poems, prayers, tables, etc.
travelogues, owis, inscriptions, Vedic
literature, stupa, coin, Puranas Activity :
Material Written Oral Collect pictures / photos of material and
sources sources sources written sources and exhibit them.
----- ----- ----- ***
----- ----- -----
----- ----- -----
3. Observe the picture of earthen pots and
try to make models of them.
9
Telegram Channel -: Mpsc Super Batch 11 Compiled by Sanjay Pahade 11
3. The Harappan Civilization
3.1 Harappan civilization 3.1 Harappan Civilization
3.2 Housing and town planning Archaeological excavation first began
in 1921 CE at Harappa in the Punjab on the
3.3 Seals and pots
banks of the river Ravi. That is how this
3.4 The Great Bath civilization came to be known as Harappan
3.5 Life in the Harappan civilization Civilization. It is also known as Indus
3.6 Trade Civilization.
3.7 Reasons for decline Excavations were also carried out at
Mohen-jo-daro, a place about 650 km to
the south of Harappa in the Indus valley.
Try this. There was a striking resemblance between
the remains of structures and artefacts
Show the sites of Harappan found at the two places. Similar remains
civilization on an outline map of India. have been found at Dholavira, Lothal,
Kalibangan, Daimabad, etc.

10
10
Telegram Channel -: Mpsc Super Batch 12 Compiled by Sanjay Pahade 12
Generally, the same characteristics are
seen in all the places of Harappan Can you tell ?
Civilization. These include town planning,
roads, construction of houses, drainage ● What problems regarding health
system, seals, pots and toys and burial and hygiene will arise if the drains are
customs. not covered ?

Can you tell ? The towns were divided into two or


more sections and each section had
● Describe the structure of houses in separate fortification.
your locality.
● Do they have flat roofs or tiled,
sloping roofs? Try this.
Cut a potato into half. Carve out
3.2 Housing and town planning some letters or figures on the cut surfaces
using a nail. Dip the surface into ink or
The houses and other structures in colour and then press it on a piece of
Harappan Civilization were mainly built paper. Observe what happens.
with baked bricks. In some places,
unbaked bricks and stones were also
used for construction. The houses had 3.3 Seals and pots
rooms built around a central courtyard. The seals of the Harappan Civilization
The houses had wells, bathrooms and were mainly square-shaped and made
toilets. There was an excellent drainage from a stone called steatite. These seals
system in which baked earthern conduits bear pictures of animals. They include
were used. The streets had covered drains
pictures of real animals like bulls,
built with bricks. This shows great
buffaloes, oxen, elephants, rhinos, tigers
concern regarding public health.
etc. and also of imaginary ones like the
The streets were broad and laid out unicorn. Human figures are also seen.
in a grid pattern. Houses were built in
These seals were used as stamps.
the rectangular blocks created by them.
Earthen pots of various types and
shapes have been found at the excavation

Harappan Civilization well Seals

11
Telegram Channel -: Mpsc Super Batch 13 Compiled by Sanjay Pahade 13
Do this. Observe.

Interview a person skilled in making Visit a swimming tank in your


earthen pots to learn how they are made. locality. Observe how the water in the
tank is changed.
Ɣ What type of clay is used ?
Compare a modern swimming tank
Ɣ Where do we get the clay ? to the Harappan Bath.
Ɣ How much time is needed to make
one pot ? 3.4 The Great Bath
A spacious bath has been discovered at
sites of Harappan Civilization. There are Mohen-jo-daro. The tank in the Great Bath
red terra cotta pots with patterns and was nearly 2.5 metres deep. It was 12
designs in black colour. The patterns metres long and 7 metres wide. It was lined
include fishscales, interlocking circles, with baked bricks to prevent seepage of
pipal leaves etc. water. There were steps leading down to the
The Harappan people buried their tank. There was also a provision for
dead. They used to bury earthern pots with draining, cleaning and re-filling the tank
the bodies. from time to time.

Earthenware

The Great Bath at Mohen-jo-daro

Can you tell ?

Ɣ Name the fruits and crops grown


in your area.
Ɣ What clothing do the people in
your area use ?
Ɣ Write the names of the ornaments
you know.

12
Telegram Channel -: Mpsc Super Batch 14 Compiled by Sanjay Pahade 14
3.5 Life in the Harappa Civilization
The Harappan people practised
agriculture. The excavation at Kalibangan
provides evidence of a ploughed field. The
people there grew different crops, mainly
wheat and barley. In Rajasthan, barley was
grown on a large scale while in Gujarat, it
was ragi. Peas, sesame and lentils (masoor)
were also cultivated. The Harappan people
were also familiar with cottton.
The statues, the pictures on the seals
and the remains of cloth found in the
excavations indicate that the Harappan
people used to weave cloth. The dress of A specimen of Harappan art
both men and women consisted of knee-
length cloth and an upper garment. Try this.

Visit a grocery shop nearby. From


where does the shopkeeper buy the items
in his shop? List the items.

3.6 Trade
The Harappan people carried on trade
wihin India as well as with countries
outside. The Indus Valley was known for
its high quality cotton. This cotton was
Ornaments in Harappan Civilization exported to other regions like West Asia,
Southern Europe and Egypt. Cotton cloth,
Many types of ornaments have been too, was exported. The traders of the
found in excavations. They are made of Harappan Civilization provided muslin
gold, copper, precious stones as also shells, cloth to Egypt. Silver, zinc, gemstones,
cowries, seeds etc. Men and women both rubies, deodar wood were some of the items
used multistrand necklaces, rings, armlets that were imported from Kashmir, South
and waistbands. Women wore bangles right India, Iran, Afghanistan and Baluchistan.
up to their upper arm. Trade was carried on both by land routes
A statue found at a Harappan site and sea routes. Some of the excavated seals
presents an excellent specimen of their art. bear pictures of ships. A huge dockyard has
It shows the man’s facial features very been discovered at Lothal. The trade of the
clearly. A cloak with a beautiful trefoil Harappan Civilization was carried on along
pattern is draped across his shoulder. the coastline of the Arabian Sea.
13
Telegram Channel -: Mpsc Super Batch 15 Compiled by Sanjay Pahade 15
3.7 Reasons for decline of the Harappan
Civilization
Frequent floods, invasions by foreign
tribes, decline in trade were some of the
reasons for the decline of the Harappan
Civilization. Some regions became arid due
to weakening of monsoon, drying up of
river basins, earthquakes, changes in sea-
level, etc. People migrated to other places
leading to the decline of cities.
An artist’s visualization of the
dockyard at Lothal The Harappan Civilization was a
(Reconstructed with the help of prosperous and flourishing urban
the remains) civilization. It laid the foundation of the
Indian Civilization.

Exercise

1. Answer in one sentence. 6. On an outline map of the world, show


(1) How did the civilization get the name the other civilizations that existed in
Harappa ? other parts of the world during the
(2) What patterns are seen on the Harappan period.
Harappan pots? Activity :
(1) Prepare an outline map of your school
(3) What cloth did the Harappan traders
and show the various places like a
supply to Egypt?
library, playground, computer room,
2. What will you do when you visit an etc. on the map.
ancient site to obtain more information
(2) Prepare a detailed note on the grain
about it, to conserve it, etc ?
storage system used in your family
3. Draw a picture of the Great Bath at and your locality.
Mohen-jo-daro.
4. In the following chart, fill in the details
***
regarding human life during the
Harappan period.
Major Crops Clothes Ornaments

(1) ---- ---- ----

(2) ---- ---- ----

(3) ---- ---- ----

(4) ---- ---- ----

5. Answer in one word and frame similar


questions. Write their answers.
What stone was used to make the Harappan
Harappan toys
seals?
14
Telegram Channel -: Mpsc Super Batch 16 Compiled by Sanjay Pahade 16
4. The Vedic Civilization
4.1 The Vedic literature
4.2 Family system, day-to-day life Do you know ?
4.3 Agriculture, animal husbandry, The meaning of some suktas
economic and social life
* O Almighty, make it rain in plenty.
4.4 Religious ideas
May we get a plentiful crop in our
4.5 Political system
field. May there be plenty of milk for
4.1 The Vedic literature our children.
* May the cows come to our homes and
The civilization that is based on Vedic
stay happily in their pens. May they
literature is known as Vedic civilization.
bear many calves.
The Vedas are considered to be our earliest
literature. The Vedas were composed by * Arise, O People. Darkness has
many Sages or rishis. Some vedic hymns vanished with the advent of dawn and
were composed by women. sunlight comes. Dawn has awakened
the whole world. Let us pursue our
Vedic literature is in Sanskrit. Vedic own occupations and earn riches.
literature is very rich and diverse. The
Rigveda is considered to be the first text. It Samaveda Samhita : The Samaveda
is in the form of verses. There are four provides guidance on singing Rigvedic
Vedas - Rigveda, Yajurveda, Samaveda verses to a set rhythm and tune at the time
and Atharvaveda. The Vedic texts are of yajna rites. The Samaveda has
known as Samhita. ‘Vid’ means ‘to know’. contributed in a major way in laying the
The term ‘Veda’ derives from vid. It means foundation of Indian music.
‘knowledge’. The Vedas have been
preserved through the oral tradition. The Atharvaveda Samhita : The
Vedas are also called as Shruti. Atharvaveda is named after Atharva rishi.
This Veda is different from the other three
Rigveda Samhita : Each verse of the Vedas. It is more concerned with many
Veda is known as a richa. So, the Veda things in the day-to-day life of the people. It
consisting of richas is called ‘Rigveda’. includes solutions to adversities and
Richa means a verse composed to praise a
diseases. It also provides information about
deity. When a number of richas are put
medicinal herbs. The Atharvaveda provides
together to praise a particular deity, the
guidance to a king about the way he should
poetic composition is called a sukta. The
administer his kingdom.
Rigveda consists of many suktas composed
to praise different deities. The Brahmana texts, Aranyakas and
Yajurveda Samhita : The Yajurveda Upanishads were written after the
contains mantras uttered during a yajna. composition of the Samhitas. They are also
Yajurveda provides guidance about how included in Vedic literature.
and when the different mantras must be Brahmana texts : They were composed
chanted during yajna rites. This text to provide guidance on using Vedic mantras
includes the verses or poetic incantations in yajna rituals. Each Veda has independent
and their explanation in prose. texts called Brahmanas .

15
15
Telegram Channel -: Mpsc Super Batch 17 Compiled by Sanjay Pahade 17
Aranyakas : Aranyakas consist of the be found in the Vedic literature. Yava means
reflections or thoughts expressed after ‘barley’, godhoom - wheat, vrihi - rice. The
meditating with deep concentration in
forests or wilderness. The Aranyakas are
concerned with accurate performance of
yajna rites.
Upanishads : The term Upanishad
indicates knowledge acquired by sitting at
the feet of the teacher. Human beings have
always wondered about events like life and
death. It is not easy to find answers to them.
The Upanishads discuss such serious and
profound issues.
The four Vedas, Brahmanas, Houses in the Vedic period
Aranyakas and the Upanishads were
composed over a period of one thousand Vedic people relished milk, yoghurt, ghee,
and five hundred years. During this long butter, honey. Black gram (udad), lentil
period, the Vedic civilization underwent (masoor), sesame and meat were also a
many changes. The Vedic literature is an part of their diet.
important source for the study of those People in the Vedic period used cotton
changes and of the day-to-day life in the and woolen garments. They also used
Vedic period. valkals or clothes made from barks of trees.
4.2 Family system, day-to-day life Similarly, animal skins were also used as
Joint family system was prevalent in clothing. Men and women used ornaments
the Vedic period. The grihapati was the of gold, flower garlands, strings made of
head of the family, a man responsible for beads, etc. A kind of pendant called nishka
the family. His family included his old seems to have been popular. It was also
parents, wife and children and also the used as a currency.
families of the children, the younger Singing, playing musical instruments,
brothers and their families. This family dance, board games, chariot-race and
system was patriarchal. In the initial period, hunting were the means of recreation. Their
there are references to women scholars such main musical instruments were veena,
as Lopamudra, Gargi and Maitreyi in Vedic shat-tantu, cymbals and the conch.
literature. However, slowly the restrictions Percussion instruments like damru and
on women went on increasing and their mridanga were also used.
position in the family and society became
secondary.
The houses during the Vedic period were
earthen or wattle and daub constructions.
Wattle means woven lattice of wooden strips
which is daubed with clay mixed with
cowdung. A house was called griha or shala.
The diet of the people mainly consisted
of cereals like wheat, barley and rice. From
this, they made different preparations.
Words like yava, godhoom, vrihi, etc. can Vedic musical instruments
16
Telegram Channel -: Mpsc Super Batch 18 Compiled by Sanjay Pahade 18
4.3 Agriculture, animal husbandry,
economic and social life Do you know ?
Agriculture was the main occupation ‘Horse power’ is a unit of measuring
during the Vedic period. Many oxen were power - the rate at which work is done
yoked together to plough the fields. The by an engine.
plough had iron ploughshares. The
Atharvaveda considers insects, pests and
other animals that destroy crops and
suggests certain remedies. Cowdung was
used as manure.
During the Vedic period, animals like
horses, cows and dogs had special
importance. Cows were used as means of In this period, apart from agriculture
exchange. That is why, cows were highly and animal husbandry, several other
priced. People took special care to ensure occupations had also flourished. Artisans
that their cows would not be stolen. The and people practising various occupations
horse was valued because of its speed. The were an important part of the social system.
Vedic people were skilled in taming the They formed independent organizations
horses and harnessing them to the chariots. known as shrenis or guilds. The head of
the guild was known as shreshthi. However,
Chariots in the Vedic period had spoked
slowly, the status of skilled artisans
wheels which are lighter than solid wheels. became secondary.
The horse drawn and spoked wheel chariots During this period, there were four
of the Vedic period were swift and speedy. varnas - Brahmin, Kshatriya, Vaishya and
Shudra. A varna was determined by the
occupation of the people. In the later period,
the varnas came to be determined on the
basis of birth. This gave rise to castes. The
caste system led to inequality in the society.
During the Vedic period, certain notions
about leading an ideal life had become well
established. There were four stages or four
ashramas, from birth to death. The first
ashrama was the brahmacharyashrama, or
the period of staying with the Guru to learn
from him.
The next stage after the successful
completion of brahmacharyashrama was
grihasthashrama. It was expected that
during this period, a man would carry out
his duties towards his family and society
with the help of his wife. The third stage
A chariot was vanaprasthashrama when a man was

17
Telegram Channel -: Mpsc Super Batch 19 Compiled by Sanjay Pahade 19
People in the Vedic period had thought
about how the natural phenomena occur.
Summer is followed by the rainy season
and the rainy season by winter. This is a
regular cycle in nature. The cycle in nature
and the life cycle that moved with it were
termed by the Vedic people as rita. The life
of all living beings is a part of the cycle in
nature. Disturbance in the cycle of nature
leads to calamities. Everyone should be
Guru and disciples careful not to let it happen. Nobody should
break the laws of nature. It was believed
expected to detach himself from the that following these rules was to follow
household, retire to a solitary place and dharma.
lead a very simple life. The fourth stage
was the sanyasashrama. At this stage, the
convention was to renounce all relations, Let’s discuss.
lead life in order to understand the meaning What could be the causes that
of human life, and not stay in one place. upset the cycle of nature? What
4.4 Religious ideas attempts can you make to avoid it?
In the religious ideas of the Vedic For example, what will you do for
period, forces of nature such as the sun, effective management of drinking
wind, rain, lightning, storms and rivers water when there is scanty rainfall?
were given the form of deities. Vedic
compositions pray them to become life 4.5 Political System
giving forces. The Vedic people put offerings In the Vedic period, each village had a
into the yajna fire to please them. These head known as gramani. A group of villages
offerings were called havi. Yajna is the act formed a vish, whose head was called
of offering havi into fire. Initially, the yajna vishpati. Several vish formed a jana. Later,
rites were simple. Later on, the rules of when the jana settled into a specific region,
yajna became harder and harder. The it came to be known as a janapad. The head
importance of priests went on increasing. of the jana was known as nripa or raja
(king). The main duties of the king were to
protect the people, collect taxes and run the
administration efficiently.
In order to run the administration
smoothly, the king appointed various
officers. The purohit (priest) and the
senapati of the army were officers of special
importance. The officer who collected the
taxes was known as bhagdugh. Bhag means
share. The person who collected the king’s
share of the produce of a jana was called
bhagdugh. There were four institutions who
guided the king - sabha, samiti, vidath
Yajna
and jana.
18
Telegram Channel -: Mpsc Super Batch 20 Compiled by Sanjay Pahade 20
The people of the State participated in Vedas, Smritis, Puranas and local traditions
their working. Women, too, participated in came to be known as Hinduism.
the working of sabha and vidath. The sabha Along with the Vedic stream, there
consisted of senior citizens of the State were other religious trends which took a
whereas the samiti was a general body of different position from the Vedic one,
the people. regarding yajna rites and the varna system.
Later on in the Vedic thought, Smriti We will learn more about them in the next
and Puranas were composed. After a period chapter.
of time, the religious stream based on the

Exercise

1. Name the following with reference to 4. Name the following :


the lesson. (1) The musical instruments you know
(1) Women scholars in Vedic literature ................................................. .
................................................. . (2) Two modern ornaments of women
(2) Means of entertainment in the Vedic ................................................. .
period (3) Todays means of entertainment
................................................. . ................................................. .
(3) The four ashramas
................................................. . 5. Answer in short.
(1) What foods were included in the diet
2. Right or wrong? of the Vedic people?
(1) Rigveda - Mantras uttered during (2) Why were cows priced highly ?
yajna. (3) What was a man expected to do in
(2) Samaveda - Guidance on singing a sanyasashrama ?
mantra during yajna rites.
(3) Atharvaveda - The Veda derived its 6. Write notes on :
name from Atharva rishi. (1) The religious ideas during the Vedic
period.
3. Answer in one word each.
(2) Houses in the Vedic period.
(1) The language of Vedic literature
(3) Political system in the Vedic period.
................
(2) The meaning of vid................ Activity :
(3) The head of a family. ................ (1) Interview some artisans in your
(4) The head of shrenis ................ locality and write about them.
(2) List the new words in the lesson and
find their meaning.

***

19
Telegram Channel -: Mpsc Super Batch 21 Compiled by Sanjay Pahade 21
5. Religious Trends in Ancient India
5.1 Jainism
5.2 Buddhism
5.3 Judaism
5.4 Christianity
5.5 Islam
5.6 Zoroastrianism
Towards the end of the Vedic period,
the minute details of yajna rites acquired
undue importance. Only the priestly class
had knowledge of those details. Others no
longer had the freedom to gain that Vardhaman Mahavir
knowledge. The Varna System restrictions 5.1 Jainism
became very hard in the course of time. A Jainism is one of the ancient religions
person’s social position was decided by the in India. This religion gives importance to
varna into which he was born rather than the principle of non-violence. According to
by his achievements. That is why, from the the Jain tradition, a person who reveals
Upanishad period, we see that attempts religious knowledge is known as a
were made to give a wider scope to religious Tirthankar. There have been 24 Tirthankars
thought and not restrict it only to yajna rites. in all. Vardhaman Mahavir is the twenty-
However, the thought in Upanishads fourth Tirthankar in the Jain religious
focussed on the existence and nature of the tradition.
soul. It was difficult for ordinary people to
understand it. This gave rise to different Vardhaman Mahavir (599 BCE to 527
sects that emphasized the worship of BCE)
particular deities. For example, the Shaiva There was a mahajanapada known as
sect of Shiva worshippers and the Vrijji in what is known as the State of Bihar
Vaishnava sect the of the worshippers of today. Its capital was Vaishali. Vardhaman
Vishnu. Different Puranas were written Mahavir was born in Kundagram, a part of
with reference to these deities. Vaishali. His father’s name was Siddharth,
Certain trends around the sixth century and mother’s, Trishala.
BCE made attempts to express religious Vardhaman Mahavir left his home and
thought in such a way that the common
all comforts for the attainment of
man would understand it easily. Many
knowledge. He attained enlightenment after
people realized that every person is free to
twelve and a half years of rigorous tapa.
find ways of his own upliftment. This led to
the establishment of new religions. These This knowledge was pure or keval.
religions emphatically stated that Therefore, he is also known as Kevali. He
discrimination on the basis of caste has no was called Jina or Conqueror because the
place in an individual’s upliftment. The joy derived from physical comforts and the
work of Vardhaman Mahavir and Gautama discomfort felt due to undesired things had
Buddha is of special importance among the no impact whatsoever on him. The term
proponents of new thoughts. Jain derives from the word jina. Vardhaman
20
Telegram Channel -: Mpsc Super Batch 22 Compiled by Sanjay Pahade 22
is said to be Mahavir because he had the 1. Samyak Darshan : To understand
resilience, the courage to conquer all the truth in the preaching of the Tirthankar
passions. After the attainment of knowledge, and to have faith in it.
he preached for about thirty years to explain 2. Samyak Jnan : Studying the
the essence of religion to people. To make it preaching and philosophy of the Tirthankar
easy for people to understand it, he spoke to regularly and learning its deep meaning.
them in Ardhamagadhi, a people’s
language. The religion that he expounded 3. Samyak Charitra : Strictly following
laid stress on good conduct. The essence of the Five Great Vows.
his teachings for good conduct is contained Essence of his teachings : Among the
in the Five Great Vows (Pancha teachings of Mahavir, Anekantavada is
Mahavratas) and the Three Jewels considered to be very important in the quest
(Tri-ratna). The assembly held by the for truth. It means pluralism or multiple
Tirthankar to preach to people was known viewpoints. In our quest for truth, if we
as ‘Samavasaran’ in Ardhamagadhi. focus on only one or two aspects or
Samavasaran was based on equality. People veiwpoints and draw conclusions, the whole
of all varnas had entry to it. truth is not known. Therefore, it is necessary
to pay attention to all aspects of an issue. If
The Pancha Mahavratas (The Five
people follow this, they develop tolerance
Great Vows) : These are five rules to be
towards the opinions of other people in
followed strictly.
society, and give up the attitude of stubborn
1. Ahimsa (Non-violence) : No living adherence to their own opinions.
being should be hurt, injured or harmed Vardhaman Mahavir taught the people
through one’s behaviour. that the greatness of man does not depend
2. Satya (Truth) : Every speech and on his varna, but on his excellent conduct.
action should be true. In the Vedic tradition, the doors of
3. Asteya (Non-stealing) : Asteya knowledge had slowly been closed to
means theft. Taking what belongs to others women. But Vardhaman Mahavir gave the
without their consent is theft or stealing. right of sanyas (the right to renounce the
Asteya means ‘not stealing’ anything. world) to women, too. His teachings were :
‘Love all living things’, ‘Have mercy and
4. Aparigraha ( Non-attachment) : compassion’, and ‘Live and let live’.
Man tends to accumulate property due to
greed. Aparigraha means not hoarding or 5.2 Buddhism
accumulating anything in this way. Buddhism spread in India and in many
5. Brahmacharya (Chastity) : It countries outside India. Gautama Buddha
was the founder of Buddhism.
means leaving bodily pleasures and
following the vows. Gautama Buddha (563 BCE To 483 BCE)
Gautama Buddha was born at Lumbini
Tri-ratna (The Three Jewels) : The in Nepal. His father’s name was
three jewels are the three principles. Shuddhodana and mother’s, Mayadevi. His
1. Samyak Darshan (Right Faith) 2. Samyak birth name was Siddharth. He had attained
Jhan (Right Knowledge) and 3. Samyak knowledge of human life in its entirety.
Charitra (Right Conduct) Samyak means That is why he came to be known as the
‘balanced’.
21
Telegram Channel -: Mpsc Super Batch 23 Compiled by Sanjay Pahade 23
to preach dhamma. He preached in the
people’s language, Pali. In Buddhism, the
concept of taking refuge in Buddha,
Dhamma and Sangha is important. It is
known as Trisharan. The essence of the
dhamma he expounded is as follows :
Aryasatyas (Noble Truths) : There are
four truths at the root of all human affairs.
They are called Noble Truths or Aryasatyas.
Gautama Buddha
1. Dukkha (Suffering) : There is
‘Buddha’. He wanted to know why there is suffering in human life.
sorrow and suffering in human life. He left
his home and all comforts in pursuit of the 2. The Cause of Dukkha : There is a
answer to this question. On Vaishakha cause of suffering.
Purnima, he was sitting in deep meditation
3. Dukkha-nivaran : It is possible to
under a pipal tree at Uruvela near the city
end suffering.
of Gaya in Bihar. That is when he attained
‘Bodhi’ - enlightenment or the highest 4. Pratipad : Pratipad means the ‘way’.
knowledge. The tree is now known as the This way leads to the end of suffering. This
‘Bodhi’ tree (Bodhivriksha) and the place is the way of good conduct. It is known as
Uruvela is known as Bodhgaya. He the Ashtangik or Eight-fold Path.

Panchasheel : Gautama Buddha asked


people to follow five rules. The rules are
called Panchasheel.
1. Stay away from killing animals.
2. Stay away from stealing.
3. Stay away from unethical conduct.
4. Stay away from telling lies.
5. Stay away from intoxicants.

Bauddha Sangha : He created a


sangha of bhikkhus to preach his religion.
Bodhi Tree Followers who gave up their domestic life
delivered his first sermon at Sarnath near and entered the sangha were called
bhikkhus. They, too, travelled on foot like
Varanasi. His teachings in the first sermon
the Buddha to preach dhamma to the people.
are termed dhamma. He set in motion the
There was a separate sangha of women.
wheel of dhamma in this sermon. Therefore They are called bhikkhunis. People of all
this event is called dhamma-chakka- varnas and castes could embrace Buddhism.
pavattan in Pali and dharma-chakra-
pravartan in Sanskrit. Later, he travelled Essence of his teachings : Gautama
Buddha announced the freedom of human
on foot (charika) for nearly forty-five years
22
Telegram Channel -: Mpsc Super Batch 24 Compiled by Sanjay Pahade 24
Do you know ? The tolerance preached by Gautama
Buddha is a guiding principle not only for
The Eightfold Path the Indian society but for all mankind even
1. Right View : The knowledge of the today.
four noble truths. Lokayat
2. Right resolve : Giving up cruelty, The ancient trend of thought known as
etc. Lokayat or Charvak is also important. It
3. Right speech : Refrain from telling emphasized independent thought, and
lies, telling tales, rude, harsh and rejected the authority of the Vedas.
meaningless speech. In the ancient period, new religious
4. Right conduct : Stay away from trends and thoughts went on emerging in
killing animals, stealing and India. Later on, religions like Judaism,
uncontrolled behavior. Christianity, Islam and Zoroastrianism also
5. Right livelihood : Using only the took root in the Indian society.
proper means of livelihood. 5.3 Judaism
6. Right effort : Making effort to avoid People belonging to the Jewish religion
wrong acts, giving up wrong acts, may have arrived in Kochi in Kerala around
undertaking and maintaining good the first to third century of the Christian
acts. Era. They believe that there is only one
7. Right mindfulness : Being mindful God. Judaism emphasizes justice, truth,
to remove passions and concentrating peace, love, compassion, humility, charity,
on trying to understand one’s own ethical speech and self-respect. Their prayer
feelings and mind. house is known as a synagogue.
8. Right concentration : Experiencing
deep meditation with concentration.

intelligence. He refused the discrimination


based on things like varna. No one is greater
or smaller by birth. Greatness depends on
one’s behaviour or conduct. One of his
famous quotes is that ‘Even the little quail
can chirp freely in her nest’. This shows his
thinking on the values of freedom and
equality. He preached that like men,
women, too, had the right to their own Synagogue
upliftment. He opposed the rituals of yajna. 5.4 Christianity
His teachings of wisdom (Prajna), moral
virtue (Sheel), and other values are aimed Christianity is a religion founded
at the welfare of man. Compassion (Karuna) by Jesus Christ. It has spread all over the
for all living beings was an extraordinary world. St. Thomas, one of the 12 disciples
feature of his personality. of Christ, came to Kerala in the first century

23
Telegram Channel -: Mpsc Super Batch 25 Compiled by Sanjay Pahade 25
of the Christian Era. He established a between India and Arabia. Arab traders
church at Pallayur in Trichur district, in used to visit the ports on the coast on Kerala.
52 CE. According to the teaching of Islam spread in Arabia in the 7th century
Christianty, there is only one God. He is the CE. Islam arrived in India in the same
loving father of all and is omnipotent. It is century through the Arab traders. The
believed that Jesus Christ is the son of God prayer house of Islam is known as a mosque
who came to the earth for the salvation of or masjid.
mankind. According to the teachings of
Christianity, we are all brothers and sisters.
We should love everyone including our
enemies. We should forgive those who err
or go wrong. The Bible is the holy book of
Christianity and their prayerhouse is known
as a Church.

Mosque

5.6 Zoroastrianism
Since ancient times, the Zoroastrian
people and Vedic people have shared ties.
The sacred text of the Parsee or Zoroastrian
Church people is the ‘Avesta’. The language of the
Rigveda and Avesta is similar. The Parsees
5.5 Islam came to India from the Pars or Fars province
Islam is a monotheistic religion. There of Iran. Therefore, they are known as
is only one Allah and Muhammad is his Parsees. It is mostly believed that they first
Prophet. The message of God is revealed came to Gujarat in the eighth century CE.
through him in the holy book of Quran Zarathushtra or Zoroaster was the founder
Sharif. The word Islam means peace. It of their religion. ‘Ahur Mazda’ is the name
also means surrender to Allah. The teaching of their God. The elements of fire and water
of Islam is that Allah is eternal, absolute, are very important in their religion. The
all powerful and merciful. The purpose of sacred fire burns in the temples which are
human existence is to worship Allah. The known as Agyaris. At the core of Parsee
Quran Sharif provides guidance on how thinking are three main principles of
man should behave in life. Since ancient conduct, namely, good thoughts, good
times there have been trade relations words and good deeds.

24
Telegram Channel -: Mpsc Super Batch 26 Compiled by Sanjay Pahade 26
Telegram Channel -: Mpsc Super Batch 27 Compiled by Sanjay Pahade 27
Agyari

Exercise
1. Fill in the blanks.
(1) Ahimsa (2) Samyak Darshan
(1) The principle of ............. is very
(3) Satya (4) Asteya (5) Samyak Jnan
important in Jainism.
(6) Aparigraha (7) Samyak Charitra
(2) An extraordinary feature of
(8) Brahmacharya
Gautama Buddha’s personality
is his ......... for all living beings. Five Great Vows Three Jewels
2. Answer in short. (1) ................... (1) ...................
(1) What were the teachings of (2) ................... (2) ...................
Vardhaman Mahavir? (3) ................... (3) ...................
(2) Which famous quote of Gautama (4) ...................
Buddha have you read here? What (5) ...................
values does it uphold? 5. Give reasons :
(3) What virtues are emphasized in (1) Why was the name ‘Jina’ given to
Judaism? Vardhaman Mahavir?
(4) What are the teachings of (2) Why did Gautama Buddha came
Christianity? to be called the Buddha?
(5) What are the teachings of Islam?
Activity :
(6) What is at the core of Parsee
(1) Collect the information about and
thinking?
the pictures of various festivals.
3. Write notes on :
(2) Visit the prayer houses of different
(1) Aryasatyas (2) Panchasheel religions and describe the precincts
4. Put the following in the proper place in the classroom.
in the chart of the Five Great Vows
and the Three Jewels. ***
25
Telegram Channel -: Mpsc Super Batch 28 Compiled by Sanjay Pahade 28
6. Janapadas and Mahajanapadas

6.1 Janapadas 6.1 Janapadas


6.2 Mahajanapadas The period from about 1000 BC to
600 BC is considered to be the
6.3 Rise of the Magadha Empire
post-Vedic period. It is in this period that
the janapadas came into existence.
Janapadas were the many small States
Try this. that spread from today’s Afghanistan
Write the names of the sixteen which is to the northwest of the Indian
mahajanapadas in the proper places on subcontinent to Bengal and Odisha in the
an outline map of India. east and to Maharashtra in the south.
26
Telegram Channel -: Mpsc Super Batch 29 Compiled by Sanjay Pahade 29
The janapada called ‘Ashmak’ occupied Some janapadas had a gana-parishad of
a part of today’s Maharashtra. The senior citizens. Members of the gana-
names of these janapadas can be found parishad came together for discussions
in Sanskrit, Pali and Ardhamagadhi and made decisions regarding
literature. One can also find information administrative issues. The place where
about them in the writings of Greek these discussions took place was known
historians. Some of the janapadas were as the santhagar. Gautam Buddha hailed
monarchies, while others were republics. from the Shakya Republic. Every
6.2 Mahajanapadas janapada had its own coinage.

Mahajanapadas
Kosala Vatsa Avanti Magadha

z The Kosala zThe mahajanapada z The ancient z The ancient


mahajanapada Vatsa was located kingdom of mahajanapada
was located in the in the region around Avanti was of Magadha was
foothills of the Prayag, that is located in the spread through
Himalayas in the Allahabad, in Uttar Malwa region of the regions of
region of Uttar Pradesh. Madhya Pradesh. Gaya, Patna in
Pradesh and z Kosam, the capital z Ujjayani (Ujjain) Bihar and some
Nepal. of Vatsa, was the was its capital. regions in
z The famous ancient city of z The city was an Bengal.
cities of Kosala Kaushambi. important trade z The capital of
were Shravasti, z It was an important centre. Magadha was
Kushavati and centre for trade. Rajgriha
z The king of
Saket. z Three extremely (Rajgir).
Avanti, Pradyot,
z Shravasti was rich merchants of was a z King
the capital of Kaushambi had contemporary of Bimbisara’s
Kosala. built three viharas Gautama Buddha palace was built
z Gautama Buddha for Gautama and Vardhaman by the architect
had lived in the Buddha and his Mahavir. Mahagovind.
famous vihara followers. z Jeevaka, the
z During the reign
Jetvan at z King Udayana was famous
of King
Shravasti for a a contemporary of physician, was
Nandivardhan,
long time. Gautama Buddha. at the court of
the State of
z The Kosala Avanti was Bimbisara.
z After king
king Prasenjit merged into Bimbisara had
Udayana, the State z
was a the Magadha become a
of Vatsa could not
contemporary of Empire. follower of
maintain its
the Buddha and Gautama
independent
Vardhaman Buddha.
existence for long.
Mahavir.
The king of Avanti
z Later, the State mahajanapada
of Kosala merged conquered the State.
with Magadha.

27
Telegram Channel -: Mpsc Super Batch 30 Compiled by Sanjay Pahade 30
Some janapadas gradually became and Vatsa to Magadha. Nearly the entire
stronger and expanded their geographical region of northern India came under the
boundaries. Such janapadas came to be control of Magadha. That was how the
known as mahajanapadas. From the Magadha Empire took shape.
literature of that period, it is clear that The Nanda Kings of Magadha : The
up to the sixth century BCE, sixteen Nandas ruled the Magadha Empire
mahajanapadas had acquired special between 364 BCE and 324 BCE. They
importance. Kosala, Vatsa, Avanti and had set up a good administrative system
Magadha among them became more necessary to run the huge empire. They
powerful. had a huge four-column army of infantry,
6.3 Rise of the Magadha Empire cavalry, chariots and elephants. The
Ajat shatru, the son of Bimbisara, Nandas also introduced the system of
continued with the policy of expansion standard weights and measures.
of the Magadha Empire. He successfully King Dhananand was the last king
conquered many republics of the east. of the Nanda dynasty. By this time, the
The kingdom of Magadha prospered Magadha Empire had extended up to the
during the reign of Ajatshatru. He had Punjab in the west. However, during
become a follower of Gautama Buddha. Dhananand’s reign, the ambitious youth
After the Mahaparinirvana of Gautama Chandragupta Maurya won Pataliputra,
Buddha, it was during his reign that the ended the Nanda regime and laid the
first Buddhist Council or Sangiti was foundation of the Maurya Empire.
held at Rajgriha. In the next chapter, we will read
about the foreign invasions on the western
and northwestern frontiers of India during
the rise of the Maurya Empire. Also, we
will read about the Maurya Empire in
greater detail.

Do you know ?
The ancient and modern names of the
16 mahajanapadas :
(1) Kashi (Benaras), (2) Kosal
(Lucknow), (3) Malla (Gorakhpur),
Ajatshatru Sculpture (4) Vatsa (Allahabad), (5) Chedi (Kanpur),
The foundation for the new capital (6) Kuru (Delhi), (7) Panchal (Rohilkhand),
Pataligram of Magadha was laid during (8) Matsya (Jaipur), (9) Shurasen
Ajatshatru’s period. Later, it became (Mathura), (10) Ashmak (Aurangabad,
famous as ‘Pataliputra’. Pataliputra was Maharashtra), (11) Avanti (Ujjain),
probably in the vicinity of today’s
(12) Ang (Champa East Bihar),
Patna city.
(13) Magadha (South Bihar), (14) Vrijji
A noteworthy successor of Ajatshatru
was the Magadha king Shishunag. He (North Bihar), (15) Gandhara (Peshawar),
annexed the kingdoms of Avanti, Kosala (16) Kamboj (Near Gandhara)

28
Telegram Channel -: Mpsc Super Batch 31 Compiled by Sanjay Pahade 31
Exercise

1. Answer in one sentence. (4) Gautama Buddha belonged to this


(1) What is meant by janapada ? janapada –
(2) What is meant by mahajanapada ? (5) They had a four-column army –

(3) Where was the first Buddhist Council 3. Match the following.
held ? Group A Group B
(4) Who introduced the system of standard (1) Sangiti (a) Ajatshatru
weights and measures ? (2) Dhananand (b) Parishad
2. Can you name the following ? (3) Pataligram (c) Mahagovind
(1) Some part of today’s Maharashtra (d) Nanda King
was occupied by this janapada – 4. Make a table showing the various
(2) This was an assembly of the senior kingdoms in the Indian sub-continent
citizens of a janapada – and their capitals.
(3) This was the venue of discussions –

Activity :
(1) Visit a nearby fort and find out the following :
(1) Type of the fort (2) In whose reign it was built (3) The Killedar...........
(4) The main feature.
(2) Find out the different arms and services of the Indian Army.
(3) Complete the following table :

Name of the Name of the most


S.No. Location Capital
mahajanapada important king
At the foot of the
1. ----- ----- -----
Himalayas

2. Vatsa ----- ----- -----

3. ----- ----- ----- Pradyot

Region around the


4. ----- ----- -----
cities of Patna, Gaya.

***

29
Telegram Channel -: Mpsc Super Batch 32 Compiled by Sanjay Pahade 32
7. India during the Maurya Period
7.1 The Greek Emperor Alexander’s
invasion
7.2 The Maurya Empire

Do you know ?

In the 6th century BCE, a king


named Cyrus had established a vast
empire in Iran. This empire extended
from Northwest India to Rome and to
Egypt in Africa. Around 518 BCE, an
Iranian Emperor named Daryush had Emperor Alexander
conquered the region to the northwest 326 BCE. Crossing the Indus (Sindhu)
of India up to Punjab. Daryush had river, he reached Takshashila. On the
recruited some soldiers from this area way, he met with forceful opposition
into his army. We learn about this from the local Indian kings. He defeated
from the writings of Greek historians. them all and successfully reached the
Political relations between India and Punjab. However, in this invasion, his
Iran were established during the reign army had to suffer severe hardships. The
of Emperor Daryush. This led to soldiers were eager to go back home.
greater exchanges in the fields of They rebelled against Alexander and he
trade and art. Emperor Daryush had was forced to turn back. He, therefore,
introduced a uniform currency called appointed Greek officers to administer
‘Darik’ in all parts of his empire. This the conquered territories. They were
made trading easier. The capital city called satraps. He started his return
of Persepolis was built during his journey but died on the way back in
reign. Persepolis is in Iran. 323 BCE at Babylon. Today, Babylon
is in Iraq.
Alexander’s campaign led to an
increase in the trade between India and
the western world. The historians who
accompanied Alexander introduced India
to the western world through their
writings. Greek sculpture influenced
Indian art, giving rise to the Gandhara
school of art. The Greek kings minted
Darik characteristic coins. On one side, the
coin had a picture of the king who had
7.1 The Greek Emperor Alexander’s minted the coin and on the other side, a
invasion picture of a Greek god. The name of the
Alexander, the Greek emperor, king was also written on the coin.
invaded India’s northwestern frontier in Alexander’s coins were of the same type.

30
30
Telegram Channel -: Mpsc Super Batch 33 Compiled by Sanjay Pahade 33
Megasthenes, Seleucus Nicator’s
ambassador stayed back at Chandragupta
Maurya’s court. His book ‘Indica’ is an
important source for the study of India
during the Maurya period.
There is an inscription stating that
Emperor Chandragupta Maurya had built
a dam called ‘Sudarshan’ near Junagadh
Alexander’s silver coin - both sides in Gujarat State.
Later, Indian kings also started minting
similar coins. Do you know ?
7.2 The Maurya Empire According to the Jain tradition, it
Chandragupta Maurya : is believed that Chandragupta Maurya
Chandragupta Maurya founded the had accepted the Jain religion. Towards
Maurya Empire. People were tired of the the end of his life, he abdicated the
tyrannical rule of the Nanda king of throne and spent his remaining years
Magadha, Dhanananda. Chandragupta at Shravanabelagola in Karnataka.
Maurya brought it to an end and That was where he breathed his last.
established his own rule in Magadha
around 325 BCE. He won Avanti and Emperor Ashoka : After
Saurashtra and began to extend the Chandragupta renounced the throne, he
boundaries of his empire. After was succeeded by his son Bindusara.
Alexander’s death, a struggle for power After Bindusara’s death, his son, Ashoka
began among the satraps he had
succeeded to the throne in 273 BCE.
appointed. Seleucus Nicator was
Alexander’s General. He became the Ashoka had been appointed the Governor
King of Babylon after Alexander’s death. of Takshashila and Ujjain before
He invaded the northwest frontier region he came to the throne. As the Governor,
of India and the Punjab. Chandragupta he had successfully crushed the revolt at
Maurya successfully resisted his Takshashila. After becoming the Emperor
aggression. The defeat of Seleucus of Magadha, he launched a campaign
Nicator led to the inclusion of the against the State of Kalinga. Kalinga
northwest region of Kabul, Kandahar occupied the region of today’s Odisha
and Herat in the Maurya Empire. State. Emperor Ashoka conquered
Kalinga.
Do you know ?
Ashoka’s empire extended from
Vishakhadatta, the Sanskrit Afghanistan in the northwest and Nepal
playwright, wrote a play called in the north to Karnataka and Andhra
‘Mudrarakshasa’. It relates how Pradesh in the south, and from Bengal
Chandragupta Maurya defeated
in the east to Saurashtra in the west.
Dhanananda and established an
independent power. The plot gives The Kalinga War : Ashoka was
special importance to the contribution of deeply moved by the bloodshed of the
Arya Chanakya, also known as Kautilya. Kalinga war. He decided never to wage

31
Telegram Channel -: Mpsc Super Batch 34 Compiled by Sanjay Pahade 34
Always remember...

The message of Emperor Ashoka


zIt is good and right to serve one’s Try this.
parents.
zThe victory which arouses Mark on an outline map of India, the
compassion and love is the real places where inscriptions and edicts
victory. of Emperor Ashoka are located.

32
Telegram Channel -: Mpsc Super Batch 35 Compiled by Sanjay Pahade 35
a war again. He came to believe that Sri Lanka for the spread of Buddhism.
truth, non-violence, compassion and He also sent Bauddha bhikkhus to West
forgiveness were the important values. Asia and Central Asia for spreading
Ashoka wanted to spread these teachings Buddhism. He also built many stupas
among the common people and therefore, and viharas.
he had pillar edicts and stone inscriptions
engraved throughout his kingdom. These Try this.
writings are in the Brahmi script. In Prepare a report about the
these inscriptions, he is referred to as public welfare activities carried out
‘Devanampiyo Piyadasi’ (Beloved-of the- by the institutions in your locality.
Gods and He who Looks on with
Affection). Another inscription states that Public welfare activities : Emperor
eight years after he ascended the throne, Ashoka laid stress on the creation of
he made the conquest of Kalinga and facilities for the welfare of the people.
had a change of heart on seeing the For example, he made provisions for free
devastation it caused. medicines and medical treatment for
An inscription of Emperor Ashoka at people as well as for animals. He built
Delhi-Topda shows that he had put strict many roads and planted trees for shade
restrictions on the hunting of bats, on both sides of the roads. He dug wells
monkeys, rhinoceroses and on setting fire and built dharmashalas.
to forests. Administration during the Maurya
period : Pataliputra was the capital of
the Maurya empire. For the sake of
administrative convenience, the empire
was divided into four regions, each with
its own capital.
1. Eastern region - Toshali (Odisha)
2. Western region - Ujjayani
(Madhya Pradesh)
3. Southern region - Suvarnagiri
(Kanakgiri in Karnataka)
4. Northern region - Takshashila
(Pakistan)
There was a Council of Ministers
which advised the King in matters of
administration. There were many officers
working at different levels. There was an
Emperor Ashoka
efficient intelligence service which kept
Emperor Ashoka’s work for the an eye on the administration and on the
spread of religion : Ashoka had movements of enemies.
embraced Buddhism. He had convened Life of the people during the
the Third Religious Council on Buddhism Maurya period : During the Maurya
at Pataliputra. Ashoka sent his own son period, agricultural production had great
Mahendra and daughter Sanghamitra to importance. Trade and other occupations
33
Telegram Channel -: Mpsc Super Batch 36 Compiled by Sanjay Pahade 36
had also prospered well. There were
many occupations like carving and Do you know ?
engraving on ivory, weaving and dyeing
cloth, and metal work. Black, glazed The official seal of the
pottery was also produced. Ship building Government of India is based on the
was carried out on a large scale. In capital or pillarhead of the Ashoka
metal work, the technology of making pillar at Sarnath. The Ashoka pillar
articles of other metals as well as those has four lions. On a horizontal strip
of iron had developed. below each lion, there is a wheel or
chakra. We can view only one of
Different festivals and functions were
these chakras at a time. There is a
celebrated in villages and towns. Dance
horse on one side and a bull on the
other side of the chakra. Similarly, on
the side that is not in view on the
seal, there is an elephant and a lion
on either side of the chakra.

and music concerts were a part of


entertainment. Wrestling bouts and chariot
races were popular and so were board
games and chess. Chess was called
Ashtapad.
Art and Literature : During the period
of Emperor Ashoka, the art of rock cut
sculpture was promoted. The pillars erected
by Ashoka are excellent specimens of
Indian sculpture. There are excellent
carvings of animals like the lion, elephant,
bull on these pillars. The chakra or wheel
The Lion Capital on the Ashoka pillar at Sarnath has a place

A cave at
Barabar

34
Telegram Channel -: Mpsc Super Batch 37 Compiled by Sanjay Pahade 37
of honour on the Indian national flag. The After the death of Emperor Ashoka, the
pillar has lions on all four sides but only Maurya Empire began to decline. After the
three are visible from the front. This is the Mauryas, many new States and some
official seal of the Government of India. empires also emerged in India. The Maurya
The cave sculptures at Barabar Hills carved Empire was the largest empire in ancient
during the period of Ashoka are well- India. In the next chapter, we shall discuss
known. These caves are in Bihar. They are the political and cultural happenings in the
the oldest cave sculptures in India. post-Maurya period.

Exercise

1. Answer in one sentence. 5. What is your opinion?


(1) Why did the satraps begin to fight (1) Alexandar had to turn back.
among themselves ? (2) Coins of Greek kings were
(2) Who did Ashoka send to Sri Lanka for characteristic.
the spread of Buddhism ? (3) Emperor Ashoka decided never to
(3) What were the occupations of the wage war again.
people during the Maurya period ?
6. Describe in your own words.
(4) Which animals’ engravings are found
(1) Public welfare works of Emperor
on the pillars erected by Emperor
Ashoka.
Ashoka ?
(2) Means of entertainment and sport of
2. Can you explain ? the Maurya period.
(1) Satrap
Activity :
(2) Sudarshan
(1) Find out about the public welfare
(3) ‘Devanampiyo Piyadasi’
activities carried out by the people’s
(4) Ashtapad representatives in your area and
3. Recall and describe : write in detail about them.
(1) The extent of Chandragupta Maurya’s (2) Read more about the life of Emperor
empire. Ashoka and make a presentation
based on it in the form of a play.
(2) The extent of Ashoka’s empire.
4. Match the following.
***
Group A Group B
(1) Emperor (a) Ambassador of
Alexander Seleucus Nicator
(2) Megasthenes (b) Emperor of Greece
(3) Emperor (c) Emperor of
Ashoka Rome
(d) Emperor of Magadha

35
Telegram Channel -: Mpsc Super Batch 38 Compiled by Sanjay Pahade 38
8. States after the Maurya Empire

8.1 The Shunga dynasty 8.1 The Shunga dynasty


8.2 Indo-Greek kings After Emperor Ashoka, Maurya power
8.3 Kushana kings started declining. The last Maurya Emperor
8.4 The Gupta dynasty was called Brihadratha. The Maurya
8.5 The Vardhan dynasty General Pushyamitra Shunga revolted
against Brihadratha, killed him and
8.6 Powers in Northeast India
became the king himself.
36
36
Telegram Channel -: Mpsc Super Batch 39 Compiled by Sanjay Pahade 39
8.2 Indo-Greek kings Emperor Kanishka : Kanishka’s
During this period, there were several empire extended from Kabul in the west
small kingdoms ruled by Greek kings to to Varanasi in the east. Gold and copper
the northwest of the Indian subcontinent. coins minted by him have been found in
Those kings are known as Indo-Greek North India. The fourth Buddhist
kings. In the history of the coins of ancient Council was held in Kashmir during his
India, the coins of these kings are very reign. He established the city of
important. They had a tradition of putting Kanishkapur in Kashmir. It is believed
the picture of the king on one side and that the village of Kampur near Srinagar
that of a deity on the other side. This today could be Kanishkapur.
tradition later took root in India. One of The well-known poet Ashvaghosh
the famous Indo-Greek kings was lived during the reign of Kanishka. He
Menander, who discussed Buddhist wrote the texts ‘Buddhacharita’ and
philosophy with the Buddhist bhikkhu, ‘Vajrasuchi’. The famous vaidya
Nagasena. Menander is also referred to as Charaka was also in Kanishka’s court.
‘Milinda’. The questions that he discussed
with Bhikkhu Nagasena led to the creation
of the book ‘Milind Panha’. The Pali word
‘panha’ means ‘question’.

A gold coin of Kanishka - both sides


A silver coin of Menander - both sides

8.3 Kushana kings Do you know ?


India was invaded from time to time
Kanishka’s gold coin : It was
by several tribes. The Kushanas were one
minted by Emperor Kanishka. It has
such tribe from Central Asia. They
the words ‘Shao Nano Shao
established their rule in the northwestern
Kaneshki Koshano’ on one side. It
region and in Kashmir in the first century
means ‘King of kings, Emperor
CE. The Kushana kings were the first to
Kanishka Kushana’. On the other
start minting of gold coins in India. They
side, there is an image of Gautama
started the custom of putting the images
Buddha and the word ‘Boddo’,
of Gautama Buddha and different Indian
meaning Buddha, written in the
deities on the coins. The Kushan king
Greek script.
Kanishka extended their empire.
37
Telegram Channel -: Mpsc Super Batch 40 Compiled by Sanjay Pahade 40
8.4 The Gupta Dynasty Chandragupta II : After
The end of the third century CE saw Samudragupta, his son Chandragupta II
the rise of the Gupta dynasty in North ascended the throne. He extended the
India. The Guptas remained in power for Gupta empire towards the northwest. He
nearly three centuries. ‘Shrigupta’ was the also won Malwa, Gujarat and Saurashtra.
founder of the Gupta dynasty. He established good relations with the
Samudragupta and Chandragupta II were powerful Vakataka rulers in the south by
the notable kings of the Gupta dynasty. giving his daughter Prabhavati in marriage
Samudragupta : The expansion of to Rudrasen II.
the Gupta empire began during the reign There is an iron pillar at Mehrauli
of Chandragupta I. His son, Samudragupta, near Delhi. It has not rusted even in the
defeated the smaller neighbouring course of the last fifteen hundred years. It
kingdoms and extended the empire further. is an excellent specimen of the metallurgical
In his time, the Gupta empire spread from skill of the ancient Indian people. The
Assam upto the Punjab. He had also inscription on the pillar bears the name of
conquered the eastern coastal region up to a king called ‘Chandra’. It is on this basis
Kanchi in Tamilnadu. Due to these that the iron pillar is assumed to be of the
victories, his power came to be recognised period of Chandragupta II.
everywhere. As a result, kings on the
northwestern frontiers as well as those in
Sri Lanka made treaties of friendship with Do you know ?
him. A pillar inscription at Prayag The Chinese traveller Fa Hien
describes Samudragupta’s conquests and came to India during the reign of
victories. This inscription is known as Chandragupta II. In his travelogue, he
‘Prayag-prashasti’ and also as ‘Allahabad has described the social life during the
Prashasti’. He was an expert veena player. Gupta period. He says that Indian
He minted coins which had a variety of cities are big and prosperous. There
images engraved on them. In one of them, are several guest houses for travellers
he is seen playing the veena. His name and also several charitable
Samudragupta, is engraved on it. organizations. The city has hospitals
where the poor get medical treatment
free of charge. There are great vihars
and temples. People are free to choose
any occupation. They move about
freely - there are no restrictions on
their movement. Government officers
and soldiers are paid their salaries
regularly. People do not drink alcohol
or commit violence. The administration
of the Gupta rule is conducted in a
A gold coin of Samudragupta - both sides proper and efficient manner.

38
Telegram Channel -: Mpsc Super Batch 41 Compiled by Sanjay Pahade 41
In this period, the Bauddha bhikkhu, all parts of India. He stayed at the
Fa Hien, came to India from China. Nalanda University for two years. On
He has written an account of his travels returning to his homeland, he translated
in India. From his writings, we learn many Buddhist manuscripts into Chinese.
about the efficient administration of the
Gupta emperors.
8.5 The Vardhan Dynasty
With the decline of the Gupta power,
many States emerged in North India. The
Vardhan dynasty was one of them.
Prabhakar Vardhan was the king of
Thanesar, near Delhi. The Vardhan
dynasty became powerful during his reign.
His son, Harshavardhan, expanded the
Vardhan Empire up to Nepal in the north,
up to the river Narmada in the south,
Assam in the east and Gujarat in the west.
He had cordial relations with Raja
Bhaskarvarman of Kamrup, i.e., ancient
Assam. He had also established friendly
relations with the Emperor of China and
even sent his ambassador to the Chinese
court.
The capital of Harshvardhan’s
Empire was Kanauj. Trade flourished
during his reign. He spent a large portion
of his revenue for the welfare of the
people. Every five years, he would Yuan Chwang
distribute all his wealth amongst the
people. Do you know ?
The court poet Banabhatta wrote
Yuan Chwang travelled all over
‘Harshacharita’, a biography of Emperor
India. He has words of praise for the
Harshavardhan. This text provides
people of Maharashtra. He writes,
information on the life and achievements ‘The people of Maharashtra are a
of Harshavardhan. Harshavardhan had proud people. They never forget a
become a follower of Buddhism but gave favour done to them but they do not
generous patronage to other religions, spare anyone who insults them. They
too. He wrote three Sanskrit plays will help another in distress without a
‘Ratnavali’, ‘Naganand’ and care even for their own life. They do
‘Priyadarshika’. The Buddhist bhikkhu, not harm anyone who takes refuge
Yuan Chwang had come to India from with them.’
China during his regime. He travelled to
39
Telegram Channel -: Mpsc Super Batch 42 Compiled by Sanjay Pahade 42
Nalanda University

8.6 Powers in Northeast India In the book ‘Periplus of the Erythrean


A story in Mahabharata tells about Sea’, Kamrup is mentioned as ‘Kirhadiya’
the marriage of Arjuna and Ulupi - the or ‘the region of the Kirat people’. The
princess of Manipur State in East India. Kamrup kingdom extended in the
Brahmaputra river basin, Bhutan, some
The State of ‘Kamrup’ emerged in the
parts of Bengal and Bihar. During the
fourth century CE. It was established by
reign of king Bhaskarvarman, Yuan
Pushyavarman. His name has been
Chwang had visited ‘Kamrup’.
mentioned in the pillar inscription of
In this chapter, we learnt about the
Samudragupta at Allahabad. Many
different kingdoms in North India that
inscriptions of the Kamrup kings are
emerged in the period after the Mauryas.
available. The epics Mahabharata and Similarly, we also learnt about the
Ramayana use the name ‘Pragjyotish’ for situation in the northeastern part of India
Kamrup. The capital of that State was during that period. In the next chapter, we
‘Pragjyotishpur’. Today, we know it as will get acquainted with the kingdoms of
the city of Guwahati in Assam. the south of the same period.

Do you know ?
According to the Indian tradition, Kashmir was known as Kashyapapur in
ancient times. Greek historians have mentioned it by the names of Kaspapyros,
Kaspatyros and Kasperia. There is a mention that the Kamboj dynasty ruled there
during the period of the Mahabharata. During Emperor Ashoka’s period, Kashmir
had become a part of the Maurya Empire. In the 7th century CE, Kashmir was
ruled by the Karkot dynasty. Kalhan has written about it in his book ‘Rajtarangini’.

40
Telegram Channel -: Mpsc Super Batch 43 Compiled by Sanjay Pahade 43
Exercise
1. Can you tell ? 7. Solve the puzzle.
(1) The kings who started the minting of 1 2 3 4

gold coins in India.


5
(2) The city established by Kanishka in
Kashmir.
(3) The king who played the veena.
(4) Another name for Kamrup.
2. Observe the map in the lesson. List the 6
names of the modern cities which were
part of the Gupta Empire.
7
3. Discuss and write.
(1) Emperor Kanishka
(2) The Iron Pillar at Mehrauli 8

4. Make a list of the various books and


authors mentioned in the lesson. Down
1. (A word for) King in Indian languages.
5. Make a comparative chart of the 2. A Chinese traveller to India
Vardhan and Gupta dynasties based on 3. A famous Indo-Greek King had discussions
the followng points. with this Bauddha bhikkhu.
4. A poet at the court of Emperor
Gupta Vardhan Harshavardhan, who also wrote his
Points
Dynasty Dynasty biography.
6. Chandragupta II conquered this State
Founder neighbouring Gujarat.
Expansion of the Across
kingdom / empire 5. A Pali word that means question
Achievements 6. The most famous Indo-Greek King
7. Kanishka was the first Emperor to make
this from gold.
6. What would you do if you met foreign 8. An ancient Indian University where the
travellers like Yuan Chwang ? Chinese traveller Yuan Chwang stayed for
two years.
Activity :
Obtain more information about the rulers of the period following the Maurya period in India.
Enact the role of a ruler of your choice.
***

41
Telegram Channel -: Mpsc Super Batch 44 Compiled by Sanjay Pahade 44
Telegram Channel -: Mpsc Super Batch 45 Compiled by Sanjay Pahade 45
9. Ancient Kingdoms of the South
9.1 The Cher, Pandya and Chola dynasties 9.4 The Chalukya dynasty
9.2 The Satavahana dynasty 9.5 The Pallava dynasty
9.3 The Vakataka dynasty 9.6 The Rashtrakut dynasty

You can do this. 9.1 The Cher, Pandya and Chola


dynasties
Mark the important places in Of the many dynasties in South
South India on an outline map of India, three are mentioned in the literature
India. of those times. These are the Cher,
Pandya and Chola dynasties. These
42
42
Telegram Channel -: Mpsc Super Batch 46 Compiled by Sanjay Pahade 46
dynasties were in power around the
fourth century BCE or even before that. Do you know ?
They find mention in the Ramayana and
Mahabharata epics. They have also been Nane Pass (Naneghat) : The
mentioned in the ‘Sangham Literature’ in mountain pass, part of Junnar
Tamil and in the inscriptions of the Murbad, joining Pune and Thane
Maurya Emperor Ashoka. In the ‘Periplus districts is known as Naneghat. This
of the Erythrean Sea’, ‘Muziris’ is 5 km long road was built about two
mentioned as an important port on the thousand years ago during the rule
coast of Kerala. This port was in the of the Satavahanas. It used to be
Cher kingdom and was a major centre one of the main trade routes between
for the export of spices, pearls, precious the Konkan coast and the Deccan
stones etc. to Rome in Italy and to other Plateau (Desh). It was used for trade
countries of the west. The Pandya and transport. A carved pitcher
kingdom was a part of today’s Tamil (ranjan) can still be seen there.
Nadu. There was a great demand for Statues of the Satavahana kings and
their high quality pearls. The capital of some ancient inscriptions are to be
this kingdom was Madurai. The ancient found in the caves at Naneghat.
kingdom of the Cholas occupied the Descriptions of the donations given
region of Tiruchirapalli in Tamil Nadu. by the Satavahana kings and queens
are to be found in the cave
9.2 The Satvahana Dynasty
inscriptions.
As in the north, after the decline of
the Maurya Empire, the local kings in means ‘water’. The title means, ‘One
Maharashtra, Andhra Pradesh, Karnataka whose horses have drunk the water of
also became independent. They three seas’. The three seas are the
established small kingdoms of their own. Arabian Sea, the Bay of Bengal and the
One of these was the Satvahana dynasty. Indian Occean. During his reign, the
Pratishthan or the present day Paithan in empire of the Satavahanas extended from
Maharashtra was their capital. King the river Narmada in the north to the
Simuk was the founder of the Satavahana Tungabhadra river in the south.
dynasty. The inscriptions in the Naneghat The famous poetic work ‘Gatha-
caves near Junnar in Pune district bear Saptashati’ in Maharashtri, a Prakrit
the names of the important persons of language, was composed by the
this dynasty. Some of the Satavahana Satavahana king Hala. This work
kings were known to write their mother’s provides information about the life of the
name before their own. For example, people during the Satavahana period.
Gautamiputra Satakarni. Indian trade increased greatly during
Of the Satavahana kings, the Satavahana period. Paithan, Ter,
Gautamiputra Satakarni is particularly Bhokardan, Kolhapur were recognised as
well known. A stone inscription in a important centres of trade. During this
cave at Nashik describes his deeds of period, many artistic objects were
valour. He had defeated the Shaka king produced at these places. Indian goods
Nahpana. He is described as were exported as far away as Rome.
‘Tri-samudra-toya-peet-vahan’. ‘Toya’ Some of the Satavahana coins carry

43
Telegram Channel -: Mpsc Super Batch 47 Compiled by Sanjay Pahade 47
images of ships. The cave sculptures at have read that the Gupta Emperor
Ajanta, Nashik, Karla, Bhaje, Kanheri, Chandragupta II had married his daughter
Junnar in Maharashtra were carved Prabhavati to Rudrasen II, the Vakataka
king. Varahdev, a minister of the
Vakataka king Harishen, was a follower
of Buddhism. Cave number 16 at Ajanta
was dug at his instance. Some other

A Satvahana coin with a ship’s image

A cave at Ajanta

caves at Ajanta were dug and beautified


with paintings during Harishena’s reign.
The Vakataka king, Pravarsen II,
The Chaityagriha at Karla
composed ‘Setubandh’ in Maharashtri, a
during the Satavahana period. Prakrit language. Similarly, Kalidasa’s
9.3 The Vakataka Dynasty ‘Meghadoot’ also belongs to this period.
The power of the Satavahanas began
to weaken by the third century CE.
Among the dynasties that emerged after
that, the Vakataka was a powerful
dynasty. It was founded by the king
‘Vindhyashakti’. He was succeeded by
Pravarsen I. After his death, the kingdom
of the Vakatakas got divided. There were
two main branches. The capital of the
first was at Nandivardhan (near Nagpur)
and the capital of the other branch was
at Vatsagulm which is today’s Washim.
Pravarsen I, son of Vindhyashakti,
extended the Vakataka kingdom in the
north up to Malwa and Gujarat and in
the south up to Kolhapur. At that time,
Kolhapur was known as ‘Kuntal’. We Bodhisattva Padmapani - Ajanta
44
Telegram Channel -: Mpsc Super Batch 48 Compiled by Sanjay Pahade 48
9.4 The Chalukya Dynasty the Chalukya dynasty in the sixth century
The Chalukya dynasty in Karnataka CE. His capital was Badami which was
was a powerful one. The kingdoms of earlier called ‘Vatapi’. The Chalukya
Kadamba, Kalachuri had become King Pulakeshi II had successfully
powerful after the decline of the repulsed Emperor Harshavardhan’s
Vakatakas. But the Chalukya kings invasion. The famous temples at Badami,
established their dominance over all of Aihole and Pattadakal were built during
them. Pulakeshi I established the rule of the Chalukya period.

The temple at Pattadakal


9.5 The Pallava dynasty
The Pallavas were also a powerful dynasty in South India. Kanchipuram in

The ratha or chariot temples at Mahabalipuram

45
Telegram Channel -: Mpsc Super Batch 49 Compiled by Sanjay Pahade 49
Tamil Nadu was their capital. Mahendra and just treatment under the rule of the
Varman was an able Pallava ruler. He Pallava kings.
expanded the Pallava kingdom. He was 9.6 The Rashtrakuta Dynasty
also a playwright. His son At the height of their power, the
Narasimhavarman repulsed the attack on dominion of the Rashtrakuta dynasty
his kingdom by the Chalukya king, extended from the Vindhya mountains up
Pulakeshi II. The famous ‘ratha’ temples to Kanniyakumari in the south. Their
of Mahabalipuram were sculpted during power was first established in
his reign. These temples have been Maharashtra by king Dantidurg. Krishna
sculpted in single rock. Raja I had the famous Kailas temple
The Pallavas had a powerful and carved at Ellora.
well-equipped navy. During this period, Up to now, we have learnt about the
India came in close contact with the different dynasties that ruled ancient
countries of Southeast Asia. Internal and India. In the next lesson, we will review
foreign trade flourished. Yuan Chwang the social and cultural life in ancient
had visited Kanchi and recorded that India.
people of all religions received tolerant

The Kailas Temple at Ellora (Verul)

46
Telegram Channel -: Mpsc Super Batch 50 Compiled by Sanjay Pahade 50
Do you know ?

Periplus of the Erythrean Sea :


A periplus is a handbook. The periplus gives information about the trade
Erythrean sea is the Red Sea. For the that went on along the Indian coastline,
Greeks, the Indian Ocean and the Persian the Persian Gulf and Egypt. Ports like
Gulf were also a part of the Red Sea. Barigaza i.e. Badoch, Nala Sopara,
Periplus of the Erythrean Sea is a Kalyan, Muziris are mentioned in the
handbook of the Red Sea. It was written Periplus. Muziris was a port near Kochi in
some time in the first century CE. The Kerala. This ancient port does not exist
sailor who wrote it lived in Egypt. The any more.

Exercise

1. Can you tell ? 4. Observe any three picture in the lesson


(1) Whose name did the Satavahana and write in your own words what you
kings write before their own ? learn from them.
(2) The ancient name of Kolhapur. 5. Answer in one sentence.
2. Complete the table by reading the map (1) Name the ancient dynasties of
in the lesson. Southern India.
(2) After the decline of the Maurya
Pallav Kanchi
Empire which local kings became
Aihole, Badami, independent ?
Pattdakal 6. Anwer in brief.
(1) Write about the achievements of
Satavahana Mahendravarman.
(2) What is meant by ‘Tri-samudra-toya-
3. Tabulate the following names as peet-vahana’.
dynasties and their capitals : (3) What goods were exported from the
Satavahana, Pandya, Chalukya, Muziris port ?
Vakataka, Pallava, Madurai, Pratishthan,
Activity :
Kanchipuram, Vatapi
Make a collection of the pictures in
S.No. Dynasty Capital the chapter and display them in the
school exhibition.
1.
2. ***
3.
4.

47
Telegram Channel -: Mpsc Super Batch 51 Compiled by Sanjay Pahade 51
10. Ancient India : Cultural
10.1 Language and literature
Do you know ?
10.2 Life of the people
10.3 Science It is believed that most of the
10.4 Centres of Education languages in the region from North
India up to Maharashtra are derived
10.5 Art and architecture
from Prakrit and Sanskrit. The word
10.1 Language and literature ‘prakrit’ is derived from a word meaning
‘natural’. The prakrit languages were
There was an uninterrupted literary
languages in daily use of the people.
tradition in Ancient India. This literature
They can be divided into four groups,
was produced in the Sanskrit,
namely, the Paishachi, Shauraseni,
Ardhamagadhi, Pali and Tamil languages.
Magadhi and Maharashtri languages.
It included religious literature, treatises on
Marathi developed from Maharashtri.
grammar, epics, plays, stories, etc.
In this process of the development of
Sangham Literature : ‘Sangham’ modern languages like Marathi from the
means a gathering of learned men. The prakrit languages, their original forms
literature compiled in such gatherings is changed. They are called ‘Apabhramsha
known as ‘Sangham Literature’. It is the languages’. Modern languages have
most ancient literature in Tamil. developed from Apabhramsha languages.
‘Silappadhikaram’ and ‘Manimekhalai’
are two of its well-known epics. From Tipitika has three pitaka’s or parts.
Sangham literature, we learn about the The word pitaka means a basket or
political and social life in South India collections. Here, it means a section. The
during the period. Tipitaka is written in Pali. It consists of
Religious literature : The important three categories of texts. 1. Sutta Pitaka : It
texts include Aagamgranth, Tipitaka and includes the texts of Gautama Buddhas
Bhagwad Gita. teachings or sermons. They are called
The ‘Jain Agamgranthas’ are written suktas. 2. Vinay Pitaka : The word Vinay
in the Prakrit languages of Ardhamagadhi, here means ‘rules’. The Vinay Pitaka gives
Shaurseni and Maharashtri. The the rules of behaviour which bhikkhus and
Agamgranthas are a collection of the bhikkhunis in the Bauddha Sangha should
teachings of Vardhaman Mahavir. Literary follow in their day-to-day lives.
works like Mahapuranas, biographies, 3. Abhidhamma Pitaka : In this, Buddhist
stories, etc. are available in the doctrine has been explained. A text titled
Apabhramsha language. Siddhasen ‘Attakatha’ (Arthakatha) which explains
Diwakar wrote ‘Sammaisutta’ a work in the Tipitakas, is well-known. Learned
Prakrit, on jurisprudence. Vimalsuri has women composed gathas (narratives)
told the story of Rama in ‘Paumchariya’, about their own experiences. They have
a poetic work in Prakrit. Haribhadrasuri’s been compiled in the ‘Therigatha’. They
‘Samaraichchakaha’ and Udyotansuri’s are in the Pali language.
‘Kuvalayamalakaha’ are well known The ‘Bhagvad Gita’, which is a sacred
works.
48
48
Telegram Channel -: Mpsc Super Batch 52 Compiled by Sanjay Pahade 52
text of the Hindus, is a part of the
Mahabharata. The Bhagvad Gita tells us Do you know ?
that each one of us should do our duty
without expecting rewards. It also says Arthashastra : This is a work by
that the path of devotion to God is open Kautilya. It consists of detailed
to all. discussions of administrative matters
Adi Shankaracharya lived during the such as the duties of a king, criteria
eighth century CE. He emphasised for selecting a minister, systems of
knowledge and renunciation. He wrote defence, types of forts, the formation
commentaries explaining the ‘Upanishads’, of an army, plans for espionage,
‘Brahmasutras’ and the ‘Bhagvad Gita’. organization of the treasury and other
He established four muths in four directions offices, judicial system, investigation
of India at Badrinath, Dwarka, of theft, types of punishment, etc.
Jagannathpuri and Shringeri.
Arsha and classical epics :
‘Ramayana’ and ‘Mahabharata’ are the
two ‘Arsha’ epics of ancient India. ‘Arsha’
means composed by rishis or sages. The
‘Ramayana’ was composed by the rishi
‘Valmiki’. The main character of
Ramayana is Shriram. ‘Mahabharata’ has
been composed by the Sage Vyas. Its main
theme is the war between the Kauravas
and Pandavas. It also tells us about Lord
Krishna’s life. The Mahabharata provides
a comprehensive picture of the various
human sentiments and emotions and their
consequences.
Sometimes, there is such a period in
the history of language, literature and art,
that its glory remains undiminished even
later on. The art, literature, etc. produced
Adi Shankaracharya in such a period is said to be ‘classical’.
‘Raghuvansha’ and ‘Kumarsambhava’ by
Kautilya wrote the Arthashastra in
Kalidasa, ‘Kiratarjuniya’ by Bharavi and
which he discusses at length, what
‘Shishupalvadh’ by Magha are well-known
constitutes on excellent administrative
system. compositions of the ancient period written
Treatises on Grammar : in classical Sanskrit.
‘Ashtadhyayi’ written by the grammarian Theatre : India has an ancient tradition
Panini is considered to be the standard of telling a story through songs, music and
work on Sanskrit grammar. Patanjali wrote dance. These arts have been discussed in
‘Mahabhashya’. It explains the principles great detail in the ‘Natyashastra’ by
given in Panini’s ‘Ashtadhyayi’. Bharatmuni. When these arts are presented
49
Telegram Channel -: Mpsc Super Batch 53 Compiled by Sanjay Pahade 53
with supporting dialogues, they are known 10.3 Science
as theatre. Among the ancient Sanskrit Medicine : Indian medical science is
plays, ‘Swapnavasavadutta’ by Bhasa, known as ‘Ayurveda’. It has a very old
‘Abhijnanshakuntal’ by Kalidasa, etc. are tradition. It seeks to understand the
famous. symptoms of an illness, its diagnosis and
Narrative Literature ः In the ancient treatment. Also, much thought has been
times, storytelling was used to educate given to prevention of illness. Jeevaka was
people through entertainment. Gunadhya’s a well known vaidya, at the court of King
‘Bruhatkatha’ written in a language called Bimbisara. The ‘Charaka Samhita’ contains
‘Paishachi’ is well-known. ‘Panchatantra’ detailed information about clinical diagnosis
composed by Pandit Vishnusharma is an and pharmacy. It was written by Charak.
excellent example of narrative literature. The famous surgeon Sushruta has discussed
This text has been translated into many the diagnosis of different ailments and their
languages. Similarly, Baudhha Jataka remedies in his treatise the ‘Sushruta
tales are also very well-known. Samhita’. The importance of this text is
that it discusses the different causes leading
to injuries, fractures, their types and the
Try this.
various types of surgeries required for them.
Choose a tale from the Panchatantra. The text was translated into the Arabic
Write a skit based on it and enact it. language and was called ‘Kitaab-e-susud’.
Vagbhata also wrote many books on
10.2 Life of the people
medical science. The ‘Ashtang-sangraha’
The literature of ancient India sheds and ‘Ashtang-hridayasamhita’ are the most
light on the way of life of the common important of them. The Bauddha bhikkhu,
people of the time. Those were prosperous Siddha Nagarjuna in his book
times, due to the flourishing internal as ‘Rasaratnakara’ describes various
well as foreign trade. The society was chemicals and metals.
divided into different castes. There were Mathematics and Astronomy : The
organizations of traders as well as artisans. ancient Indians had studied Mathematics
These organizations were called shrenis. and Astronomy at great depth. Indians
Trade was carried out by sea as well as were the first to use the numerals 1 to 9
land routes. Indian goods like fine textiles, and zero. They invented the concept of
ivory, precious stones, spices, beautifully decimal system in which the value of a
made earthern pottery were in great digit changes according to its place ekam,
demand in foreign countries. The main daham, (units, tens) etc. The scientist
crops were rice, wheat, barley and lentil Aryabhata wrote the book ‘Aryabhatiya’,
(masoor). The diet of the people included which included many formulae for
food items made from these besides meat, mathematical operations. Aryabhata was
fish, milk, ghee and fruits. The people also an astronomer. He stated that the
mostly wore cotton garments, though silk earth revolves around the sun. Varahmihir
and woollen garments were also worn. wrote the famous text ‘Panchasidhantika’
Their clothes resembled today’s dhoti, in the sixth century AD. This text discusses
uparane, mundase, saree, etc. The concept principles of Indian astronomy along with
of stitching clothes was introduced in India the principles of astronomy from the Greek,
during the Kushana period. Roman and Egyptian civilizations. The
50
Telegram Channel -: Mpsc Super Batch 54 Compiled by Sanjay Pahade 54
texts of the mathematician Brahmagupta literature, Buddhist philosophy, economics,
who lived in the seventh century CE, were logic, etc.
translated into the Arabic language. Varanasi : The river Ganga has two
tributaries - Varana and Asi. The city
Do you know ? located between them came to be called
Varanasi. Since ancient times, it has had
Kanad : Kanad wrote the book centres which provided education in the
called ‘Vaisheshik Darshan’. It mainly areas of Vedic as well as Jain and Buddhist
discusses anu and paramanu. According philosophy.
to Kanad, the universe is full of Valabhi : Valabhi was an ancient city
innumerable objects. These objects are in Saurashtra, Gujarat. From the fifth to
nothing but the different forms of ‘anu’s. the eighth century CE, it was an important
These forms might change but the anu centre of Jain and Buddhist philosophy.
Yuan Chwang and Itsing, the Chinese
remains unchanged.
Bauddha bhikkhus had visited Valabhi.
10.4 Centres of education Nalanda University : The remains of
the ancient Nalanda University can be
There were many famous centres of
found near today’s Patna city in Bihar.
education in ancient India. Students from
Emperor Harshavardhan made generous
other countries also came there for their
donations to this university. According to
studies.
the descriptions of Yuan Chwang and
Takshashila University : Takshashila
Itsing, Nalanda University could
was an important city on the ancient Indian
accommodate thousands of students. The
trade route. Today, it is in Pakistan.
library stocked thousands of books.
Archaeological evidence found there
Students seeking admission to the university
suggests that the city was established in
had to appear for an examination at the
the sixth century BCE Jeevaka, a
entrance gate.
contemporary of Gautama Buddha and a
famous vaidya, had studied at Takshashila
University. By the fourth century BCE, the
fame of the university had spread far and
wide. Chandragupta Maurya the founder
of the Maurya Empire was educated at this
university. The grammarian Panini and the
vaidya Charaka were also students of
Takshashila University. The Greek
historians who accompanied Alexander
have also given a description of the
university. They have stated that such The seal of the Nalanda Mahavihara
a university did not exist anywhere in Vikramshila University : This
Greece. The famous Chinese Bauddha university was located near today’s
bhikkhu, Fa Hien who came to India Bhagalpur in Bihar. It was established by
around 400 CE also visited the Takshashila a king named Dharmapal in the eighth
University. The university provided century CE. It had six viharas, each having
education in various subjects such as Vedic its own separate entrance.
51
Telegram Channel -: Mpsc Super Batch 55 Compiled by Sanjay Pahade 55
Kanchi : During the reign of the
Pallava dynasty (sixth century CE), Kanchi
in Tamil Nadu, emerged as an important
centre of education. It was a centre for the
teaching and learning of Vedic, Jain and
Buddhist texts.
10.5 Art and architecture
Indian architecture reached its peak
during the Maurya and the Gupta periods.
The stone pillars erected by Emperor The stupa at Sanchi
Ashoka at various places are excellent
examples of Indian sculpture. The stupa at images of deities emerged during the
Sanchi and cave sculptures at Udayagiri, period of the Pallavas. The iron pillar at
Khandagiri, Karla, Nashik, Ajanta, Ellora, Mehrauli near Delhi is evidence of the
etc. show that the same tradition advanced advanced knowledge of metallurgy of the
even further. The art of making images ancient Indians.
was developed during the Gupta period. Thus, it is clear that ancient Indian
Temple architecture developed during the culture was very prosperous and advanced.
reigns of the Chalukya and Pallava In the next chapter, we will study India’s
dynasties in South India. The temples of contact with other civilisations and its
Mahabalipuram are a testimony to this far-reaching impact.
architecture. The art of making bronze

Bronze statue of Natraj The iron pillar at Mehrauli

52
Telegram Channel -: Mpsc Super Batch 56 Compiled by Sanjay Pahade 56
Exercise

1. Answer in one sentence. 5. Discuss.


(1) Make a list of ancient Indian Art and architecture of the Maurya and
universities. Gupta period.
(2) Make a list of the Indian goods that 6. What would you do ?
were in demand in foreign countries. (1) How would you obtain information
2. Name the following. about Ayurvedic medicine and use it
Epics and poetic compositions of ancient in your day to day life ?
India. (2) On a trip to a historical site, your
3. Fill in the blanks. friend is writing his name on the
(1) The epic Ramayana was composed by structure.
rishi ........... . 7. Observe the picture of the stupa at
(2) The science of Indian medicine is Sanchi and obtain more information
called .......... . about it.
(3) Thousands of students could live at
Activity :
the ............. university.
(1) Obtain information from the elders in
4. Answer in brief. your family, about some special
(1) Explain what is meant by ‘Tipitaka’. buildings in your neighbourhood.
(2) What is the the message of the
(2) Visit some historical buildings,
Bhagvad Gita ?
memorials in your neighbourhood
(3) What aspects does Ayurveda take into
and write a note about the history
account ?
you learn from the visits there.
(4) What is meant by Sangham Literature ?
***

The caves at Nashik

53
Telegram Channel -: Mpsc Super Batch 57 Compiled by Sanjay Pahade 57
11. Ancient India and the World
11.1 India and the countries of the west The facial features of the statues made in
11.2 India and other countries in Asia this style are similar to those of the Greek
people. The early coins that were minted
11.1 India and the countries of the west in India were also like Greek coins.
People of the Harappan civilisation
had developed trade relations with the
countries of the west. Ever since then,
India has had economic and cultural
exchanges with the outside world.
Buddhism had spread to Afghanistan and
to many countries of Central Asia. During
the period of the Iranian empire, India’s
contact with the western world increased.
The Greek historians of that period grew
more curious about India. Their writings
A sculpture in the Gandhara style on a stupa at
introduced India to the western world. Hadda in Afghanistan. (Greek costumes,
The routes which were later used by amphora and musical instruments)
Alexander, were opened for trade between
India and the countries of the west. Under Around the first or second century CE,
the influence of the Greek art of sculpture, trade between India and Rome flourished.
a new form of art emerged in India during The ports in South India also had a large
the Kushana period. It is called Gandhara share in this trade. Some articles made
school of art. In the Gandhar art school of bronze were found in the excavations
images of Gautam Buddha were mainly at Kolhapur. They have been made in
carved. As these images were mostly found Rome. The excavations at Arikamedu in
in the Gandhara region of Afghanistan, Tamil Nadu have also brought to light
the style is called the ‘Gandhara style’. many articles made in Rome. Both these
places were important centres of trade
between India and Rome. Many such trade
centres are mentioned in the literature of
those times.

A statue of
Gautama
Buddha in a
museum in
Paris - in the A gold coin of the Roman Emperor
Gandhara Style. Augustus found at Arikamedu

54
Telegram Channel -: Mpsc Super Batch 58 Compiled by Sanjay Pahade 58
Alexandria was an important port in
Egypt. The Arab merchants carried Indian
goods up to Alexandria. From there, they
were sent to the countries in Europe.
Besides Indian goods, the Arabs also
conveyed Indian philosophy and science
to Europe. The concept of ‘zero’ is one of
India’s major contributions to the world. It
was the Arabs who introduced this Indian
concept to Europe.

11.2 India and other countries in Asia

Many countries in Asia were greatly


influenced by the Indian culture of those
times. A wall painting in the Sigiriya caves

55
Telegram Channel -: Mpsc Super Batch 59 Compiled by Sanjay Pahade 59
Sri Lanka : King Ashoka had sent bhikkhus Dharmarakshaka and
his son Mahendra and daughter Kashyapmatanga to China. The bhikkhus
Sanghamitra to Sri Lanka to spread translated many Indian Buddhist texts into
Buddhism. Their names are mentioned in the Chinese language. This gave impetus
the Buddhist text ‘Mahavamsa’ written in to the spread of Buddhism in China. Later
Sri Lanka. Sanghamitra carried a branch on, Buddhism also reached Japan, Korea
of the Bodhi tree with her. The Sri Lankan and Vietnam.
tradition believes that the Bodhi tree at Countries in Southeast Asia : The
Anuradhpur in Sri Lanka has grown from ancient kingdom of ‘Funan’ in Cambodia
the same branch. was established in the first century CE.
There was a great demand in India for Evidence is available from Chinese
Sri Lankan pearls and other precious traditions that an Indian named Kaundinya
stones. In the fifth century CE King established it. People of Funan knew the
Kashyap had caves dug out at a place Sanskrit language. A stone inscription of
called ‘Sigiriya’. The wall paintings in that period is still in existence. It is in
these caves bear a resemblance to the Sanskrit. In many other countries of
paintings in the Ajanta Caves. The Sri Southeast Asia too, small kingdoms
Lankan Buddhist works ‘Dipavamsa’ and established by people of Indian origin had
‘Mahavamsa’ provide information about emerged. These kingdoms helped to spread
the close relations between India and Sri Indian culture in Southeast Asia.
Lanka. They are written in the Pali Indian culture had a deep impact on
language. the art and cultural life of Southeast Asia.
China and other countries : Right Dance-dramas based on stories from the
from the ancient times, India and China two Indian epics ‘Ramayana’ and
had established trade and cultural relations. ‘Mahabharata’ are popular in Indonesia
Emperor Harshavardhan had even sent an even today. This influence continued to
ambassador to the Chinese court. Chinese increase over the years. In the ancient
silk was known as ‘Chinanshuka’, and period, the Buddhist religion spread to
was in great demand in India. Merchants Myanmar, Thailand, Indonesia, etc. In the
in ancient India supplied Chinese silk to later years, temples of Lord Shiva and
western countries. The land route by which Vishnu were also built there.
the silk trade was carried on, was called This year, we have reviewed Indian
the ‘Silk Route’. Some ancient places in history from 3000 BCE to the eighth
India were connected to this route. century CE. Next year, we will study the
Nala-sopara near Mumbai in Maharashtra history of India from the ninth century CE
was one such place. The Chinese Buddhist to the eighteenth century CE. The history
bhikkhus Fa Hien and Yuan Chwang also of this period is known as ‘Medieval
came to India through this silk route. History’.
In the first century CE, the Ming
Emperor of China invited the Buddhist
56
Telegram Channel -: Mpsc Super Batch 60 Compiled by Sanjay Pahade 60
Exercise

1. Name the following. 4. Describe a picture.


(1) Places where articles made in Rome A picture of the Gandhara style sculpture
were found. on a stupa in Hadda, Afghanistan is
(2) A new style of art that emerged during given in this chapter. Observe it and write
the Kushana period in India. about it.
(3) The language of the books, 5. Find out more about -
Mahavansa and Deepvansa.
(1) The Gandhara style of art
(4) A country to which Buddhism had
spread in the ancient period. (2) The Silk Route

2. Think and expand/elaborate : 6. Mark the countries of Southeast Asia


mentioned in the chapter on an outline
(1) Indian culture had an impact on map.
Southeast Asia.
(2) The spread of Buddhism in China Activity :
became easier.
Obtain information about a form of art of
3. What would you do if you are your choice and present it in your class.
encouraged in your favourite hobby ?
***

57
Telegram Channel -: Mpsc Super Batch 61 Compiled by Sanjay Pahade 61
1. First Farmers

1.1 River Valley Civilisations of the neolithic age. The river valley
1.2 Beginning of Cultivation : civilisations grew in four regions of the
Agricultural Production world. Those regions were Mesopotamia,
1.3 The First Farmers in India Egypt, Indian Subcontinent and China. How
did this all happen? We shall briefly review
1.4 Agricultural Settlements :
it in this lesson.
Organisation and Administration
1.5 Trade and Transport
1.6 Beginning of Urbanisation
You would like to know : Scientists
in the Bar Elan University of Israel
1.1 River Valley Civilisations have found out that ancient people had
attempted cultivation, some 23000
Homo habilis made the first stone tools.
years ago. They have recovered its
These tools were useful only for simple jobs
archaeological evidence from ‘Ohalo’,
like scraping meat from the hides of dead
animals, splitting bones to extract marrow,
a palaeolithic camp site near the Sea
breaking nuts, etc. To strike the stone with of Galili. At Ohalo the evidence of
just enough impact and to remove flakes Barley and some other cereals, seeds
from a stone in order to shape a tool, was of fruits has been found along with
the mankind’s first big step in the field of traces of human occupation. There
technology. The later species of mankind were also a few seeds of various
after Homo habilis were more evolved. They weeds. These weeds are of evolved
advanced human technology much further. types and they are found only with
Through close observation of the seasonal cultivated crops. This fact confirms
cycles in the nature, the Mesolithic people that the cereals found at Ohalo were
could domesticate various plants and animals. the remains of cultivated crops and not
It resulted into the rise of the ‘Neolithic of the wild variety. Additional evidence
Age’. Cultivation and pastoralism (animal of grinding stones also came forth
husbandry) became a way of life for neolithic from this site. They were used to grind
people. It was the end of nomadic-semi the cereals before cooking them.
nomadic life for them. Settled villages came
into being. The archaeological evidence (1) The Valley of Euphrates and
shows that systematic agriculture began some Tigris - Mesopotamia : Ancient Mesopotamia
12000-11000 years ago. Beginning of included modern Iraq, Syria as well as the
agriculture and rise of settled villages are the western regions of Iran and south-eastern
two phenomena that are inevitably associated regions of Turkey.
with each other. The progress in the techniques Mesopotamia is the Greek name of the
of systematic cultivation resulted in the rise region between the two rivers, Tigris and
of the most ancient river valley civilisations Euphrates. Mesos means the ‘middle one’.

1
Telegram Channel -: Mpsc Super Batch 62 Compiled by Sanjay Pahade 62
Potamos means ‘river’. The region between
two rivers is ‘Mesopotamia’. The Mesolithic You would like to know : Egypt
people began to stay in this region for longer was originally known as ‘Kemet’.
times because of the availability of ample Egypt got this name because of the
water. Also both the rivers flooded annually, black soil that was deposited by the
making the soil on their banks very fertile. Nile floods. Later Egypt came to be
The camps of Mesolithic people developed known as ‘Hwat-ka-Pta’. It means
into the first settled neolithic villages. These the temple of God. The Greeks
first villages are dated to 10000 years B.C.E. changed it to ‘Egyptus’. In the course
The farmers in these villages grew wheat and of time it was changed to ‘Egypt’.
barley. The Arabic name of Egypt is ‘Misra’.
(2) The Valley of Nile - Egypt : The
country in the northern parts of the African ancient remains in Egypt and published it.
continent and in the valley of the Nile river These studies received a momentum because
is ‘Egypt’. of the decipherment of the Rosetta inscriptions.
Napoleon Bonaparte had taken along It became possible to read the Egyptian
many scholars from various fields during his hieroglyphs.*
invasion of Egypt in 1798. They studied the
*Hieroglyphs = pictorial script

2
Telegram Channel -: Mpsc Super Batch 63 Compiled by Sanjay Pahade 63
The first neolithic villages in Egypt came (4) The Valleys of Sindhu (Indus) and
into being at about 6000 B.C.E. The beginning Saraswati - Indian Subcontinent : The region
of agriculture in Egypt coincides with the of the valleys of Sindhu and Saraswati is
same time. The first crops grown in these now divided between India and Pakistan.
villages were wheat and barley. The archaeological excavations at
(3) The Valley of the Huang He - Harappa on the banks of Ravi in Punjab and
China : The valley of Huang He river is Mohenjo Daro on the banks of Sindhu in
considered to be the region of origin of the Sindh, indicate the existance of a fully
Chinese culture. The Chinese culture evolved developed civilisation in the Indian
here. subcontinent which was dated to circa 3000
Agriculture in the neolithic villages of B.C.E.
China began at about 7000 B.C.E. Wheat, A number of archaeological sites of
foxtail millet and rice were grown by the neolithic villages have been now discovered.
farmers in these villages. It proves that there were well settled villages
in the Indian subcontinent as early as 8000
B.C.E. There is a general consensus among
You would like to know : ‘Yellow
the scholars that the Harappan cities evolved
River’ is the English translation of
from these early villages.
the Chinese name ‘Huang He’. The
yellow silt brought by her has earned Barley was the main crop grown by the
farmers in these villages. However, wheat
her this name. ‘River’ and ‘Mother’
was also grown on a smaller scale. The
are her other names. These names
farmers in these villages were the first
indicate her extraordinary place in
farmers of the Indian subcontinent. They
the Chinese culture. The name ‘River’
kept cattle and goat-sheep. They lived in
indicates that she is considered to be mud houses.
the only river of importance. The fact
The archaeological site at Mehrgarh in
that she is considered to be the
Baluchistan has yielded evidence of the
mother of the Chinese culture, is
continuous human occupation from the
obvious in the name, ‘Mother’. neolithic times to the rise of Harappan
Huang He and Yangtse in China civilisation and the material culture of the
are the rivers, which receive waters respective periods.
from the Himalayan glaciers as do 1.2 Beginning of Cultivation :
the rivers Ganga, Sindhu and Agricultural Production
Brahmaputra. Huang He flows so The transition from hunter-gatherer’s life
rapidly that it continuously keeps to the beginning of cultivation was a very
changing its course. The torrential slow process drawn over thousands of years.
floods caused because of it used to Beginning of cultivation is also the beginning
be very disastrous causing great loss of the neolithic age. It spans through 10000
of life and assets. Hence, she had – 8700 B.C.E. It may vary to some extent
earned one more name, ‘Sorrow’ or from place to place. This period marked the
the ‘River of Tears’. Now the terrible beginning of domestication of animals along
river has been brought under control with the beginning of cultivation. Barley was
by creating a number of dams and the main crop in this period. Other crops
bunds on her. were wheat and flaxseed.

3
Telegram Channel -: Mpsc Super Batch 64 Compiled by Sanjay Pahade 64
The blades used for Mesolithic tools were removed from siliceous stones by the
technique known as ‘Fluted Core technique’. These blades are as tiny as our fingernails.
So they are called ‘microliths’. The Mesolithic people used wooden arrows fixed with
microlithic arrowheads.

Mesolithic Blades
(Microliths) Microliths used
as arrowheads

Fluted Core

The serrated knife made by


hafting microliths
The serrated knife made by hafting microliths

It is apparent that the common animals and vegetation for food increased.
prerequisites were present in Mesopotamia, This period had also witnessed extinction of
Egypt, China and Indian Subcontinent to species of gigantic animals like mammoth.
initiate the onset of the Neolithic age and However, varied species of fish and small
agriculture. The last Glacial period in the animals like goat, sheep, deer, etc. were
history of our planet came to an end at amply available for food. The heavy tools of
about 12000-11000 years ago. It was the Palaeolithic times were of no use either for
beginning of a new epoch, which is known fishing or for hunting small animals.
as ‘Holocene’. The technique of removing long stone
With the beginning of the Holocene, blades had been developed in the Upper
glaciers began to melt and the water volume Palaeolithic age. In the Mesolithic age the
in the water bodies like rivers and ocean same technique was used and blades as small
increased. As a result, the availability of as fingernails were removed to make tools.
These blades known as microliths were either
hafted* on the tips of wooden/bone shafts or
in the grooves made on those shafts. Thus,
modified composite tools like fishing harpoons,
spears and arrows could be made. Ample
cereals and fruits, grown naturally, were
available. Tools like sickle and scythe were
made by hafting microliths for harvesting
the cereals and fruits.

*To haft - fix with the help of rope and resin/gum


Mammoth

4
Telegram Channel -: Mpsc Super Batch 65 Compiled by Sanjay Pahade 65
Thus, the technology evolved; the transition was drawn over a prolonged period.
techniques of hunting and obtaining food Nevertheless, considering radical changes in
improved; food became available in plenty. the way of life during the Neolithic Age, the
This resulted in making the Mesolithic people Australian archaeologist Gordon Childe
stay at one place for a longer time of the coined the term ‘Neolithic Revolution’.
year. Harvesting of naturally grown cereals 1.3 First Farmers of India
led them into systematic cultivation. The Neo means ‘new’. Lithic means ‘of
technique of agricultural and pastoral stone’. Neolithic age got its name because of
operations (animal husbandry) continued to the stone tools of completely new types. It
evolve. The nature of these operations is included short and long heads of axe, made
such that the farmer is compelled to settle of polished stone and polished stone rings.
down permanently near the cultivated land. The axe heads were hafted on wooden shafts
He doesn’t need to go out in search of food. and used for cutting trees, scraping wood to
This is how the villages were settled on a give it a desired shape, etc. It was essential
permanent basis. This prepared the ground to clear jungles by felling trees in order to
for the rise of the Neolithic Age. This obtain land for cultivation and settlement.
The new tools came handy for this work.
You would like to know : The We have already seen that the site of
Palestinian city of Jericho on the banks Mehrgarh came into existence around 7000
of the river Jordan, is a historical city B.C.E. This site is of great importance among
with hoary past. It first settled as a the neolithic villages in the Indian subcontinent
village in 9000 B.C.E. It was one of established by the first farmers. The Mehrgarh
the first neolithic permanent settlements. farmers cultivated barley and wheat. There
It started getting organised into a well-
is another site of a neolithic village that is
knit society at about 8000 B.C.E. The
contemporary to Mehrgarh. The site was
village at this time had a protective
found at ‘Lahuradeva’ in Sant Kabirnagar
wall around it, complete with a watch
tower. This is undoubtedly an evidence district of Uttar Pradesh.
of an organised society. In Maharashtra, the mesolithic man
The beginning of cultivation at existed during 10000-4000 B.C.E. He stayed
Jericho and in the surrounding region in natural caves and under rock-shelters. He
began earlier than the establishment of moved along the river banks. He made
the village by a few centuries. Its microliths from silicious* stones. However,
evidence has been discovered at Gilgal, no evidence of purely neolithic sites have
a site near Jericho. A fire stricken been found in Maharashtra. What we have
house at Gilgal was excavated, which instead, are the Chalcolithic villages. The
yielded burnt remains of figs. After Chalcolithic people were the first farmers of
carrying out laboratory analysis of Maharashtra. Inamgaon, in the Shirur taluka
these fig remains, scientists have of Pune district, is an important site of the
concluded that the neolithic people at
village of Chalcolithic farmers in Maharashtra.
Gilgal had systematically planted fig
cuttings. This stands to be the first
attempt of planned cultivation. *Siliceous stones - stones like quartz, chalcedony,
agate, etc.

5
Telegram Channel -: Mpsc Super Batch 66 Compiled by Sanjay Pahade 66
1.4 Agricultural Settlements : making of pottery seems to have begun in
Organisation and Administration the 2nd phase of the neolithic period. Pottery
Researchers have attempted to estimate making, along with the polished stone tools,
the population of neolithic villages with the is supposed to be an essential characteristic
help of various mathematical and statistical of the neolithic cultures. The earthen pots
models. With the help of those estimates we were used for the purpose of cooking, serving
may gather some idea with regards to food and for storage. In the beginning these
population compositions of those villages. pots were handmade. At a later stage, wheel-
When Mesolithic people were witnessing the made pottery was introduced. The neolithic
transition from nomadic-semi nomadic life to pottery in the beginning was in monochrome
settled life, there could have been 25-40 (single colour), with a burnished* surface.
people in any given group. Life dependent Some pots had carved designs. Also the
on regular cultivation would need that number techniques of stamping and applique work**.
to increase at least to 50. A village with at Gradually neolithic people began to decorate
least a population of 100 could meet the their pots with painted designs. Thus, in the
needs of individual members who would be latter half of the neolithic age, pottery making
completely dependent on agriculture and had become an art that required special
animal husbandry. skills.
Neolithic villages were small in size. *Burnished - polished by rubbing with a wooden
Permanently occupied dwellings (mostly or stone tool
round huts) were characteristic of these **applique work - pasting wet clay ribbons and
villages. Other than these dwellings, some making patterns on them
places in the central part of the village were
perhaps used as common storage places of To master the art of pottery-making one
food-grains and other things. This internal needs to possess knowledge of following
settlement pattern of the village indicates that things:
the transactions related to food production (1) To know the source of clay of
were centrally controlled. Accordingly a excellent quality. (2) To manage to obtain
hierarchical chain of power positions must clay. (3) To knead and prepare the clay for
have been established. At the same time pottery production. (4) To be able to turn the
other social norms were also established. For pot to the desired shape. (5) To have artistic
example: boundaries of individual households- skill for decorating the pottery. (6) To know
families, ownership of land, food production the technique of baking pottery at proper
to meet the needs of individual household- temperatures (approximately 850º-900º
family and rules of kinship. The tradition of Celsius).
imparting training of unique family skills of The class of skilled potters, equipped
production to the younger members is with essential knowledge of all stages of
supposed to have begun as early as the pottery-making, was already established in
neolithic times. In other words, it may be the 2nd phase of the neolithic period. Ancient
said that the hierarchical social and family pottery can throw light on various aspects of
structure and the concept of inheritance have ancient cultures. We get to know about the
its roots in the neolithic times. cultural history, the material richness of the
The culture known as ‘Jomon Culture’ place where it was found. With the help of
of Japan was a mesolithic culture, which has pottery we can also know about the contacts
yielded evidence of pottery making. Except of the residents of the place with other
this example, everywhere else in the world cultures.

6
Telegram Channel -: Mpsc Super Batch 67 Compiled by Sanjay Pahade 67
The neolithic period also saw rise of Excavations at neolithic sites yield beads,
other skilled artisan classes based on waste material cast away during bead making
specialised crafts. It is apparent that the skill process, faulty and incomplete beads and the
of making beads of various types was tools used by the bead maker. The following
acquired by neolithic artisans from the are the stages of bead making:
beginning. They made beads of various types (1) To know the sources of siliceous
of siliceous stones and chank shells. stones and chank shells. (2) To obtain the
raw material. (3) To transport the raw
For additional information : material to the manufacturing site. (4) To
Some important Neolithic sites in India manufacture finished beads from the stones
of irregular shape and the core of chank
1. The northwest region of the
shells. The artisan has to be conversant with
Indian continent : Phase I (7000-
every stage of the bead manufacturing.
6000 B.C.E.) No evidence of pottery
(earthen pots). Phase II (6000-4000 In brief, agricultural production, as well
B.C.E.) Beginning of pottery as production of agricultural tools, pottery,
making, e.g. Mehrgarh. beads etc. is a chain of various skill based
2. Jammu and Kashmir - around operations, which are completed step by step.
2500 B.C.E. neolithic settlements It is a complex system, which calls for
established at the sites of Burzhom systematic division of labour and training of
and Gufkral. people for various skills. This process leads
to craft specialisation and creates various
3. Uttar Pradesh - around 6000
classes of skilled artisans.
B.C.E. neolithic settlements
established at the sites of Chopani 1.5 Trade and Transport
Mando, Koldihwa and Mahagara. The nomadic-semi nomadic groups of
4. Bihar - around 2000 B.C.E. Mesolithic people had a system of barter
neolithic settlements established at among themselves. The wheel was not yet
sites like Chirand, Senuwar, etc. invented. It means that the means of
5. Northeast India - The evidence of mechanical transport were not available to
neolithic settlement was first them. In the latter half of the neolithic age
discovered at the site of Daojali people had started using beasts of burden for
Hading in Assam. The village was transporting goods.
established around 2700 B.C.E. The Tools like axe, scrapers and chisels
neolithic tools found here show enabled neolithic people to cut trees and to
more affinity to neolithic tools work on wood. Most probably round shaped
found in China. pieces of wooden logs were used as wheels.
6. South India - The neolithic Such round pieces can easily gain momentum.
settlements in Karnatak, Andhra The neolithic people invented wheel and
Pradesh and Tamilnadu first came the technology they had developed so far,
into being around 4th - 3rd century underwent a revolution.
B.C.E. Karnatak sites - Sangankallu, The neolithic people began to produce
Maski, Brahmagiri, Tekkalkota, wheel-made pottery. The wheel technology
Piklihal, Hallur, etc. Andhra Pradesh made it possible to produce pottery on mass
site - Nagarjunikonda. Tamilnadu scale. The invention of wheel also made it
- Payyampalli possible to obtain raw material from distant
places in much lesser time. It became easier

7
Telegram Channel -: Mpsc Super Batch 68 Compiled by Sanjay Pahade 68
Telegram Channel -: Mpsc Super Batch 69 Compiled by Sanjay Pahade 69
to send the finished goods to distant markets social organisation and administration.
where it was in more demand. It helped to Eventually the villages expanded. Along with
develop trade and transport on a greater it, awareness of collective land holdings and
scale. the village boundaries became prominent.
Thus, it was felt that the village community
had a natural right over the immediate
By the Way : territory around the village periphery. This
The neolithic wheel kind of awareness of natural right over a
might have been particular region is known as ‘territoriality’.
similar to the wheel Rules and social norms were created in order
seen here. We do to manage the collective resources, water
not know, how old sources, crafts dependent on it, trade and
is this wheel. It community life, etc. Consequently rituals and
appears here as an their details gained importance. The need of
illustration, only to managing and keeping records of trade,
give some idea of a rituals, writing systems and administrative
wooden wheel, made by using a round machineries developed. It gave rise to
shaped piece cut from a wooden log. administrative centres. Such administrative
centres attract large number of population
1.6 Beginning of Urbanisation which includes officials, people from various
occupations. As a result, periphery of the
After settling down at one place for a
original settlement kept expanding and cities
long time, a feeling of ownership arose
came into being. Thus began the process of
among the neolithic people. This feeling was
urbanisation in the neolithic age.
focused on their individual dwellings and the
piece of land cultivated by them. This has In the next lesson we shall learn about
been already mentioned in the context of the first cities of India.

Exercise

Q.1 (A) Choose the correct alternative and (c) Herodotus


write the complete sentences. (d) Collingwood
(1) ‘Yellow River’ is the English
(3) The neolithic people at Gilgal had
translation of the Chinese name
systematically planted trees.
.
(a) Guava (b) Sapodilla
(a) Kemet (b) Mother
(c) Fig (d) Indian blackberry
(c) Sorrow (d) Huang He
(2) The Australian archaeologist (4) in the Shirur taluka of
coined the term ‘Neolithic Pune district, is an important site of
Revolution’. the village of Chalocolithic farmers
(a) Gorden Willey in Maharashtra.
(b) Gorden Childe (a) Saradwadi (b) Ranjangaon
(c) Pabal (d) Inamgaon

8
Telegram Channel -: Mpsc Super Batch 70 Compiled by Sanjay Pahade 70
(B) Find the incorrect pair from set B Q.4 State your opinion.
and write the correct ones. (1) There were radical changes in
Set ‘A’ Set ‘B’ the way of human life during the
(1) The valley of Mesopotamia neolithic age.
Tigris and Euphrates (2) The neolithic people invented
(2) The valley of Nile Iran wheel and the technology they
(3) The valley of China had developed so far, underwent a
Huang He revolution.
(4) The valley of Indian Q.5 Observe the map on page 2 of your
Sindhu (Indus) subcontinent textbook and answer the following
and Saraswati questions based on it.
Q.2 Complete the concept map. (1) What is the name of the sea at the
Stage 2 north of the African continent?
(2) In which continent did the Harappan
civilisation originate ?
(3) What is the name of the ocean at
Stage 4
the south of India?
Stage 1
Q.6 Write short notes.
Bead-making (1) Neolithic ‘Jericho’
Method and Stages
of production (2) ‘Holocene’ epoch
Stage 3
Q.7 Answer the following questions in
Manufacturing finished detail.
beads from the stones of
irregular shape and the
(1) Write in detail about the ‘first
core of chank shells. farmers’ and beginning of agriculture
in India.
(2) What knowledge was essential for
Q.3 Explain the following statements with
the neolithic people to make pottery?
reasons.
(3) Write about the trade and transport
(1) The camps of Mesolithic people
in the neolithic age.
developed into the first settled
(4) Write about the beginning of
villages of neolithic people in
urbanisation in the neolithic age.
Mesopotemia.
(2) ‘Huang He’ river is considered to be Activity
the mother of the Chinese culture. Write in detail about any of the ancient
(3) In the latter half of the neolithic civilisations.
age, pottery-making had become an

art that required special skills.

9
Telegram Channel -: Mpsc Super Batch 71 Compiled by Sanjay Pahade 71
Neolithic Age
8000-3000 B.C.E.

First Farmers You womenfolk began


to grow food in the yards.
Now there is no need to
go hunting everyday.

Yes ! There is
always some
food available.

The villagers have


chosen me as their Is there any
‘Chief ’. It’s a problem ?
great responsibility.

Yes ! Now
There should Our ancestors I need to
be equal had to wander in complete a few
justice to all. the wilderness for chores.
food and water.

Our coming
generations will We should take
be here in this village. good care of
We are well-rooted in our land, water
this soil, now. and cattle.

I took a round There used to be a small village here.


in the village market. Now look ! We are a big city ! Let’s pray !
Saw some people May our city be safe forever !
from neighbouring May there never be any hard times !
villages.

Good !
We will have
better trade
with them.

10
Telegram Channel -: Mpsc Super Batch 72 Compiled by Sanjay Pahade 72
2. First Cities of India

2.1 Characteristics of Harappan Harayana. The total area covered by it is


Civilisation about 1500000 Sq. Kms. The Harappan
2.2 Interrelationship between Cities civilisation belonged to ‘Bronze Age’. So far,
and Villages more than 2000 sites of the Harappan
2.3 Production, Trade, Organisation civilisation have been recorded. The remains
and Administration found at cities like Harappa, Mohenjodaro,
2.4 Decline of the Cities Kalibangan, Lothal, Dholavira, Rakhigarhi,
etc. are the evidence of the past glory of this
2.1 Characteristics of Harappan civilisation.
Civilisation The history of these cities had three
It has been mentioned earlier that the stages: (1) Early Harappan period
roots of the Harappan civilisation are to be (2) Mature Harappan (urban) period (3) Late
found in the pre-Harappan culture of Neolithic Harappan period.
times at Mehrgarh in Baluchistan. This The question regarding the authors of the
culture is known as ‘Togao culture’. It shows Harappan civilisation still remains mysterious.
traces of the beginning of the Harappan Some of the main characteristics of the
culture. Jean Francois Jarrige and Richard Harappan cities are noted below.
Meadow conducted the excavations at (1) Systematic Town Planning :
Mehrgarh. Another neolithic culture of the Houses of baked bricks complete with
pre-Harappan period is known as ‘Ravi or bathrooms, toilets, wells, etc., existance of
Hakra culture’. Its remains have been found granaries, impressive public monuments,
during the excavations at sites like Harappa English bond masonry (method of using two
(Punjab, Pakistan), Kunal, Bhirrana and headers and two stretchers alternatively while
Farmana (Harayana). building a wall – this method is specially
A few years ago it was strongly believed useful in earthquake prone regions), Excellent
that the Vedic Aryans arrived in India at drainage system, public baths, two or more
about 1500 B.C.E. However, no information divisions of the city with each division
was available regarding the period before enclosed by independent fortification walls,
that. This situation continued till the discovery Streets crossing each other in right angles
of Harappa in 1921 and Mohenjodaro in and the resulted blocks used for building
1922. The discovery of the Harappan houses (grid pattern).
Civillisation at these two places stretched the (2) Central Administration :
antiquity of Indian history to 3500-3000 Administrative control over the distribution of
B.C.E. At that time a well-developed and water and other resources, Standardisation :
rich civilisation had flourished in the Indian For example, the size of bricks in the ratio
subcontinent. It extended from Afghanistan to of 1:2:4, weights increasing in the proportion
Maharashtra and from Makaran coast to of 8, set style of shapes and painted designs

11
Telegram Channel -: Mpsc Super Batch 73 Compiled by Sanjay Pahade 73
of pottery, majestic and non-residential is important. At that time, he was the
buildings for public administrative offices. Director General of the Archaeological
(3) Social organisation : Social hierarchy Survey of India. During the excavation he
based on position of power, classes of skilled could trace the fortification wall around the
artisans and individuals based on craft citadel of Harappa.
specialisation, belief systems and artefacts/ The first settlement of the Early Harappan
architecture indicative of them, evidence of period was established around 3300 B.C.E. It
burials indicative of rituals after death. evolved to the Mature Harappan (urban)
(4) Economy : Mass production of goods phase around 2600 B.C.E. It reached its
for trading purpose : For example, well- peak during 2450-1900 B.C.E. Around 1900
shaped, beautiful earthen pots, statues, metal B.C.E. some internal changes begin to appear
objects of gold, silver, copper and bronze, in the Harappan cities. This is the time of
various types of beautiful beads, etc. the onset of the Late Harappan phase. This
Concentration of factories and residences of phase indicates the decline of Harappan
artisans in a particular area of the city for civilisation.
the sake of convenience of production. The evidence of the first excavations at
Flourishing internal and distant trade, Harappa had indicated that the city of
administrative control over trade transactions. Harappa was divided into two parts - the
(5) Use of Developed Writing System  : ‘Citadel’ and the ‘Lower Town’. However,
The script on Harappan seals. recent excavations show that it had four
divisions. Apart from the citadel and the
Some Cities of the Harappan
lower town there were factories and the
Civilisation
quarters of artisans in the Southeast part of
Harappa : Harappa is situated on the
the city. The area toward the north of the
banks of the river Ravi (District Sahiwal in
citadel had a granary and the quarters of the
Punjab, Pakistan). The ancient site of Harappa
workers working there. It is obvious that the
had spread on 150 Hectares. The first
officials staying in the citadel supervised
excavation at Harappa began in 1921. More
over the granary and the workers working
excavations were conducted at the site, from
there. A few working platforms were found
time to time. Among them the excavations
near the granary.
conducted in 1946 by Sir Mortimer Wheeler

For additional Information : 1. The bed remains dry through the year except
archaeological cultures are named after the during the monsoon. It is known as
site where the characteristic pottery and Ghaggar in India and as Hakra in Pakistan.
artifacts of the culture were discovered for A number of Harappan settlements were
the first time. discovered along its dry beds during
2. The river known as ‘Ghaggar- exploratory surveys.
Hakra’ flows only during the monsoon. It 3. Many scholars are of the opinion
originates in the Siwalik hills in the that the dry beds of Ghaggar-Hakra are
Himachal Pradesh and flows to Rajasthan the traces of the vanished river ‘Saraswati’.
through Punjab and Haryana. From there Satellite images of dry beds are being
it enters the desert of Cholistan in Pakistan. studied in depth by scholars to gather
Ultimately it enters the Rann of Kutch. Its more information about these dry beds.

12
Telegram Channel -: Mpsc Super Batch 74 Compiled by Sanjay Pahade 74
You would like to know : Before the of Harappa by the reports written by Masson
discovery of the Harappan civilisation, and Burnes. He conducted excavations at
Alexander’s invasion in 326 B.C.E. was the Harappa during 1872-73 C.E. After that for
only evidence available in the context of next 48 years no excavations were conducted
ancient History of India and Pakistan. How at the site. However, during this long interval
this situation entirely changed because of the the curiosity for Harappan site among the
discovery of the Harappan cities, is a fascinating European scholars had not vanished. A
story. It began in the year of 1829. Charles general impression prevailed among them
Masson, a British traveller was the first one to that the Harappan seals had some connection
visit and write about the archaeological remains with the Mesopotamian civilisation.
at Harappa. He thought that these remains
could be that of the capital of King Porus who The drawing of a
Harappan seal published
fought with Alexander. Lieutenant Alexander by Sir Alexander
Burnes, a British officer was the next one to Cunningham. The
visit this site. Harappan seal here
appears to be rectangular.
Sir Alexander Cunningham was the first However, Harappan
Director General of the Archaeological Survey seals, in reality are
of India. His attention was attracted to the site square in shape.

13
Telegram Channel -: Mpsc Super Batch 75 Compiled by Sanjay Pahade 75
Mohenjodaro : The excavations at
Mohenjodaro by Rakhaldas Banerjee began
in 1921-22. He concluded, because of the
similarity between the seals and other
artefacts, that
the ancient
civilisations of
Mesopotamia
and Harappa A multi-roomed house at Mohenjodaro
had cultural Chandigarh. Considering the economic and
Harappan Pots
ties with each human resources required for the creation of
other. In 1923- a pre-planned city like Mohenjodaro, it is
24, to gather more information about obvious that it was created to serve a
Mohenjodaro Madho Sarup Vats, Kashinath distinctive purpose.
Narayan Dikshit, Earnest Mackay and others
Kalibangan : The site of Kalibangan is
conducted further excavations under Sir John
205 kilometres away from Bikaner. It is
Marshall’s direction. He was the Director
located in the Hanumangarh district of
General of the Archaeological Survey of
Rajasthan. It was one of the important urban
India at that time. During these excavations
centres of the Harappan civilisation, situated
various artefacts, houses and public
on the banks of the river Ghaggar. L. P.
monuments were unearthed.
Tessitore, an Italian linguist had visited
Mohenjodaro was built on the banks of Kalibangan during his study tour of the
the river Sindhu (Indus) in Pakistan (District region. He had reported that Kalibangan is
Larkana, Sindh). It was not possible to reach an ancient site and it could be prehistoric. It
the bottom layer during the excavations was confirmed to be a Harappan city only
because of the high levels of groundwater. after the excavations began there in 1960. It
However, it is likely that a neolithic settlement was conducted under the direction of Brijabasi
contemporary to the one at Mehrgarh existed Lal, the then Director General of the
there. Archaeological Survey of India and Balkrishna
Areawise, Mohenjodaro is the biggest Thapar.
city, among those discovered in Pakistan so At Kalibangan two settlements, one of
far. It was thought that the city was divided the Mature Harappan period and the other of
into two fortified sections, namely the the Early Harappan period, were found.
‘Citadel’ and the ‘Lower Town’. However, Kalibangan was a small city compared to
there was an additional section of the market. Harappa and Mohenjodaro. The division of
In this section were found various workshops, the city into the ‘Citadel’ and the ‘Lower
kilns for baking pottery and beads. Town’ was found here also. The site of
The remains at Mohenjodaro were Kalibangan is important for two things. The
considerably intact. So the glory of the city first is the ploughed field of the Early
was revealed in way of the houses, majestic Harappan period. This field is dated to about
buildings, wide streets, etc. Thus the evidence 2800 B.C.E. The furrow marks in this ancient
of the impressive town planning, and public field are similar to the furrows of modern
administration, characteristic of the Harappan fields in the surrounding area. The other
civilisation came into light. The town planning thing is the series of six to seven fire altars
of the Harappan cities can be easily compared in the citadel. Evidence of fire alters inside
with the town planning of a modern city like the houses of common people has also been

14
Telegram Channel -: Mpsc Super Batch 76 Compiled by Sanjay Pahade 76
unearthed. Apart from this there was a house
with a series of fire altars outside the
fortification wall. Some of them contained
animal bones. According to some
archaeologists, this might be indicative of
fire worship.
Lothal : The centre of the Harappan
civilisation at Lothal was situated near the
Gulf of Kutch, in Gujarat, 80 kilometres
away from Ahmedabad. It is known for the
remains of Harappan dock. Lothal was
excavated from 1955 to 1960 under the The Water Tank at Dholavira
supervision of S.R. Rao. sites, extentwise Dholavira is the fifth largest
The ‘Citadel’ and the ‘Lower Town’ at city. Its town planning is somewhat different
Lothal do not have separate fortification compared to other Harappan cities.
walls. Rather they are surrounded by a single An Early Harappan settlement was
fortification wall. discovered at Dholavira. There was a
Lothal was established on the banks of protective wall built around it by using mud
the river Bhogao. The presence of platforms bricks (unbaked bricks) and dressed stones.
built for stalls, a warehouse, and the dock The layers above this settlement had the
confirm that Lothal was an important Harppan city of Mature Harappan phase. It
Harappan port and trading centre. The was surrounded by an outer fortification wall.
architecture of the dock at Lothal is an The settlement within the outer fortification
example of the engineering excellence of was divided into four sections.
ancient people. At present Lothal is away (1) Citadel (2) The adjacent section
from the sea. However, in the ancient times reserved for high officials (3) Lower Town
Lothal was in the delta region of the gulf of – These three sections had walls, which
Kutch. It is clear that the Harappan settlers separated them from each other. (4) The
of Lothal had a good knowledge of the high fourth section inside the outer fortification
and low tides and accordingly the rising and did not have any additional separating walls.
receding pattern of Bhogao waters. The dock This fourth section was perhaps meant for
was built to facilitate the entry of boats the workers. A dam of baked bricks and
during the high tide and their exit during low stone was built to obstruct the water of two
tides. Once the boats entered the dock, the streams, which flowed near the closed city
water was drained out through a sluice gate and the water was diverted to canals and
into a canal. The sluice gate of baked bricks lakes. This was the most ancient system of
was built to reduce the water level in the planned water management.
dock as and when required. Then the repair
Rakhigarhi : In the Hissar district of
work of the boats was completed and with
Haryana is situated the site of Rakhigarhi, a
the rise of water the boats could sail back.
Harappan city. It is on the distance of 150
Dholavira : Dholavira was discovered kilometres from Delhi. It is located on the
by J.P. Joshi, the Director General of
banks of Chautang (ancient Drishdvati river).
Archaeological Survey of India. The site is
Rakhigarhi is the largest Harappan site,
in ‘Khadirbet’ in Gujarat (Dist. Kutch).
among all the sites from India and Pakistan.
Excavations at the site were started by R.S.
Its total area was more than 350 Hectares.
Bisht in 1990. Among the excavated Harappan
The excavations at Rakhigarhi started in

15
Telegram Channel -: Mpsc Super Batch 77 Compiled by Sanjay Pahade 77
1963. It continued in 1997-2000. Later, administration. The raw material needed for
Dr.Vasant Shinde of the Deccan College, Harappan industrial production included clay,
Pune also conducted excavations at the site. various siliceous and semi-precious stones,
At Rakhigarhi, all characteristics of the metals, etc. The interrelationship between
Harappan cities were present. It was the Harappan cities and villages was dependent
main centre of the Harappan civilisation in on the mechanism of making available,
its eastern region. At this site too, evidence things like food grains and the raw materials.
of fire altars like Kalibangan have been Not that all the pre-Harappan settlements
found. were transformed into cities and towns.
Indeed, cemeteries were found at various Some of them did not change notably. There
other Harappan sites, but the human skeletal was a network of small towns, small and big
remains from there were not studied to villages and camp sites of semi-nomadic
analyse the hereditary characteristics of the people, which functioned to cater to the needs
population. However, the human bones from of major urban centres of the Harappan
Rakhigarhi were subjected to genetic analysis. civilisation. It included villages in remote
Scientists believe that in due course it would regions as well. The best example of it is
provide satisfactory answers to the question the Harappan settlement of ‘Shortugai’ in the
of the origin of Harappan people. Badakshan province of Afghanistan. This
region is rich with mines of lapis lazuli.
For Additional Information : (1) Many There was a great demand for this semi-
sites of the Early Harappan and Mature precious stone in Mesopotamia. The
Harappan (Urban) phases have been found Mesopotamian epics
in the basin of the river Ghaggar. The describe Goddess Inanna’s
latest research based on the evidence from palace, the walls of which
sites like Kunal, Bhirrana, Farmana, were embedded with this
Girwad and Mitathal, etc. indicate a stone. This stone was a
possibility of the rise of the Mature very important commodity Lapis lazuli stone
Harappan more than 5000 years ago. If
in the Harappan trade
this is proved in due course, then it may
with Mesopotamia.
be said that it first developed in the
Ghaggar basin. Evidence from Rakhigarhi 2.3 Production, Trade, Organisation
may prove to be instrumental in reaching and Administration
positive conclusion in this context. In the early phase of agriculture, making
(2) The Harappan sites of Mohenjodaro, earthen pots and farming were the jobs of
Harappa, Ganweriwala in Pakistan and women. The pots fashioned by women were
Rakhigarhi and Dholavira in India have handmade. They used a digging stick for
been declares as World Heritage sites. sowing the seeds. With these methods women
could manage production, just enough to
2.2 Interrelationship between Cities meet the family needs or at the most those
and Villages of the extended family. Surplus production
We have seen that the Harappan cities was not possible with their methods.
arose as a result of the process of the The use of bullocks as draft animals (for
development and expansion of the pre- ploughing and transport) had begun in the
Harappan settlements. The city people are Early Harappan period. The use of wheel
dependent on the natural resources and enabled speedy and mass production of pottery.
villages in their vicinity in order to meet the Use of ploughs pulled by bullocks enhanced
needs of urban way of life and urban the agricultural production. The clay replicas

16
Telegram Channel -: Mpsc Super Batch 78 Compiled by Sanjay Pahade 78
of ploughs are recovered from the Harappan were also exclusively industrial settlements.
site at Banawali in Harayana. It became Chanudaro in Sindh was an important
possible to obtain necessary commodities in industrial town. It is apparent that the
exchange of agricultural produce. This was outposts in Gujarat were specially established
the beginning of barter system. The food grains for the convenience of trade. It was easier
were exchanged, especially to obtain salt, to reach the sources of copper, chank shells
metals and precious objects. and semi-precious stones from the outposts
During the Mature (urban) Harappan like Kuntasi, Nageshwar, Bagasara, etc. in
period the sphere of commodity exchange Kutch-Saurashtra. It was also easier to ship
expanded and their import-export increased out the finished goods from these outposts
to a great extent. The internal and distant located near the coast of Saurashtra. Carrying
trade received momentum. Trade with distant out all the operations like procurement of the
places like Mesopotamia became regular. raw material, production of finished goods
Trade with Mesopotamia had begun during and its export from one and the same region,
Early Harappan times. The Akkadian empire certainly proves to be beneficial for trade.
was founded in 2334 B.C.E. by Emperor Harappan people built the trading outposts
Sargon I. During his reign the Harappan with this view in mind. These were small in
trade with Mesopotamia flourished and the extent, but were important industrial centres.
Harappan cities reached their peak. One of Dholavira was the city built to oversee the
the administrative records of emperor Sargon trading transactions in Sourashtra. Lothal
describes the trade with distant places. It was an important sea port.
seems that the importance of trade by land
routes passing through Iran and Central Asia For Additional Information :
had diminished in the Mature Harappan Mesopotamia is known for its grand
period and the sea trade had become the temples. The administrative system of
preferred mode. ‘Dilmun’, ‘Makan’ and Mesopotamian cities revolved around the
‘Meluhha’ are mentioned as three important administration of these temples.
Mesopotamian temples are known as
centres on the sea route. Dilmun is identified
‘ziggurats’. The high priest of the temple
with Bahrain, Makan with the coastal region
used to be the ruler of the city. The social
of Oman-Iran-Baluchistan and Meluhha with life, cultural events, power and hierarchy
the region of Harappan civilisation. Copper of officials, etc. were organised in
was available in large amounts in Meluhha. accordance with the rituals and festivals of
The name Meluhha is supposed to have its the presiding deity of the temple. The
origin in the red colour of copper. The notion of a ‘Priest-King’ of the Harappan
commodities exported from Meluhha to civilisation was formulated by presuming
Mesopotamia included copper, ivory objects, its close similarity with the Mesopotamian
lapis lazuli, carnelian beads, textiles, timber culture. Mesopotamia was rich in
as well as monkeys and peacocks. The bodies agricultural production. However, it did not
(mummies) of dead royalties in Egypt were have enough sources of precious metals,
wrapped in blue coloured cloth. Indigo, gems and timber. These things were
necessary for dyeing this cloth was exported imported there. Many of the Mesopotamian
brick inscriptions contain the lists of
by Harappan traders. Harappan traders are
imported commodities and the place names
likely to have received wool, gold and silver
from where they were imported. The
in exchange for these commodities.
exports from Mesopotamia included textiles,
The Harappan cities had a reserved pottery and leather products.
section for the industrial production. There

17
Telegram Channel -: Mpsc Super Batch 79 Compiled by Sanjay Pahade 79
It is clear that a well organised continuous internal conflicts in Mesopotemia.
administrative system existed to manage the Along with it increasing salination of
industrial production, import-export, the agricultural land was the main reason of the
interrelationship between cities managing decline of the Mesopotamian civilisation. It
trading operations and villages around them. weakened the Mesopotamian market that was
The town planning, standardisation of bricks, available to the Harappan goods.
weights, seals, shapes and ornamentation of 2. Deteriorating environment was the
various objects, etc. confirm the presence of main reason of the decline of Mesopotamia.
such administrative system. However the Similarly, climatic changes and deteriorating
nature of Harappan polity, whether it was a environment were the main reasons of the
single state or a federation of small states, decline of the Harappan civilisation too.
is not yet known. The cities like Mohenjodaro, Around 2000 B.C.E. the climate became
Harappa, Dholavira and Rakhigarhi were increasingly arid and there were frequent
perhaps regional capitals. Cities like famines. Degradation of cultivable land was
Mohenjodaro, Kalibangan, Lothal and on the increase. It is mentioned earlier that
Rakhigarhi were important religious centres. sites of Harappan civilisation have been
2.4 Decline of the Cities discovered in large numbers in the basin of
1. When Harappan civilisation was Saraswati, which is now known as Ghaggar/
discovered, it was presumed that its cities Hakra. The interrelationship between
were destroyed by invading tribes who came Harappan cities and villages that formed the
from outside. Indra is described as ‘Purandara’ support system of the socio-economic life of
meaning the destroyer of fortified cities. The the cities dwindled in this period. The
Harappan cities were fortified, thus befitting devastating earthquake in the Saraswati basin
the definition of ‘pura’. Sir Mortimer Wheeler was the major reason responsible for it. The
on this basis concluded that Indra destroyed earthquake raised the ground level of the
the Harappan cities. In turn, he declared that river bed causing a barrier causing Sutlaj
the Vedic Aryans destroyed the Harappan and Yamuna, the tributaries of the Saraswati
cities under the leadership of Indra. This to change its course. As a result the Saraswati
declaration by Wheeler received a general dried up and the Harappans had to migrate
approval among the scholars at that time. elsewhere. Harappan cities were abandoned.
However, more concrete evidence in the The settlements that came up on the
context of Harappan civilisation is available remains of the Harappan cities were not
now, which does not agree with Wheeler’s developed and rich. These settlements
opinion. It is now known that the Harappan belonged to Late Harappan people. The
civilisation began to decline around 2000- pottery, houses, funerary customs of these
1900 B.C.E. People had to migrate elsewhere. people were different. The Late Harappan
With this, began the Late Harappan period. people were also forced to migrate because
The Harappan civilisation deteriorated as of natural calamities. New villages were
the cumulative effect of various factors such settled by the Mature Harappan and Late
as decline in trade, climatic changes, Harappan people wherever they went. It gave
weakening of the economy, etc. rise to new rural cultures. The spread of
The Harappan trade with Mesopotamia these new cultures is seen in Rajasthan,
was on the decline in the Late Harappan Gujarat, Malawa and Maharashtra. These
period because the Mesopotamian economy new cultures are known as ‘Chalcolithic
had weakened. One reason of it was Cultures’. We will learn about these cultures
in the next lesson.

18
Telegram Channel -: Mpsc Super Batch 80 Compiled by Sanjay Pahade 80
Note : Terminology
(1) Pre-Harappan Cultures - Cultures of the period preceding the Harappan civilisation
(2) Early Harappan period - Early (beginning) Harappan phase
(3) Mature (Urban) Harappan period - Fully developed phase of Harappan civilisation
(4) Late Harappan period - Period after the decline of the mature (Urban) phase
(5) Post Harappan cultures - Rural cultures that came up after the Harappan civilisation vanished.

Exercise

Q.1 Choose the correct alternative and (3) Shortugai Mesopotamia


write the complete sentences. (4) Meluhha Region of Harappan
(1) In the early phase of agriculture, civilisation
making pots and farming
were the jobs of women. Q.3 Explain the statements with reasons.
(a) copper (b) bronze (1) The remains found at cities like
(c) earthen (d) stone Harappa, Mohenjodaro, Kalibangan,
Lothal, Dholavira, Rakhigarhi, etc.
(2)
Lothal
is known for its ancient are the evidence of the past glory of
. Harappan civilisation.
(a) agriculture (b) dock
(c) textiles (d) tools (2) Lapis lazuli had a very important
(3) A general impression prevailed that the place in the Harappan trade.
Harappan seals had some connection (3) Harappan civilisation declined.
with the civilisation. Q.4 State your opinion.
(a) Chinese (b) Greek
(c) Mesopotamian (d) Egyptian (1) The Harappan cities and villages in
the vicinity were interdependent.
(4)
The bodies (mummies) of dead (2) The Harappan cities seem to have a
royalties in Egypt were wrapped in well-organised administrative system.
coloured cloth.
(a) white (b) black Q.5 Answer the following question with the
(c) red (d) blue help of given points.
Q.2 (A) Choose the correct reason and Write about the characteristics of
complete the sentence. Harappan cities with the help of given
The major reason for the decline of the points.
Mesopotamian civilisation was - (a) Town planning
(a) foreign invasion (b) Social organisation
(b) deteriorting environment (c) Administration
(c) loss in trade (d) Economy
(d) migration Activity
(B) Find the incorrect pair from Set B Collect information and illustrations with
and write the correct ones. the help of internet about the town
Set ‘A’ Set ‘B’ planning of the Harappan cities and
(1) Dilmun Bahrain Chandigarh. Compare them.
(2) Makan Oman-Iran-
Baluchistan coastal
region 

19
Telegram Channel -: Mpsc Super Batch 81 Compiled by Sanjay Pahade 81
20
Telegram Channel -: Mpsc Super Batch 82 Compiled by Sanjay Pahade 82
3. Chalcolithic Villages in India

3.1 Chalcolithic Period in India


3.2 Chalcolithic Maharashtra
3.3 Megalithic Period in India

3.1 Chalcolithic Period in India


Design on Harappan Jar - Cemetery ‘H’
When the Mature (urban) Harappan
civilisation collapsed completely, the people on the Mature Harappan pottery. On one of
of Late Harappan cultures who had settled the cemetery ‘H’ pots dead humans are seen
on the ruins of Mature Harappan cities had carried by peacocks in their stomach. In brief
to migrate elsewhere. The urban Harappans the characteristics of the Late Harappan
and the Late Harappans dispersed. Wherever culture were different.
these people reached, new rural cultures came Some archaeologists are of the opinion
into being. that the Late Harappans were perhaps Vedic
The discipline of architecture, town Aryans. However, more research is required
planning of the Mature Harappan period was to know whether the Late Harappans were
absent in the Late Harappan settlements, the successors of the Mature Harappan people
established on the ruins of the Harappan
or whether they were the Vedic Aryans.
cities. The designs found on the burial pots
in ‘Cemetary H’ at Harappa were different. Rajasthan
These designs include motifs like sun, moon, (1) ‘Ahar’ or ‘Banas’ Culture : The
fish, deer and peacock, which do not occur chalcolthic cultures in India generally belong

Revision in Brief : There were these things to the local people wherever
Neolithic villages before the rise of the they went. Thus, new rural cultures with
Harappan cities. Neolithic people had regional variations came into being.
the technology of making copper objects. These cultures are known as Chalcolithic
They were also skilled potters. Eventually, culture. Chalcolithic cultures belonged to
their technology improved and the people who used tools of copper as well
progress led to the origin of the Harappan as stone. However copper was used
cities. sparingly by chalcolithic people.
People of the Mature Harappan and A number of Chalocolithic sites have
the Late Harappan culture had the been found in Rajasthan, the Ganga
knowledge of wheel-made pottery, valley, Gujarat, Bihar, Bengal, Odisha,
cultivation of wheat and barley, making Madhya Pradesh and Maharashtra.
of copper objects. They must have taught

21
Telegram Channel -: Mpsc Super Batch 83 Compiled by Sanjay Pahade 83
to the post-Harappan period. However, the was surrounded by a fortification wall. These
‘Ahar’ or ‘Banas’ culture in the Mewad characteristics confirm that Balathal people
region of Rajasthan was contemporary to the were in close contact with the Harappans.
Harappan civilisation. Balathal and Gilund The Balathal people obtained copper
near Udaipur are the important sites of Ahar from the Khetri mines in Rajasthan. They
culture. ‘Ahar’ culture at Balathal is dated knew the technology of smelting copper. It
to 4000 B.C.E. seems Balathal was the source for the
It was first discovered at Ahar near Harappans to obtain copper and copper
Udaipur, so it was named as ‘Ahar’ culture. objects.
Ahar is located on the banks of Ahar, a (2) Ganeshwar-Jodhpura Culture :
tributary of the river Banas, So it is also Many sites of the culture known as
known as ‘Banas’ culture. ‘Ganeshwar-Jodhpura’ culture have been
The archaeological evidence shows that found in the vicinity of the copper mines at
Balathal was a centre of mass production of Khetri. The settlements there are earlier than
pottery. The pottery made at Balathal was the Harappan civilisation. During the
supplied to other settlements of Ahar culture. excavations at Ganeshwar copper artefacts
Along with pottery, terracotta bull figurines, like arrowheads, spearheads, harpoons,
objects made of chank shell, stone blades, bangles, chisels and also pottery was found.
chisels, arrowheads, copper tools, etc. were The people of Ganeshwar-Jodhpura culture
found here, in large quantities. The houses at supplied copper objects to the Harappans.
Balathal were built of baked bricks, by The Ganga Valley
placing two headers and two stretchers Ochre Coloured Pottery and Copper
alternatingly (English bond method). Balathal Hoards : Initially the Ochre Colured Pottery
(OCP) was mostly found in river beds.
By the way : An archaeological Generally the potsherds of OCP are found in
culture is usually named after the worn out, rounded and brittle conditions as
site where it was discovered first. they remained in flowing water for a long
Every culture has its characteristic time. Now, a number of sites of the OCP
pottery. The pottery is the main clue culture are found in Punjab, Haryana,
Rajasthan and the Western region of Uttar
to identify a particular culture.
Pradesh.
Among the types of pottery of
House floors
the Ahar culture Black-and-Red type
of these people
pottery is important. It is made on
were made by
wheel. The inner side and the outer ramming. On the
portion near the neck is red and rest house floors were
of the pot is black. If the pot is kept found traces of
in a reverted position while baking hearths, terracotta
its inner side turns black and the male figurines
outer surface turns red. Or else, the and bull figurines.
portion, which is covered by husk or Remains of cattle
hay becomes black and the uncovered bones, rice and
portion becomes red. Copper Hoard - Harpoon, barley were also
axe head, ring etc. found. This

22
Telegram Channel -: Mpsc Super Batch 84 Compiled by Sanjay Pahade 84
evidence indicates that people of this culture troughs of various sizes.
stayed in settled villages and practiced Madhya Pradesh
agriculture. (1) Kayatha Culture : Kayatha is a site
The OCP culture in Rajasthan is dated situated on the banks of the river known as
to about 3000 B.C.E. The same culture ‘Chhoti Kali Sindh’, at a distance of 25
existed in Ganga-Yamuna Doab around 2000 kilometres from Ujjain in Madhya Pradesh.
B.C.E. Kayatha culture was contemporary to
The Copper Hoards found in India come the Harappan civilisation. The people of
from various regions, such as Uttar Pradesh, Kayatha culture subsisted on agriculture and
Bihar, Bengal, Odisha, and Madhya Pradesh. animal husbandry. They mainly used
The copper objects found in these hoards handmade pots and microliths. Artefacts like
indicate that the artisans who fashioned them copper axes and bangles, necklaces made of
were very highly skilled. The archaeological beads of semi-precious stones and small disc
sites of OCP and the find-spots of the copper beads of steatite were found in the Kayatha
hoards are often seem to be situated in the houses. People of Kayatha culture and
same vicinity, not very distant from each Harappan culture seem to have been in
other. Hence, OCP and copper hoards are contact much before the rise of the Harappan
supposed to belong to one and the same cities.
culture. After the Kayatha culture people of Ahar
Some archaeologists are of the opinion culture arrived in Madhya Pradesh from
that this culture belonged to the Harappan Rajasthan. It is quite possible that both the
people who migrated after the decline of the cultures co-existed in Madhya Pradesh for a
Harappan civilisation. Some scholars linked short time. Remains of Malwa culture are
the copper hoards to the Vedic Aryans. found in Madhya Pradesh in the times after
However, some other scholars reasoning on Ahar culture.
the basis of the proximity of OCP and copper (2) Malwa Culture : The name ‘Malwa’
hoards, believe that it must have been an obviously tells us that this culure originated
independent culture. and spread first in the Malwa region. Malwa
Bihar, Bengal, Odisha Culture existed in
Copper hoards have been found in Bihar, Madhya Pradesh
Bengal, Odisha and Madhya Pradesh. during 1800-1200
However, OCP is not found in these regions. B.C.E. ‘Navadatoli’
Chalcolithic sites have been discovered situated on the river
in these regions. The excavations at sites like Narmada, on the
Chirand, Sonpur, etc. yielded earthen pots of opposite bank of
black-and-red ware. Shapes of these pots are Maheshwar, is an
similar to those of Harappan culture. It seems important site of
that the Harappan people had gone as far as Malwa culture. The
Bihar and the local cultures were influenced A chalice of Malwa
other important sites
by them. In Bengal and Odisha also some culture - Navadatoli are Eran (District
chalcolithic sites have yielded pottery that Sagar) and Nagda
shows Harappan influence as far their shapes (District Ujjain). They were surrounded by
are concerned. They include, bowls and protective walls.

23
Telegram Channel -: Mpsc Super Batch 85 Compiled by Sanjay Pahade 85
Gujarat : The chalcolithic settlements in (1) Savalda Culture : Savalda is in
Gujarat coincide with the following phases Dhule district. It is situated on the banks of
of the Harappan culture : the river Tapi. Savalda culture is dated to
(1) Early Harappan phase (3950-2600 B.C.E.) 2000-1800 B.C.E. This culture seem to have
(2) Mature (urban) phase (2600-1900 B.C.E.) arisen by the cultural contact between the
(3) Post-Harappan phase (1900-900 B.C.E.) Mesolithic people in northern Maharashtra
There are ample sources of semi-precious and the Harappan people in Saurashtra.
stones in Gujarat. Making beads of these People of Savalda culture at Daimabad
stones was a big industry during Harappan used wheel-made pottery. The designs on
times. The neolithic settlements in Gujarat their earthen pots included arrow heads,
played a major role in procuring these stones. harpoons and figures of various animals.
People residing in the neolithic settlements of People of this culture also used copper
Gujarat were mainly pastoral, that is people objects, beads of siliceous stones, arrow
whose primary occupation was animal heads made of bones, mortar and pestle
husbandry. Probably, some of these pastoral stones, etc. Their village at Daimabad was
people were semi-nomadic. protected by a surrounding wall. They built
There are regional variations in the mud houses and they made the floors by
characteristics of chalcolithic cultures of ramming mud and alluvium together.
Gujarat. The chalcolithic pottery of Kutch- There was a cultural contact between the
Saurashtra and Northern Gujarat are distinct Savalda people and the Harappans in
from each other. The chalcolithic villages in Saurashtra. Its evidence has been found at
Kutch-Saurashtra were abandoned by 1900 the site of Kaothe in Dhule district. The
B.C.E. artefacts made of chank shells found at
In the post-Harappan period there were Kaothe confirm that the Savalda people and
two chalcolithic cultures in Gujarat. The The Harappans in Saurashtra had trade
culture in south Gujarat was known as relations.
‘Prabhas’ culture and the one in northeastern (2) Malwa and Jorwe Cultures : The
Gujarat was known as ‘Rangpur’ culture. First Farmers of Maharashtra
The pottery of these chalcolithic cultures was The people of Malwa culture reached
akin to Late Harappan pottery with regards Maharashtra around 1600 B.C.E. Permanent
to the colour, shapes and designs. These villages of farmers were first established in
cultures existed till 1800-1200 B.C.E. Maharashtra by the Malwa people. They
3.2 Chalcolithic Maharashtra were the first farmers of Maharashtra. After
The evidence for the Late Harappans in arriving in Maharashtra they came into
Maharashtra is found at Daimabad. The contact with the
chalcolithic culture before the arrival of the neolithic people in
Late Harappans at Daimabad is known as Karnataka. It
‘Savalda’ culture. The cultures in the resulted into a few
succeeding period of the Late Harappan were changes in the pot
respectively known as ‘Malwa’ and ‘Jorwe’ making technology
cultures. of Malwa people as
far as shapes of the
For a video on Daimabad, visit: pots and designs are
https://www.youtube.com/watch?v= EzHb1n954fo Earthen Jar-Daimabad concerned. This was

24
Telegram Channel -: Mpsc Super Batch 86 Compiled by Sanjay Pahade 86
the beginning of a new culture known as form of round platforms made for storage
‘Jorwe’ culture. Traces of Jorwe culture were bins, four flat stones used for resting four
first found at Jorwe in Ahmednagar district. legged storage jars and underground silos
The chalcolithic cultures in Maharashtra plastered with lime. People sometimes stayed
had spread in Tapi, Godavari and Bhima in pit dwellings, though its use was rare.
basins. Daimabad, Prakashe, (District Malwa pottery is mostly of buff
Nandurbar) and Inamgaon were the large (yellowish) colour. Designs on these pots are
villages and the main centres in the respective painted in brown. Jorwe pots are well baked
river basins. Other smaller settlements and giving a metallic sound. They are red in
farmsteads were linked to the large centre in colour with
their vicinity. For example Nevase, Nasik designs
were the large centres. Songaon and Chandoli painted in
in the Pune district, Pimpaldar in the Nasik black. Jorwe
district were smaller settlements. Walki near pottery
Inamgaon was a farmstead. Pimpaldar in the includes
Bagalan ghats was situated at a vantage shapes like
point between Tapi and Godavari basins. spouted pots,
Jorwe Pottery
Hence, it held an important position in the carinated
trade, though it was smaller in extent. Walki bowls and troughs (carination is the central
was situated at the confluence of river Ghod ridge on the pot), lota, globular jars, etc. The
and Mula. It was a very small site. It was Late Jorwe people made pots of the same
a farmstead. shapes, but without any decoration. Potter’s
Inamgaon (Taluk Shirur, District Pune) kilns were found in both Malwa and Early
was subjected to extensive, horizontal Jorwe period. They were round in shape. The
excavations. Therefore, indepth cultural kiln of Early Jorwe period was larger and of
information of the site is available. The greater capacity. In the Late Jorwe period
cultural chronology at Inamgaon : potters did not have a specially built kiln.
(1) Malwa Culture-1600-1400 B.C.E. They baked their pots by directly arranging
them on the ground. Inamgaon was the centre
(2) Early Jorwe Culture-1400-1000 B.C.E.
of pottery production, which supplied pottery
(3) Late Jorwe Culture-1000-700 B.C.E. to surrounding villages.
‘Early Jorwe’ was the flourishing phase A house, quadrangular in shape, spacious,
of the Jorwe culture that succeeded the having multiple rooms, or just a round shaped
Malwa culture. ‘Late Jorwe’ is the phase pit dwelling is the indicator of the economic
when the Jorwe culture declined. The condition of the family staying in it. The
excavations at Inamgaon has unfolded the number of round pit dwellings was negligible.
material culture and the day-to-day life of However, the number of quadrangular
the Jorwe people in great details. spacious houses diminished in the Late Jorwe
During the Malwa phase people at period and the number of round huts increased
Inamgaon lived in spacious, rectangular considerably. These round huts were different
houses. The house walls were wattle-and- than the round pit dwellings. These were
daub type. Houses were partitioned into two erected by tying a number of sticks at the
rooms by a half wall. The evidence of upper end and then pegging the lower ends
storage facilities in the houses came in the on the ground by fanning them out. Such

25
Telegram Channel -: Mpsc Super Batch 87 Compiled by Sanjay Pahade 87
huts are usually erected by nomadic people. There was an earlier burial of the same type
The climate in the Late Jorwe period without the skeletal remains. It was a
increasingly became arid. The round huts is symbolic burial. The Early Jorwe people used
the evidence of climatic change that forced globular jars for burying a dead child. In this
the Jorwe people to take up nomadic life. type of burial the dead child was placed in
The Early Jorwe people at Inamgaon two jars, joined mouth to mouth.
dug a canal for storing flood water of the 3.3 Megalithic Period in India
river. It is obvious that this was done for At about 700 B.C.E. Inamgaon was
irrigation purpose. Distribution of this water completely abandoned by the Jorwe people.
was controlled by the chief of the village. Thereafter it was never occupied till the
The chief stayed in a five roomed house built historic period. This situation prevailed in
at the central part of the chalocolithic most of the Maharashtra. However, a nomadic
Inamgaon. There were many underground people of this period erected stone circles by
silos, inside the house and in the courtyard, using huge slabs of rock. The space within
built for storing food-grains. Inamgaon these circles was used to bury dead people.
farmers cultivated wheat, barley, Sorghum, Because of the huge stone slabs used in their
lentil, horse gram (Macrotyloma uniflorum), erection, these circles are known as megaliths.
etc. meat and fish were also part of their The period of these megaliths is known as
diet. Like other chalcolithic sites Inamgaon ‘Megalithic Age’. They were used for various
also yielded coloureful stone beads, microliths purposes, but mainly they contain the remains
and copper objects. of the dead. They were the memorials of the
The rank of the ‘Chief’ of Inamgaon dead.
was an inherited one. The burials found near
the chief’s house confirm this fact. These
burials were completely different from other
burials found at Inamgaon. Generally, a dead
person was buried in a pit in an extended
position. However, in one of the burials
found near the chief’s house the dead body
was seated in a four legged, bulging jar.
Megalithic Burial
Try to complete the following
activity : With the help of clues provided Such megalithic circles are in many
below prepare a flow-chart. parts of the world. Its tradition was prevalent
since prehistoric times. A few tribes practice
Spread Pots in use
it even today. For instance, the Bodos in
Odisha, Todas and Kurumbas in South India,
Nagas and Khasis in Northeast India. Most
The chalcolithic cultures in Maharashtra
of the megalithic circles in India are dated
to 1500-500 B.C.E. Some of the megalithic
circles in north India are as old as the
House plans Means of livelihood neolithic period. The people who erected the
megalithic circles in Maharashtra are supposed
Burial type Diet to have come from south India. The megalithic

26
Telegram Channel -: Mpsc Super Batch 88 Compiled by Sanjay Pahade 88
circles in Maharastra belong to the Iron Age. culture were nomadic artisans. They were
They are dated to 1000-400 B.C.E. skilled in making iron objects. A furnace of
In the Vidarbha region of Maharashtra smelting iron of megalithic period was
Megalithic burials are found in large numbers, discovered at Naikund near Nagpur.
especially in the districts of Nagpur, Megalithic people used horse for transport
Chandrapur, and Bhandara. Excavations of and travel. Skeletal remains and copper
megalithic burials were conducted at various ornaments of horses were found buried in the
sites like Takalghat, Mahurzari, Khapa, megalithic circles. Megalithic people used
Naikund, etc. It is notable that most of these Black-and-Red pottery.
sites were situated on the ancient trade routes. Megalithic people were certainly
However, settlements of artisans of megalithic instrumental to a great extent in introducing
culture are rarely found. People of Megalithic the ‘Iron Age’ in ancient India.

Exercise

Q.1 (A) Choose the correct alternative and Q.2 (A) Explain the statements with reasons.
write the complete sentences. (1) Harappan people had to migrate.
(1) On one of the cemetery H pots dead (2) People of ‘Malwa’ culture were the
humans are seen carried by ........ in first farmers of Maharashtra.
their stomach.
Q.3 State your opinion.
(a) deer (b) peacock It seems that the Harappan people had
(c) fish (d) bull gone as far as Bihar.
(2) The archaeological evidence shows
that Balathal was a centre of mass Q.4 Write short notes.
production of ........ (1) Banas Culture (2) Malwa Culture
(3) Kayatha Culture
(a) stone pots (b) copper pots
(c) earthen pots (d) glassware Q.5 Write about the chalcolithic cultures in
(3) Permanent villages of farmers were Gujarat with the help of the given
first established in Maharashtra by points.
......... people. (a) Period (b) Means of livelihood
(a) Savalda (b) Malwa (c) Harappa (c) Geographical spread (d) Evidence of
(d) Kayatha cultural contact with other people

(B) Find the incorrect pair from set B Activity


write the correct ones. With the help of Internet, reference books,
Set ‘A’ Set ‘B’ field trips, newspaper articles, etc. obtain
(1) Savalda Culture Daimabad pictures of excavated artefacts and
architectural remains and arrange an
(2) Malwa Culture Navadatoli
exhibition under the guidance of your
(3) Ahar Culture Sonpur teachers.
(4) Jorwe Culture Inamgaon


27
Telegram Channel -: Mpsc Super Batch 89 Compiled by Sanjay Pahade 89
4. Vedic Period

4.1 Vedic Culture : Vedic literature, the basis of astronomical events. He was
Linguistics and Archaeology also of the opinion that the original home of
4.2 Vedic Literature and Social the Aryans was in the Arctic region.
organisation of Vedic Times This debate began in the 16th century.
4.3 The Early Vedic Culture as unfolded Till then the concept of the ‘Aryans’ was not
through Vedic Literature known. In the 16th century the European
4.4 Later Vedic Period academics became aware of the similarities
between Sanskrit and Latin-Greek languages.
It resulted into the notion of the ‘Indo-
4.1 Vedic Culture : Vedic literature,
European’ language family, which gave a
Linguistics and Archaeology
momentum to the search of a Mother
It is now confirmed by the research of
many years that the Harappan civilisation By the way : The notion of a
was not destroyed by foreign invaders but it language group with a common mother
declined because of various factors like language seems to have been toyed with
natural calamities and the deterioration of by some European scholars since the
environment. There are multiple unanswered 12th century C.E. However the awareness
questions, such as, who were the Aryans, did of a direct link between Sanskrit and
they arrive in India from a distant place or Latin attained the status of academic
were they natives of India, which are the discussions on much later in the 18th
archaeological sites where the remains of century. We can trace the beginnings of
their culture are found, how to identify those this kind of interest to the 16th century.
remains, etc. Most of the information about In the year 1583, an Italian merchant by
their culture is derived from the Vedic the name Fillippo Sassetti came to Kochi
literature. This literature is fundamentally in (Cochin) the port city in Kerala. He
the form of the faith systems of the Vedic never returned to his motherland. He
tribes and eulogies of their deities. stayed in Kerala and Goa. During his
Mention of the material culture occurs in stay he wrote detailed letters to his
the Vedic literature as an unintended aspect. family members about Indian life,
Also, there are references to god Indra’s language and culture. He happens to be
victories over their enemies. It was attempted the first European who wrote about his
to find answers to above questions, on the observations of the Indian society. He
basis of these details. There are various also studied Sanskrit. He was the first
opinions about the chronology of the Vedic one who observed the similarities between
culture. However, there is a general agreement Sanskrit and Latin. His observations
that the Vedic people composed Rigveda in could be said to be the first, to give
India around 1500 B.C.E. However, Lokmanya impetus to the notion of a family of
Tilak calculated this date as 6000 B.C.E. on Indo-European languages.

28
Telegram Channel -: Mpsc Super Batch 90 Compiled by Sanjay Pahade 90
language from which developed the Indo- (urban) Harappan phase was earlier. Many
European languages. It further led to the scholars are inclined to accept that succeeding
development of a branch of linguistics known Late Harappan culture was equivalent to the
as ‘Philology’. Philology is a discipline, Vedic culture.
which traces the etymological history and The Saraswati (Ghaggar-Hakra) to the
meanings of words. east, Sindhu to the west and the five rivers
At about the same time European of Punjab – Shatdru (Sutlaj), Vipasha (Bias),
scholars had also developed interest in the Asikni (Chinab), Parushni (Ravi) and Vitasta
ancient Indian literature. As a result of this (Jhelum) – together comprised the region
interest ‘Asiatic Society of Bengal’ was known as ‘Saptasindhu’. Besides these rivers
established by Sir William Jones in 1784. mentioned in the Vedic literature are Kubha
This period saw the beginning of compiling (Kabul), Gomati (gomal), Suvastu (Swat),
and translating Sanskrit texts in European etc. Vedic people called the region enriched
languages. Also, systematic research in the by these rivers as the ‘God’s Country’
field of Vedic literature and archaeological (Devnirmit Desh). However, there is no
cultures had gained academic importance. mention of the region of their origin, if they
On the basis of the research in these two had arrived in India from outside.
fields, notions like the authors of Vedic Some scholars believe that Vedic Aryans
civilisation came form the West; they brought had stayed in the region of Saptasindhu from
the knowledge of horses and speedy chariots the beginning. Now many scholars are
having spoked wheels, as well as the science searching for the answer to the question,
of mounted archery, got deeply rooted among whether the Harappans and the Vedics were
academics. In brief, it was believed that the one and the same. Apparently no final
techniques of Vedic warfare was superior to answer has been found yet. The answer may
that of the Harappans, who stayed in fortified be found if researchers could decipher the
cities. Also, it was supposed that the Harappan script.
Harappans were the enemies of the Vedic 4.2 Vedic Literature and Social
people, mentioned as ‘Dasyu’ in the Vedic organisation of Vedic Times
literature.
The Vedic literature is supposed to be
The archaeological research has now the earliest literature of India. Its language
proved that the origin, spread and decline of is Sanskrit. The four Vedas namely, Rigveda,
Harappan civilisation took place mainly in Yajurveda, Samaveda and Atharvveda form
some parts of Afghanistan, Baluchistan and the core of the Vedic literature. The texts of
Iran as well as in Punjab, Harayana, these four Vedas are known as the ‘Samhitas’.
Rajasthan and Gujarat. The Geographic ‘Vid’ in Sanskrit means to know and ‘Ved’
region described in the Vedic literature means the knowledge.
concurs with it. However, there is no
The Rigveda contains Suktas (hymns)
agreement among the scholars about the
composed to eulogise the deities. The verses
chronology of the Vedic Aryans. Therefore,
in Rigveda are known as ‘Rucha’. A number
in spite of a number of academic declarations,
of Ruchas strung together makes a Sukta
nothing conclusive can be said about the
(hymn). Many Suktas make one Mandala.
interrelationship between the Vedic and the
Harappan cultures. Chronologically Mature The Yajurveda offers explanation of the
sacrificial rituals. It explains when and how

29
Telegram Channel -: Mpsc Super Batch 91 Compiled by Sanjay Pahade 91
the mantras should be used. A Rigvedic by dividing the span of human life into four
Richa, when recited in sacrificial rituals, is successive stages, namely
regarded as Mantra. The Yajurveda is a Brahmacharyashram, Grihasthashram,
combined composition of the Rigvedic richa Vanaprasthashram and Sanyasashram. In the
in verse and the explanation of its use as a first stage one was expected to spend an
mantra in prose. austere life and earn knowledge and necessary
The Samaveda is a text that gives the occupational skills. In the second stage one
rules of reciting mantras in a musical form. was expected to fulfil one’s prescribed duties
The Samaveda is regarded as the text that with one’s wife by his side. In the third stage
is fundamental in the development of Indian one was expected to retire from the active
music. life of the householder and if the need be,
The Atharvaveda is about the day-to-day to give advise to the younger people. It was
life. It contains information about charms also desirable that one shuld stay away from
and medicines for various problems and human settlement. In the fourth and the last
diseases. It also talks about the norms of stage one was expected to renounce all the
statesmanship. family bonds, accept a wandering life and to
go away. A Sanyasi was not permitted to
Later, Brahmana texts, Aranyaka texts
settle at one place for a longer time.
and Upanishadas were created. They are also
considered as integral part of the Vedic 4.3 The Early Vedic Culture as
literature. Vedic literature was created over a unfolded through Vedic Literature
prolonged time. Its completion took about The Rigvedic culture is the culture of the
1500 years . It is the only source of Early Vedic period. The tribes of the Early
information about the life of Vedic people. It Vedic period staying in the Saptasindhu
offers a glimpse of the organisation of Vedic region have been mentioned in the Rigveda.
society, Vedic family and the day-to-day life Among them occur the names like Puru,
of the Vedic people. Anu, Yadu, Druhyu, Turvash, etc. They
Varna System were the tribes who subsited on agriculture.
The Vedic society was organized into The Rigveda mentions a battle known as
four classes known as ‘Varna’, namely, ‘Dashradnya Yuddha’ that was fought among
Brahmana, Kshatriya, Vaishya and Shudra. ten tribal chiefs. In the Early Vedic period
The varna system is first mentioned in the along with the Rigvedic tribes, other local
tenth mandala (chapter) of the Rigveda. In tribes also stayed in the Saptasindhu region,
the later Vedic period the varna system lost who did not belong to the Rigvedic people.
its flexilblity. Also the caste system got They were called ‘Das’, ‘Dasyu’ and ‘Pani’.
rooted firmly by this period. In the beginning Panis were looked upon as enemies by the
the varna or the caste was decided by one’s Vedic people. Panis used to steal the cattle
occupation. Later it came to be determined of the Vedic people.
on the basis of birth. Hence it became The tribal settlements and the Vedic
impossible to change one’s varna and caste, people residing there have been mentioned as
in which he/she was born. ‘Krishtya’ in the Rigveda. ‘Krish’ means
Ashrama System ploughing. Hence the people who ploughed
The Ashrama System of the Vedic culture and also their settlements were metioned as
lays down the norms of living an ideal life ‘Krishtya’. The importance of agriculture is

30
Telegram Channel -: Mpsc Super Batch 92 Compiled by Sanjay Pahade 92
You would like to know : There is certain that the Later Vedic people
a school of thought, according to which, migrated by two different routes. The
the Late Harappans, that is the Vedic Vedic literature mentions ‘Uttarapath’
people, migrated because of adverse (The northern route) and ‘Dakshinapath’
climatic conditions and natural calamities. (the southern route). Both of these routes
They migrated to Ganga-Yamuna Doab in pass through regions, which differed
the east and also went toward the west drastically. Accordingly their environment
to Iran, Iraq, Egypt, etc. The evidence also differed. Considering this, it becomes
of the inscriptions found at ‘Boghazkui’ clear that the ancient people managed to
in Iraq is used to confirm it. These traverse through extremely difficult
inscriptions contain the text of a treaty terrains.
between two tribes called Hittite and The northern route extended from
Mittanni. It begins by invoking deities Central Asia to Saptasindhu and from
like Indra, Varuna and Nasatya. This and there to Himalayan foothills and then
similar evidence is offered by some of the proceeding to Ganga-Yamuna Doab,
European and Indian scholars to prove reaching the delta region thereafter.
that Vedic people arrived in India from The southern route formed a link
the West. This is one of the examples to between the Ganga-Yamuna Doab and the
illustrate the way the riddles in history are southern regions of India. It extended from
created. the southern part of Sindh, through Kutch,
So far, scholars have been unable to South Rajasthan, Malwa and from there
solve the riddle of the ‘Aryans’ by putting to the Deccan Plateau. We have seen in
together the archaeological and literary the previous lesson that the Harappan
evidence. Nevertheless, it seems to be people reached Maharashtra via this route.

clearly emphasised in the 10th mandala of the palace of Lord Varuna having 1000 doors.
Rigveda. This description is an allegory, indicative of
It states, that for the farmer his the imagination of the poet. There is no
ploughshare is the means of obtaining food. evidence available so far to support this kind
The deities, Ashwins and Indra were supposed of Vedic architecture.
to be the gods of agriculture. It is mentioned Along with agriculture the Early Vedic
that the Ashwins hold the plough and make people also practised animal husbandry.
available food in large quantities. ‘Urvara’ Cattle, Buffaloes and horses formed part of
means cultivable land prepared by ploughing. their pastoral wealth. The god who protected
Indra is mentioned as the ‘Lord of Urvara’ their animals was known as ‘Pushan’. The
(Urvarapati). Barley was the main crop chariot makers in this period were known as
grown by the Vedic people. The 10th mandala ‘Rathakara’ and the carpenters were known
of the Rigveda mentions well water used for as ‘Takshan’. Rathakar and Takshan were
irrigation. According to its description a stone artisans of importance. A Rathakara was
wheel tied with earthen jars (Rahatgadage in supposed to have a good knowledge of
Marathi) was used to draw water from the carpentry too. The carpenters used to make
well. It is instructed to secure the belts firmly wooden utensils, equipments necessary for
around the jars. The Rigved describes the performing the sacrificial rituals and other

31
Telegram Channel -: Mpsc Super Batch 93 Compiled by Sanjay Pahade 93
objects of domestic utility. The Rigveda Varuna and Ashwins were the lords of water
mentions boats with hundred oars known as routes. The Rigveda also mentions trade by
‘Aritram’. These descriptions afford a glimpse exchange, negotiations and traders travelling
of the skills of the carpenters of those times. far and wide to earn profits. At times, though
The Rigveda does not have a specific rarely, an ornament called ‘Nishka’ was used
term for the potter. However there are like money.
separate terms for utensils of specific use. 4.4 Later Vedic Period
‘Ukha’ was the term used for a cooking pot.
Later Vedic Period is dated to around
The other terms like ‘patra’, ‘kumbha’,
1000-600 B.C.E. The information about this
‘kalash’ etc. also occur. A potter is mentioned
period is gathered from the treatises written
as an artisan in the Yajurveda for the first
in that period. The material culture as
time as ‘Kulal’.
reflected in the epics, Ramayana and
‘Vayya; means the weaver, and its Mahabharata was studied with the help of
feminine as it occurs in the Rigveda is archaeological evidence. A picture of the
‘Vayyaa’. Along with this there are terms migration in the Later Vedic period can be
associated with the textile industry. ‘Tantum’ gathered from the Saptasindhu region toward
(warp), ‘Otum’ (weft), Shuttle (tasar), are a the east and its geographic markers from the
few of them. Rigveda also mentions woollen literature of that period. This period saw the
cloth. ‘Urna’ is the word for wool. However, spread of Later Vedic culture from the
Rigveda does not mention cotton or silk foothills of Himalayas in the north to the
cloth. Vindhya mountains in the south. There is a
The artisan who worked on leather story of Videgha Mathava (Videha Madhava)
was known as ‘Charmnma’. Various objects in this context that occurs in the Shatapatha
of leather are mentioned in the Rigveda. Brahmana. It appears in this story that the
Objects made of gold (hiranya), of copper- Later Vedic people were settling and bringing
bronze or iron (ayas) are mentioned. Iron the land under cultivation from the west to
was not yet common in the Early Vedic the east.
period. The artisans who made the metal The Later Vedic period saw a gradual
objects were known as ‘Karmar’. The Varna formation of confederacies of the Vedic
system seems to have established by the end villages. They were known as ‘Janapada’.
of the Early Vedic period. The 10th mandala Generally, the seniors and the elites in a
of the Rigveda in which this system finds the Janapada collectively took social decisions.
first mention is supposed to be composed at They functioned like oligarchic states. The
the end of the Early Rigvedic period. powerful Janapadas among them gradually
The chariot has been already expanded into Mahajanapadas. In the next
mentioned. A cart was known as ‘Anas’. The lesson we will study the history of the 4
river transport was known as ‘Navya’. centuries from 1000-600 B.C.E.
Pushan was the lord of the land routes while

32
Telegram Channel -: Mpsc Super Batch 94 Compiled by Sanjay Pahade 94
Exercise

Q1. (A) Choose the correct alternative and Q.2 Complete the concept map given below.
write the complete sentences.
Tribes Dashradnya Yuddha
(1) The importance of agriculture is
clearly emphasised in the
mandala of the Rigveda.
(a) 4th (b) 10th (c) 8th (d) 6th Early Vedic
(2) A potter is mentioned as an artisan Culture
in the for the first time.
(a) Rigveda (b) Yajurveda
(c) Samaveda (d) Atharvveda Means of Deities
(3) The god who protected the animals Livelihood
was known as .
(a) Indra (b) Pushan Q.3 Choose the correct reason and complet
(c) Ashwin (d) Varun the sentence.
(B) Find the incorrect pair from set B Panis were looked upon as enemies
and write the correct ones. by the Vedic people because -
Set ‘A’ Set ‘B’ (a) they belonged to a different tribe.
(1) Nishka Gold ornament (b) their language was impure
(2) Barley Main crop of the Vedic (c) Panis used to steal the cattle of
people the Vedic people
(3) Krishtya The apparatus of (d) Panis did not obey the orders of
drawing water from the Vedic people.
the well Q.4 State your opinion.
(4) Navya river transport (1) There is a debate about the original
(C) Write the names home of the Aryans.
(1) A branch of linguistics - (2) The Rigvedic people subsisted on
(2) A battle fought among the ten tribal agriculture.
chiefs - Q.5 Explain the following concepts.
(3) Cultivable land prepared by (1) Origin of Aryan people
ploughing - (2) Indo-European family of languages



33
Telegram Channel -: Mpsc Super Batch 95 Compiled by Sanjay Pahade 95
5. Janapadas and Republics

5.1 ‘Jana’ and Janapadas


5.2 Janapada 5.2 Janapada
5.2.1 Territoriality and The term ‘janapada’ occurs in the
Autonomous Functioning Brahmana texts for the first time. Thereafter,
5.2.2 Expansion and Development it is frequently used in the Vedic literature
of the Janapadas and the epics – Mahabharata and Ramayana,
5.3 Federation of States (Ganarajya) as also in the Jain and the Buddist literature.
Considering the ancient Indian geographic
perception with reference to the janapadas,
5.1 ‘Jana’ and Janapadas the said literature seems to divide the Indian
Vedic people used the term Jana to subcontinent into five sections : ‘Praachya’-
designate a group of people, united under a of the east, ‘Praatichya’- of the west,
common bond of singular kinship structure. ‘Udichya’ - of the north, ‘Dakshina’ - of the
Their settlement was known as ‘Grama’. A south, ‘Madhyadesha’ - The central region.
cluster of gramas consisting of the same jana However, this division does not include
was known by the name of that particular the entire subcontinent. It mainly focuses on
jana. In the beginning, however, the concept the region to the north of the Vindhya ranges.
of jana included only an extended family The geographic awareness seems to have
(kul), its settlement (grama) and the cattle- expanded in the Puranic literature. The
pen (goshtha-gotra-gaulwada). The concept Purana texts mention the regions to the south
of jana was not yet inclusive of the concept of the Vindhyas also. These are,
of territoriality. Hence, the geographic ‘Dakshinapatha’ and ‘Aparanta’, that is the
boundaries of a jana were not defined. coastal region of Konkan.
In the last lesson we saw that the Vedic 5.2.1 Territoriality and Autonomous
people migrated and spread from the Functioning
Saptasindhu region to the delta region of the It may be noted, that the sense of
Ganga. Vedic janas gradually began to settle territoriality and the ensuing awareness of
in the new region. The concept of jana autonomy were the main factors responsible
increasingly acquired the sense of territoriality. for the formation of ancient janapadas in
The sense of territoriality instilled the sense India. However, their administrative system
of geographic boundaries among the janas. It was not much different from that of the
resulted into the emergence of informal local janas in the Rigveda. Two assemblies known
administrative structures among the janas. as ‘Sabha’ and ‘Samiti’ were at the apex of
Thus, independent Janapada came into all administrative decision making process,
existence. A region occupied by a janas was since the very beginning of the Vedic period.
called as Janapada. Gradually the Janapadas The chief of the janapada was known as
had more formal administrative structures ‘Rajan’. The Sabha and Samiti had the
transforming them into independent states. authority either to elect or to remove the
These were the first well-established states of chief. However, the administrative objectives
ancient India. However, this does not and policies, the norms of social organisation,
necessarily mean that every Janapada evolved etc. were flexible enough to accommodate
into an independent state. changing socio-geographic milieu.

34
Telegram Channel -: Mpsc Super Batch 96 Compiled by Sanjay Pahade 96
When the successive generations got kula (clan) known as the Bharatas in
established in a geographic periphery, the Rigveda, got merged with the Kurus. The
newly emerging social organisation was also two kulas united to such extent that the
changing. Now, it was not confined only to people in Bharata clan came to be regarded
the basic kinship structure. People from the as the ancestors of the Kurus.
outside of the kinship framework also got 3. The more powerful janapadas
included in the janapadas. The social conquered the less powerful ones.
organisation of the janapadas, which was
based on collective functioning began to For additional information: Some of
change and one’s immediate family became the ancient janapadas and the literary texts
more important. Accordingly, the social that mention their names.
organisation came to revolve around the head • Praachya : Anga, Magadh
of the family and family based rituals. Co- (Atharvaveda)
operation at individual level and among the : Kikata (Rigveda and
families became the base of the social Atharvaveda)
organisation. Pundra (Mahabharata)
This period also saw the rise of the • Praatichya : Anu, Alin, Bhalan,
warrior class. These people were well-trained Druhyu, Parashu,
in warfare. This class of warriors played a Pakhta, Puru, Turvasha,
major role in the shift of the primary Yadu (Rigveda),
organisation of the janapadas to more Gandhara (Rigveda and
complex state organisation. Janapadin is a Athrvaveda),
term mentioned in Panini’s Ashtadhyayi­, used Shalva (Mahabharata)
to indicate the warrior class. • Udichya : Krivi, Vaikarna
(Rigveda)
5.2.2 Expansion and Development of the Balhik (Atharvaveda)
Janapadas
• Dakshina : Andhra (Mahabharata)
The expansion and development of the
Pulinda (Emperor
janapadas seem to have occurred in three Ashoka’s edicts)
different ways:
• Madhyadesha : Aja, Chedi, Bharata,
1. Expansion and development of a Matsya, Shigru, Trutsu,
society formed into a Jana by uniting of a Ushinara, Yakshu
number of generations of a singular kula (Rigveda), Kuru,
(family). For example, the janapadas, namely, Shrinjaya (Rigveda and
Matsya, Chedi, Gandhara, Kashi, Kosala, Atharvaveda)
etc.
2. Janapadas rising out of the union of 5.3 Federation of States (Ganarajya)
more than one kula. For example, the The Later Vedic literature, Jaina and
Panchala janapada. Who were the five janas Buddhist literature indicate that most of the
united under the name of Panchalas, is not janapadas were monarchic states. However,
exactly known. According to the renowned some of the janapadas were of oligarchic
historian, Hemchandra Raychaudhuri type. The ancient literature mentions different
following were the janas who merged together types of states. Terms like Rajya, Svaarajya,
as Panchalas : Krivi, Turvasha, Keshi, Bhaujya, Vairajya, Maharajya, Saamrajya
Shrinajaya and Somaka. Later, Kurus and and Parmeshthya occur in this context.
Panchalas are mentioned together as Kuru- However, nothing more can be said about the
Panchalas. By the time of Mahabharata the nature of these states. We get to know that

35
Telegram Channel -: Mpsc Super Batch 97 Compiled by Sanjay Pahade 97
Uttar Kuru and Uttar Madra were of
For additional information : Ancient
‘Vairajya’ type. It means that the state was
Indian literature mentions two more types
not ruled by a monarch but the members of
of ganasanghas. ‘Ayudhjivi’ sangh and
the janapada run the state collectively. The
‘Varta-Shastropajivi’ sangh. They existed
ancient literature mention such type of states
in the northwest regions of the Indian
as ‘ganasangha or sangharajya’ (oligarchy/
subcontinent. The ganasangha of the
republic).
Trigartas was mentioned as Ayudhajivi.
‘Gana’ means the ruling class comprising These people were skilled warriors and
members of equal social status. Similarly warfare was the means of their livelihood.
sangha means a state formed by many kulas For example Yaudheya, Malava and
or janapadas by coming together. By 6th Kshudraka. ‘Varta’ means trade and
century B.C.E. many sangharajyas had come commerce. The people in the Varta-
into existence. Shastropajivi ganasanghas lived by trade
There were three main types of the and commerce, agriculture and animal
ancient federation of states in India. husbandry, as well as their skills in
(1) Ganarajya of the members of the same warfare. People in the Kamboj and
kula. For example, Malava and Shibi. Surashtra ganasanghas earned their
livelihood by these means.
(2) Ganarajya created by more than one
kulas coming together. For example,
Vajji Ganasangha. It included eight Each elective representative of respective
kulas. Vajji, Lichchhavi, Dnyatruk and regional zone was designated as
Videha were the important ganas among ‘Ganamukhya’. Every ganamukhya was the
them. Lichchhavi was most influential member of the assembly known as
gana of them. ‘ganaparishada’. Ganaparishada was the
highest decision-making authority concerning
(3) More than one ganrajyas coming together
all the administrative matters of the
to create a sangharajya. For example
ganasangha. The decisions made by the
Yaudheya-Kshudrak Sangh.
ganaparishada, were implemented by
We can get a glimpse of the administrative designated functionaries of various cadre.
system of the ancient ganasanghas in the The ganapramukha was the principal
Buddhist texts. Each ganasangha had its own functionary presiding over the administrative
system of decision making and its system. He was known as the ‘Adhyaksha’
implementation. We may find approximate or ‘Raja’. The rest of the main designated
parallels of these systems in the following functionaries included, Uparaja
types: (Upadhyaksha), Senapati and Bhandagarika
1. Democratic States : Some of the (Koshadhyaksha).
ganasanghas were divided into regional zones 2. Oligarchic States : In this type the
called ‘Khanda’. They functioned through a elite class in the society held all the powers
group of elected individuals, who were found of decision-making and administration. Panini
capable. Each of the elected members and Kautilya mention them as
represented his respective khanda. These ‘Rajshabdopajivi’ sangh. Panini includes
elected members were installed with collective Vajji, Andhaka, Vrishni, Yaudheya in the
authority for the smooth running of the Rajashabdopjivi type. Kautilya includes the
ganasangha. This was a democratic system. Vrijji or Vajji, Madrak, Kuru, Panchala, etc.
Ganasanghas which functioned in this in this type. This type of ganasanghas were
democratic way existed in Punjab and Sindh more prevalent in the eastern region of Uttar
at the time of Alexander’s invasion. Pradesh and Bihar.

36
Telegram Channel -: Mpsc Super Batch 98 Compiled by Sanjay Pahade 98
We have seen that the janapadas mainly
By the Way : The ganasanghas emerged as the result of the sense of
of Vajji, Shakya, Lichchhavi and Malla geographic boundaries (territoriality) and
were closely associated with the life of autonomous functioning. Some of the powerful
Gautama Buddha. Gautama Buddha was janapadas expanded and developed to the
born in the Shakya family. His father extent that they were acknowledged as
Shuddhodan was an elected president ‘Mahajanapadas’ by the 8th century B.C.E.
(Rajan) of the Shakya Ganaparishad. In the next lesson we will study the sixteen
mahajanapadas of ancient India.

Exercise

Q.1 (A) Choose the correct alternative and Q.2 Choose the correct reason and complete
write the complete sentences. the sentence.
(1) ‘A region occupied by the ‘janas’ The ganasangha of the Youdhey, Malav,
was called as . Kshudraka were mentioned as Ayudhajivi.
(a) Ganarajya (b) Ganasangha Because –
(c) Janapada (d) Gotra (a) it was in the northeast region of the
(2) The principal functionary of a Indian subcontinent.
ganasangha was known as . (b) these people were skilled warriors
(a) Senapati (b) Bhandagarika and warfare was the means of their
(c) Raja (d) Uparaja livelihood.
(3) The author of the ‘Ashtadhyayi’, (c) they were skilled in trade and
which mentions ‘janapadin’, was commerce.
. (d) these were the ganasanghas dependednt
(a) Kautilya (b) Panini on agriculture and animal husbandry.
(c) Chanakya (d) Vyas Q.3 Complete the concept chart.
(4) The sense of territoriality and the
Later Vedic Ancient Literature Brahamana
ensuing awareness were the
Literature mentioning Texts
main factors responsible for the
‘Janapada’
formation of ancient janapadas in
India.
(a) of unity (b) of authority Q.4 Explain the concept with examples.
(c) of autonomy (d) of public (1) Ganarajya and Sangharajya
authority (2) Vartashastropajivi ganasangh
(B) Find the incorrect pair from set B (3) ‘Jana’ and ‘Janapada’
and write the correct ones. Q.5 Answer the following question in detail.
Set ‘A’ Set ‘B’ Describe the democratic and oligarchic
(1) Praachya of the east states in ancient India.
(2) Praatichya of the west
Activity
(3) Udichya of the north
Present an act in the class, based on the
(4) Aparanta the region to the north
simulation of the administrative system of
of Vindhya ranges
an oligarchic state.


37
Telegram Channel -: Mpsc Super Batch 99 Compiled by Sanjay Pahade 99
6. Second Urbanisation in India

6.1 Rise of Mahajanapadas For additional information:


6.2 Ashmak/Assak Mahajanapada The Sanskrit ‘Ashmak’ and the Pali
6.3 Second Urbanisation in India ‘Assak’ are supposed to be two versions
of the same name. This can be further
6.4 Mahajanapadas and The explained by the following examples.
Contemporary Cities
(1) ‘Avantyashmak’ is a term
6.5 Mahajanapadas – Administrative mentioned in Panini’s ‘Ashtadhyayi’. It
System, Guilds means Avanti and Ashmak were two
6.6 Philosophy and Various Sects adjacent states.
6.7 New Religious Trends (2) Three states of ‘Ashwayana’,
‘Ashwakayana’ and ‘Hastikayana’ are
mentioned in Panini’s ‘Ashtadhyayi’.
When Alexander invaded India, his
6.1 Rise of Mahajanapadas
advent through Afghanistan and Punjab
The ambition of territorial expansion was fiercely resisted by some warrior
led to a conflict among the janapadas. tribes. The Greek historians have
Some janapadas proved more powerful than mentioned their names as ‘Aspasioi’,
the others in this conflict. The stronger ‘Assakenoi’ and ‘Astakenoi’. These
janapadas began to annex the conquered three names are equivalents of
territories to their own and thus, they ‘Ashwayana’, ‘Ashwakayana’ and
successfully expanded their boundaries. By ‘Hastikayana’ respectively. Assaka
600 B.C.E. sixteen mahajanapadas were mentioned in the Buddhist literature
established in India, from the northwest may be identified, according to some
region to Magadha. Conquering other historians, with the Ashwakayanas in
janapadas and annexing their territory the northeast region, which may not
permanently to one’s own, became a regular have any association with the Ashmakas.
practice in the times of mahajanapadas. (3) It seems that the king of Assaka
Ultimately, this conflict resulted into the was known as ‘Andhakraja’ during
creation of the large empire like Magadha. Gautama Buddha’s times. The region on
Ancient India once again witnessed the rise the banks of the Godavari was known
of cities. This process is known as the as ‘Andhakratta’. Sage Bavari, a
‘Second Urbanisation’. contemporary of Gautama Buddha, had
The names of sixteen mahajanapadas his ashram in this region. Originally he
are found in the Jaina and Buddhist was from the state of Kosala. According
literature and also in the Purana texts. to ‘Suttanipata’, a Buddhist text, he
Buddhist texts were written in a period, came from Shrvasti, the capital of
which was closer to the period of the Kosala to Assaka. It was situated on
mahajanapadas. Hence the names occurring the Godavari and in the region of
in the Buddhist texts are accepted as more ‘Dakshinapatha’. This evidence confirms
reliable. that the name Assaka in Pali is
equivalent to Sanskrit Ashmaka.

38
Telegram Channel -: Mpsc Super Batch 100 Compiled by Sanjay Pahade 100
Telegram Channel -: Mpsc Super Batch 101 Compiled by Sanjay Pahade 101
6.2 Ashmak/Assak Mahajanapada According to ‘Mahagovinda Suttanta’, a
Among the sixteen mahajanpadas the Buddhist text, Brahmadatta was the king of
one known as Ashmak or Assak is identified Assaka and the name of his capital was
with the region of present day Maharashtra. ‘Potana/Podana’. Potana is identified with
Ashmaka/Assaka was the only Nandura in Buldhana district. Potana was
mahajanapada that was situated in the also known by the names, ‘Potali’ and
region known as ‘Dakshinapatha’. ‘Paudanya’. Other janapadas situated in the
Remaining fifteen mahajanapadas were in region of Dakshinapatha were ‘Vidarbha’,
north India. Suttanipata describes the region ‘Bhoja’, ‘Dandaka’ and ‘Kalinga’. The first
of Dakshinapatha in great details. It was the three were part of present day Maharashtra.
region of an important trading route. It Also the tribal states of the Andhas, Shabaras,
started at Shravasti and proceeded through Pulindas and the Mutibas were also located
Ujjayini and Mahishmati. From there one in the Godavari and Krishna basins.
would reach Pratishthan after crossing the According to Jaina texts, Bahubali, the
Vindhyas. son of Rishabhanath the first Jaina

39
Telegram Channel -: Mpsc Super Batch 102 Compiled by Sanjay Pahade 102
You would like to know : The following have been a feudatory of the king of Kasi.
examples will throw light on the challenges (3) The Jataka story known as ‘Chulla
faced by historians when a single name is Kalinga Jataka’ describes the battle
mentioned in varying contexts. between King Assaka and the Kalinga
(1) The jataka story known as ‘Nimi Jataka’ king of Dantapur. King Assaka defeated
gives us the list of kings who ruled the the Kalinga king and married his sister.
state of Videha from its capital city, This marital alliance helped to establish
Mithila. It includes a king whose name congenial relations between the two states.
was ‘Assaka’. In the Hathigumpha inscription of
(2) The Jataka story known as ‘Assaka Kharavela, it is mentioned that King
Jataka’ tells us about a king named Kharvela, without fearing the dreaded
Assaka who ruled the state of Kasi, and name of Satakarni, attacked the city of
his queen. In this story, it is mentioned Asikanagara in the west and created a
that Potali was the capital of the king fright among its residents. Some historians
Assaka. The capital of ancient state of identify ‘Assaka’ occurring in the Jataka
Kasi was Varanasi. However, it is inferred stories with ‘Asiknagara’ mentioned in the
that the mention of Potali as the capital Hathigumpha inscription.
of king Assaka may indicate that he might

Tirthankara was given the kingdom of Characteristic pottery of janapadas and


Ashmaka and Podanpur was its capital. mahajanapadas has been recovered from
The magnificent statue of Bahubali stands various archaeological sites. Also, many
at Shrvanbelgola, in the Hassan district of sites have yielded iron implements and
Karnataka. Bahubali had attained Keval punch marked coins of silver and copper.
Gyan (Omniscience). The punch marked coins have been
6.3 Second Urbanisation in India mentioned in the ancient Indian literature
The janapadas with definite geographic as ‘Karshapana’ or ‘Pana’.
borders and administrative system were The following things are perceived as
established roughly around 1000 B.C.E. The markers of urbanisation : Development of
ambition to expand geographic boundaries the cities as administrative centres, network
and the political conflict caused by it left of surrounding villages to cater to the needs
some janapadas more powerful than others. of urban centres (hinterland), interlinking of
It resulted into the creation of sixteen the village administration with the central
mahajanapadas from Afghanistan in the administration, well defined rules of taxation,
northeast to Bengal in the east, stretching internal and distant trade, well-established
to the banks of the Godavari in the south. network of land routes and waterways (both
The capital cities of the mahajanapadas rivers and oceans), purchase and sales of
and some other cities, which flourished goods by increasingly using monetary
because of prospering trade once again exchange along with the barter system, a
brought the age of urbanisation in India. It stable legal system and judiciary. All these
is known as the ‘Second Urbanisation’. characteristics of urbanisation were present
The ‘Mahaparinibbansutta’ mentions in the times of mahajanapadas.
names of six cities, which were of great 6.4 Mahajanapadas and The
importance. They are, Champa, Rajagriha, Contemporary Cities
Shravasti, Saketa, Kushambi and Varanasi. 1. Kasi : This mahajanapada was a
By the 6th century B.C.E. these and a few powerful one in the beginning of the
other cities had become very prosperous.

40
Telegram Channel -: Mpsc Super Batch 103 Compiled by Sanjay Pahade 103
Pay attention to this : The Harappan in it, etc. is available in various literary
civilisation originated as a matter of course texts. For instance, enough information is
of co-ordinating trade and production available in the jataka literature about the
processes. Various factors involved in this cities visited by Gautama Buddha in his
chain as well as their smooth functioning lifetime. A moderately accurate picture of
gave rise to the Harappan city centres. the administration of these cities can also
However, as far as the statehood and the be gathered from this literature. The names
administrative machinery of the Harappan of the kings, who were contemporary to
civilisation is concerned, there is a Gautama Buddha are also mentioned in
considerable ambiguity about it. the jatakas. The geographic details
In the context of the second described in the context of Gautama
urbanisation, however, information about Buddha’s travels are supposed to be very
the geographic location of each reliable. They form a reliable source for
­mahajanapada, its capital and other cities writing the history of ancient India.

Mahajanapadas and their capitals : 2. Kosala : Ancient Kosala encompassed


1. Kasi – Varanasi the regions of Uttar Pradesh in India and
2. Kosal – Shravasti Lumbini in Nepal. Shravasti was its capital
3. Anga – Champa city. King Prasenjit (Pasenadi*) was a
4. Magadha – Girivraja/Rajgriha disciple of Gautama Buddha. Kosala was
5. Vrujji/Vajji – Vaishali destroyed and annexed permanently to
6. Malla/Malava – Kushinara/ Magadha by King Ajatashatru.
Kushinagara * Pasenadi is the Pali version of the name Prasenjit.
7. Chedi – Shuktimati/Sotthivati
8. Vamsha/Vatsa – Kaushambi 3. Anga : The city of Champa was the
9. Kuru – Indraprastha/Indrapattan capital of Anga. It was a centre of the
10. Uttar Panchala – Ahichchhatra, marine trade. It was permanently annexed
Dakshina Panchala – Kampilya to Magadha by King Bimbisara.
11. Matsya – Viratnagar 4. Magadha : Magadha had its first
12. Shoorsena – Mathura capital at the city of Girivraja, also known
13. Ashmak /Assaka – Potali / Potana / as Rajgriha. Girivraja was surrounded by
Podana five hills making it formidable for the
14. Avanti – Ujjayini and Mahishmati enemies. King Bimbisara was a
15. Gandhara – Taxila contemporary of Gautama Buddha. The
16. Kamboja – Rajpura policy of territorial annexation of other
mahajanapada period. Varanasi was its kingdoms was started during the reign of
capital. The kings of Kasi were ambitious. Kind Bimbisara.
According to the jataka stories they aspired Later Pataliputra became the capital of
for the highest position among all Magadha empire.
contemporary kings (Sabbarajunam 5. Vrujji  /  Vajji : This was a
aggaraja). The Buddhist text, ‘Mahavagga’ confederation of eight clans, known as
mentions that the king of Kasi had defeated ‘Maha Aththkula­ ’. It included clans like
Kosala and annexed it to his own kingdom. Videha, Lichchhavi, Vajji, Shakya,
Later, Ajatashatru, the king of Magadha Dnyatruk, etc. The ‘Ekapanna Jataka’
conquered and annexed Kasi mahajanapada mentions that Vaishali, the capital of this
to Magadha. mahajanapada was fortified with three

41
Telegram Channel -: Mpsc Super Batch 104 Compiled by Sanjay Pahade 104
surrounding walls. It had three entrance 10. Panchala : The mahajanapada of
gates and bastions. King Ajatashatru was Panchala was divided into, Uttara (north)
the one who annexed the territory of the Panchala and Dakshina (south) Panchala.
Vrujji/Vajji’s land to Magadha. The river Bhagirathi was the natural
6. Malla : The city of Kushinara or boundary that divided the mahajanapada.
Kushinagara (Kasia) in the Gorakhapur Ahichchhatra, the capital of Uttara Panchala
district was the capital of this mahajanapada. was located near the village of Ramanagar,
Gautama Buddha attained Mahaparinirvana district Bareilley, Uttar Pradesh. Kampilya,
at this city. A copper plate inscription (5th present-day Kampil in the Farukhabad
century C.E.) was found at the ‘Parinirvana district of Uttar Pradesh, was the capital of
Stupa’ in this city. It read, “Parinirvana Dakshina Panchala. The mahajanpadas of
chaitye tamrapatta iti”. In the later vedic Kuru and Panchala often fought for
period Malla mahajanapada was a supremacy. Both the divisions of Panchala
monarchy. Later it turned into a republic. mahajanapada were monarchies in the
There were two more cities in the beginning. At a later date, they turned into
mahajanapada, namely, ‘Pava’ and a sangharajya.
‘Bhaoganagara’. According to the Jaina 11. Matsya : The capital of this
texts, eighteen republics from the regions of mahajanapada was Viratnagar, which was
Mallas, Lichchhavis and Kashi-Kosala, had located at Bairat in the Jaipur district of
formed a confederation to fight Ajatashatru. Rajasthan. Matsya was merged into the
By the 3rd century B.C.E. the Malla empire of Magadha at a later date. Bairat
mahajanapada was merged into the Maurya is one of the places where Ashokan edicts
empire. have been found.
7. Chedi : This mahajanapada had 12. Shoorsena : This mahajanapada
occupied Bundelkhand and the region was located on the banks of the river
around it. The city of Shuktimati or Yamuna. Its capital was the city of Mathura.
Sotthivati was its capital. It is supposed to Greek historians have mentioned the name
have been situated near ‘Banda’ in Uttar of the mahajanapada as ‘Shursenoi’ and
Pradesh. Mathura as ‘Methora’. Later, the
8. Vamsha or Vatsa : Kaushambi was mahajanapada of Shoorsena was merged
the capital (Kosam near Allahabad) of this into the Maurya empire.
mahajanapada. According to the tradition 13. Ashmak/Assak : We have seen
of Purana texts Hastinapur was destroyed earlier that Potali was the capital of
by a flood of Ganga and King Nichakshu, Ashmak mahajanapada and probably it was
a descendant of the Pandavas had to shift a feudatory state of Kashi mahajanapada.
his capital to Kaushambi. The protagonist 14. Avanti : This mahajanapada
of the play ‘Swapnavasavadatta’ written by encompassed the region of Malwa, Nimad
Bhasa is King Udayan. He was the king and its neighbouring regions in Madhya
of the Vatsa mahajanapada and the Pradesh. Avanti mahjanapada was divided
contemporary of Gautama Buddha. into Uttara Avanti and Dakshina Avanti.
9. Kuru : The capital of this Ujjayini (Ujjain) was the capital of Uttar
mahajanapad was located at Indrapat near Avanti, while Mahishmati (Mandhata,
Delhi, Its name was Indraprastha or District Khandwa) was the capital of
Indrapattana. According to the Jataka Dakshina Avanti. King Pradyot was a
literature, the kings of Indraprastha belonged contemporary of Gautama Buddha. In the
to ‘Yudhitthil’ gotra. 4th century B.C.E. Avanti mahajanpada

42
Telegram Channel -: Mpsc Super Batch 105 Compiled by Sanjay Pahade 105
became an integral part of the Maurya entitlement of ‘Saamraj’. The epithet ‘Raja’
empire. always indicates a lower cadre.
15. Gandhara : This mahajanpada had A ‘Raja’ was expected to be a ‘Kshatriya’
spread into Kashmir and Afghanistan. and according to the existing norms a
Taxila was its capital. Pukkusati or Brahmin was expected to refrain from
Pushkasarin was the king of Gandhara, accepting the position of a ‘Raja’. However,
who was a contemporary of King Bimbisara. there are a number of exceptions to this norm
He had established diplomatic relations with as seen in the Vedic literature and Buddhist
King Bimbisara. By the 6th century B.C. the jatakas. The position of Raja was generally
Iranian emperor, Daryush I conquered bequeathed on the son of the ruling king
Gandhara mahajanapada. The Behistun after him. However, at times, a king was
inscription in Iran (516 B.C.E.) mentions elected by people. The first wife of the king
Gandhara as one of the satrapies of the was given the epithet of ‘Rajmahishi’. She
Iranian empire. was the one who was formally crowned
16. Kamboja : This mahajanapada is along with the king. A powerful sovereign
mentioned in the ancient literature along performed ‘Ashvamedha’ sacrifice to
with Gandhara. Rajapura (Rajauri) was its establish his supremacy. The coronation of a
capital. Kamboja was well-known for its king, principally gave him absolute authority
excellent horses and its horsemen warriors over his subjects. He was the one to decide
for their skills of warfare. Kamboja people the amount of taxes to be collected from
had resisted Sinkandara’s advent. The them. He was the ultimate lord of all the
Aspasioi (Ashvayana) was part of the land in his kingdom and so he could donate
Kamboja mahajanapada. The mahajanapada any portion of that land according to his
is mentioned in Ashoka’s edicts as ‘araj’, wish. Nevertheless, his power was not totally
meaning ‘those who do not have a king’. unrestricted. The king made his decisions by
In turn, it means that those who were a seeking advice from his officials such as
republic. ‘Purohita’, ‘Senani’, ‘Amatya’, ‘Gramani’,
6.5 Mahajanapadas - Administrative etc. Besides, there was an assembly of people
System, Guilds of all classes. When it assembled, everybody
present there could participate in the decision
Administrative System : In the 5th
making process. There were instances when
lesson, we have seen various terms in the
people’s assemblies made a king step down
context of the types of ancient states. They
from the throne.
were ‘Rajya’, ‘Svaarajya’, ‘Bhaujya’,
‘Vairajya’, ‘Maharajya’, ‘Saamrajya’ and Guilds (Shrenis) : Along with
‘Parmeshthya’. It is difficult to define these agriculture and animal husbandry, trade and
terms. However, ‘Shatapatha Brahmana’ systematic management and organisation of
and ‘Katyayana Shrautsutra’ explain the the trade are also essential factors for the
term Rajya and Saamrajya in the conext of prosperity of a state. The guilds of the
sacrificial system. According to it, the king merchants and the artisans played a great
who performs ‘Rajasooya’ sacrifice is role in the growth of the mahajanpadas into
designated as ‘Raja’. The kingdom ruled by wealthy states. These guilds had their own
him is designated as ‘Rajya’. When a ‘Raja’ way of organisation and functioning. Among
performs ‘Vajapeya’ sacrifice, he is entitled the important characteristics of the guilds
to the epithet of ‘Saamraj’ and the ‘Rajya’ were, local organisation of various
under his rule is entitled as ‘Saamrajya’. A occupations, transferring the skills to the
‘Raja’ is always desirous of the superior young members in the family with hands-on

43
Telegram Channel -: Mpsc Super Batch 106 Compiled by Sanjay Pahade 106
training, leadership naturally invested in the their funds for the operations involved in
senior and experienced members of the production and trade, as well as for the
guild and other members following them social cause. It was in the way of giving
voluntarily. donations and giving loans with interest at
Guilds had their own, strict rules. modest rates.
Hence, they had a stringent structure. This 6.6 Philosophy and Various Sects
stringent structure is perceived as one of the During the times of mahajanapadas the
main reasons of the rise of the caste system. political and social scenario in ancient India
The paid outsiders who worked in the guilds was witnessing transition and these changes
were known as ‘Karmakara’ and unpaid were bound to reflect in the material and
workers as ‘Dasa’. philosophical notions about life. In this
The guilds had acquired an important regard, the Vedic system revolved around
place in the political, social and financial the duties of a householder, sacrificial
matters by the Maurya period. According to system and material prosperity. At the end
Kautilya, the king’s control on the of the Vedic period questions about abstract
transactions of guilds was very important. aspects of human existence began to gain
Structurally the shrenis had a great potential attention. It included questions about matters
of channelling the central power. The like the meaning of human life, its
Ashokan edicts show that the roads and the significance in the infinite nature of the
transport system was maintained with great universe, the mystery of death and the
care. It had made transport of goods and journey of the soul after death. It resulted
travelling convenient. Obviously, this state in the creation of various philosophical
of affairs was favourable for the growth of texts, known as ‘Upanishadas’. The debates
trade and development of shrenis. that were churned up during this period
The functioning of the guilds was gave rise to various schools of thoughts.
controlled by the state. Detailed records They included schools propounding the
were kept of the production by the guilds, immortality of soul to the schools that
their financial operations and above all the refuted the very existence of the soul. The
traditions of guilds. Every guild had an latter included ‘Charvaka’ or ‘Lokayat’
independent space in the city, allotted to it. school, which openly opposed the social
There were independent traditions with organisation based on the Vedic authority,
respect to the interrelationship between the beliefs and rituals. This school also
guild members and the karmakaras, various propagated that the ‘Truth’ comprises only
stages of production and fixing the prices those things, which are subject to sensory
of produced goods. Each guild had its own experience. The Charvaka/Lokayat school is
rules based on its traditions. The state one of the ancient Indian schools of thought
administration did not interfere in these (darshan) which denies the existence of
aspects of the functioning of the guilds. It ‘God’. It opposes the authority of Vedas
kept the internal independence of the guilds (vedpramanya) and the notions of the
intact. If a decision has to be reached about existence of god and life after death. It also
the matters of a guild, then the king sought discarded all rituals arising out of this kind
advice from a guild representative. of belief systems. It is the only materialistic
school of philosophy ( ‘Jadavadi Darshan’)
The chief of a guild of artisans was that was developed in ancient India.
known as ‘Jyeshthaka’ or ‘Jyetthaka’. The
chief of a guild of merchants was known It may be said that the ancient Indian
as ‘Shreshthi’ or ‘Setthi’. The guilds used philosophical schools took roots in this

44
Telegram Channel -: Mpsc Super Batch 107 Compiled by Sanjay Pahade 107
period. The 6th century B.C.E. is important Siddhartha, the father of Vardhamana
from this point of view. The rising of a Mahavira, was the chief of the ‘Dnyatruka’
class of wandering mendicants known as clan. His mother, Trishaladevi was from the
‘parivrajakas’ or ‘shramanas’, was Lichchhavi clan. Vardhamana Mahavira
characteristic of this period. They renounced was born in 599 B.C.E., in Kundgram near
the householder’s life and wandered with Vaishali. His wife’s name was Yashoda.
their disciples, in search of the ultimate Vardhamana Mahavira left home at the age
truth. Among the various streams of of 30, in search of the truth. At the age of
thoughts, a large number of people were 42 he attained absolute knowledge (Keval
attracted to the teachings of Vardhamana Dnyana). Thereafter, people started
Mahavira and Gautama Buddha. Their addressing him as ‘Kevali’, ‘Jina’ and
teachings showed the way to overcome the ‘Mahavira’. His followers came to be
disparity in the society resulted from the known as ‘Jaina’. Vardhamana Mahavira
varna and caste system. travelled and preached continuously for 30
Both, the Jaina and the Budhhist years after the attainment of absolute
schools of thought are considered as the knowledge.
‘nastik’ schools. Both refuse to accept the Vardhamana Mahavira addressed people
authority of the Vedas and the Vedic rituals. in Ardhamagadhi, the language of common
Both the schools had followers in large people. He preached about good behaviour
numbers from all strata of the society. and life of devotion (devotion to the five
6.7 New Religious Trends vows). He put absolute emphasis on the three
Jainism : The Jaina religion has a principles of ‘Samyak Darshana’, ‘Samyak
tradition going back to very ancient times. Dnyana’ and ‘Samyak Charitra’. These three
According to the tradition of Jain religion, principles are known as the ‘Triratna’ of the
Vardhamana Mahavira was the 24th and the Jaina religion. They are the primary guiding
last ‘tirthankara’ (saviour and the spiritual principles for a person aspiring for the
teacher). Parshvanath was the 23rd attainment of absolute knowledge.
tirthankara. He gave four vows of non- Vardhamana Mahavira preached that all
violence (Ahimsa), truth (Satya), non- beings, animate and inanimate, possess
stealing (Asteya) and non-possesion consciousness, to a varying degree. If hurt,
(Aprigrah) to his followers. Vardhamana each of them experiences pain.
Mahavira added the fifth vow of celibacy The core principle of Vardhamana
(Brahmacharya) to it. These five vows are Mahavira’s philosophy is ‘pluralism’
known as ‘Panchmahavratas’. (Anekantavada) or ‘many sidedness’ of the
truth. According to it the truth has multiple
facets.
Mahavira founded the Jaina Sangha.
Emperor Chandragupata Maurya was a
contemporary of Bhadrabahu, the sixth
principal Acharya of the Jaina Sangha.
According to the Jaina tradition Chandragupta
Maurya had embraced Jaina religion.
Buddhism : Gautama Buddha and
Vardhamana Mahavira were contemporaries.
Gautama Buddha was born in 563 B.C.E., in
Vardhamana Mahavira Lumbini (Rummindei) in Nepal. His father’s

45
Telegram Channel -: Mpsc Super Batch 108 Compiled by Sanjay Pahade 108
name was Shuddhodana. He was a member (1) There is Sorrow (Dukkha)
of the Shakya clan. Gautama Buddha’s everywhere in the world. (2) The root cause
mother’s name was Mayadevi. She was from of sorrow is Greed (Trishna). (3) To
the Koliya clan. Gautama Budhha’s wife conquer this greed is the way to cut the
was Yashodhara. sorrow at its very root (Dukkhanirodha).
Gautama Buddha left home at the age (4) The way to stop sorrow from emerging,
of 29, in search of the truth. This event is is the eightfold path (Ashtangika Marga).
known in the Buddhist tradition as Gautama Buddha explained the eightfold
‘Mahabhinishkramana’. Thereafter, Gautama path by including:
Buddha tried to attain the ultimate knowledge (1) Samyak Drishti (To understand and
through various means, such as seeking accept that nothing happens against the
guidance from various gurus and extreme rules of the nature)
austerities. After realising that nothing of it (2) Samyak Sankalp (Right determination)
was useful, he sat down meditating at Gaya, (3) Samyak Vacha (Right speech)
under a pippal (ficus religiosa) tree, on the
(4) Samyak Karmanta (Right behaviour)
banks of the river Niranjana (Lilajana). Here
(5) Samyak Aajiva (Livelihood by right
he attained enlightenment at the age of 35.
means)
After that he was known as ‘Buddha’,
‘Tathagata’ and also as ‘Shakyamuni’. After (6) Samyak Vyayam (Conscious avoidance
the enlightenment he delivered his first of offensive things)
sermon in the deer-park at Ishipattana, near (7) Samyak Smriti (Watchfulness and
Saranath. This event is known as memory of right things)
‘Dhammachakkapabattan’. For the next 45 (8) Samyak Samadhi (Establishing the
years Gautama Buddha travelled continuously mind firmly in equanimity; a state
to preach dhamma to people. He taught in beyond pleasure and sorrow)
Pali, a language of common people. The eightfold path is also known as
‘Madhyama Pratipada’. Gautama Buddha
founded the ‘Bhikkhu Sangha’. The concept
of ‘Surrendering to Buddha, Dhamma and
Sangha is very important. This concept is
known as ‘Trisaran’. The three vows,
namely, ‘Buddham saranam gachchhami’,
‘Dhammam saranam gachchhami’ and
‘Sangham saranam gachchhami’ are
essential in the Buddhist religion.
The 6th century B.C.E. saw transition in
all the fields of life, in ancient India. The
Gautama Buddha power conflict among the mahajanapadas
In his sermons, Gautama Buddha started in this century. It continued among
addressed the fundamental question of the four mahajanapadas of Kashi, Kosala,
‘sorrow’ in human life. The root cause of Avanti and Magadha. By the 4th century
human sorrow and the way to overcome it B.C.E. the kingdom of Magadha grew
were two main issues addressed by Gautama powerful and the independent existence of
Buddha. the other mahajanapadas came to an end.
We will study the history of the rise of
Gautama Buddha preached four
Magadha in the eighth lesson.
‘Aryasatyas’ (four great truths).

46
Telegram Channel -: Mpsc Super Batch 109 Compiled by Sanjay Pahade 109
You would like to know : An Ashokan pillar inscription was discovered at the site
of Rummindei in Nepal. The inscription announces, “The king ‘Devanampiya Piyadasi’
visited this place after his coronation and he prayed here. He declares that Buddha
Shakyamuni was born here. To commemorate the fact he erected a stone pillar at this
place. He did this with the intention of making this place venerable in people’s mind. He
exempted the village of Lumbini from paying the taxes in order to enhance its prosperity.”

Exercise

Q.1 (A) Choose the correct alternative and Q.3 Complete the concept map given below.
write the complete sentences.
Jaina Religion
(1) Ashmaka is the name in
language. Panchmahavratas
(a) Pali (b) Sanskrit
(c) Ardhamagadhi (d) Prakrit
Parshwanath Vardhamana Mahavir
(2) The capital of Kashi mahajanapada
was .
(a) Gorakhpur (b) Chandanagar
(c) Rajagriha (d) Varanasi
(3) Gautama Buddha was born in  . Q.4 Explain the statements with reasons.
(a) Kushinagara (b) Sarnath (1) Rise of mahajanapadas came into
(c) Lumbini (d) Pataliputra being.
(4) The river was the natural (2) The process of Second urbanisation
boundary between Uttara Panchala began in ancient India.
and Dakshina Panchala. (3) Vardhamana Mahavira and Gautama
(a) Yamuna (b) Bhagirathi Buddha attracted a large number of
(c) Ganga (d) Niranjana followers.
(B) Find the incorrect pair from set B Q.5 Explain the following cocepts.
and write the correct ones. (1) Nastik Darshan
Set ‘A’ Set ‘B’ (2) The eightfold path preached by
(1) Kosala Shravasti Gautama Buddha
(2) Anga Champa
(3) Matsya Mathura Q.6 Describe the administrative system of
(4) Gandhara Taxila the mahajanapadas with the help of
following points.
Q.2 Choose the correct reason and complete (a) Terms showing types of states
the sentence. (b) King’s installation
Gautama Buddha travelled continuously (c) Authority of the king
for 45 years.... (d) Decision-making
(a) in the search of a Guru
(b) to practice austerities Activity
(c) to preach dhamma (1) Collect and compile the information
(d) to attain enlightenment about Jain Tirthankaras.
(2) Collect information about Jataka
stories. Select any of the Jataka
stories and present it as stage play.


47
Telegram Channel -: Mpsc Super Batch 110 Compiled by Sanjay Pahade 110
7. India and Iran (Persia)

same name, ‘Pars’. Greeks knew the city by


7.1 Indian Subcontinent and Iran
the name, ‘Persepolis’. The Iranian empire
7.2 Conflict between the Iranian
was known as ‘Persian empire’ because it
(Achaemenid) Empire and Greece
originated in the Pars region.
7.3 Iranian (Achaemenid) Empire
and India Emperor Cyrus II had begun to build a
7.4 Political and Cultural Impact of new capital at ‘Pasargade’. However, it did
the Iranian Contact not get completed in his lifetime. His son,
7.5 Taxila Cambyses II succeeded him. He moved back
7.6 Advent of Alexander to Susa and re-established his capital there.
Cambyses II had conquered Egypt.
Daryush I, who succeeded Cambyses II
7.1 Indian Subcontinent and Iran reinforced the fortification walls of Susa. He
Excavated evidence has sufficiently also built a massive palace and a hypostyle
proved that the trade and cultural relations (many columned) hall known as ‘Apadana’.
between the Indian subcontinent and Iran He also built the city of ‘Persepolis’ in the
have continued since the Harappan times. style similar to Susa. He built a palace and
The empire of Elam, contemporary to the apadana in Persepolis also.
Harappan civilisation, existed in the southwest
regions of Iran. It was culturally closer to
Mesopotamia. Its capital was at ‘Susa’.
Hence, it was also known as ‘Susiana’. Many
dynasties ruled Iran at later dates but all of Reconstructed
them maintained their capital at ‘Susa’. The picture of
archaeological evidence found at Susa has ‘Apadana’ at
Susa
confirmed the trade and cultural relations
between Iran and the Harappan civilisation.

Visit the following website to see the map of the Iranian


empire:https://upload.wikimedia.org/wikipedia/commons/3/3a/
Persian_Empire%2C_490_BC.pnghttps://commons.wikimedia.
org/wiki/File:Persian_Empire,_490_BC.png

The Iranian empire was established by


Cyrus II (also known as Cyrus the Great) of
the Achaemenid dynasty. He was from the
tribe of ‘Pars’. This tribe dwelt in the Geographically Iran is the region, which
mountainous region in the northwest of Iran forms a link between the east and west of
and adjacent to Afghanistan. This region was Asia. Naturally Iran held an important
known by the tribe’s name, ‘Pars’. Even the position in the trade and also in the cultural
main city of the region was known by the exchange between East Asia and West Asia.

48
Telegram Channel -: Mpsc Super Batch 111 Compiled by Sanjay Pahade 111
the kingdoms in West Asia and then the city
By the Way : ‘Ariana’ is another states in Greece. Then also they continued to
name of Iran. Greek historians mention have intermittent battles. The rising of the
it as ‘Ariane’. ‘Ariana’ is the Latin
Ionian Greeks against the Achaemenid rule
variation of the Greek name ‘Ariane’.
is supposed to be significant.
The trade that was conducted from Asia Cyrus II conquered Lydia, a Greek state
and the Persian Gulf to Mesopotamia and to the north of Mediterranean Sea, around
Egypt expanded to Greece and Rome. The the mid of the 6th century B.C.E. As a result,
ancient routes used by caravans of merchants the Ionian Greek city states under the Lydian
were provided with safety and security by rule, inevitably became part of the Achaemenid
the Persian emperors. They rebuilt some of empire. The Greeks in Ionia had migrated to
the routes. Cyrus II and Daryush I developed Anatolia, that is, the Asian part of the
the route beginning from Susa and reaching Turkey. It is also known as ‘Asia Minor’.
the Mediterranean Sea. This route is known During the reign of Daryush I all Ionian
as the ‘Royal Road’. Some of the branches city states got together and rebelled against
of the Royal Road linked the Indian the Achaemenid rule. The rebellion was
subcontinent and Egypt. Alexander started subdued completely after a struggle of five
from Macedonia on his expedition to India. years by the Achaemenid army.
He used the same route to reach India. The effects of this unsuccessful rebellion
are important for understanding the
By the way : (1) To see, how the interrelationship between Greece and Persia
name Alexander was changed to in the ensuing period. The Ionian Greeks had
‘Sikander’ in Indian languages, is a received help from the states of ‘Athens’ and
matter of linguistic analysis. Alexander ‘Eretria’ in Greece. Hence, Daryush I attacked
is a common version of the Greek Athens. At this time the battle between
name ‘Alexandros’. The meaning of Daryush I and Athens was fought on the
‘Alexin’ is ‘to protect’. ‘Andros’ means grounds of Marathon near Athens. Therefore,
‘man’. Thus, the name ‘Alexander’ it is known as the ‘Battle of Marathon’.
means ‘the protector’. ‘Iskander’, Daryush I was defeated in this battle.
‘Skander’ are the Persian variations of
Xerexes, the successor of Daryush I,
Alexander. Iskadnser or Skander turned
tried once again to invade Greece. However,
into ‘Sikandar’.
he was also defeated. Thereafter the conflict
(2) The inscriptions of the emperor,
between the Achaemenids and the Greeks
Daryush I at Susa, Pasargade,
continued over a prolonged time. Ultimately,
Persepolis and Behistun have been
Athens and the confederacy of other Greek
enlisted as the ‘World Cultural
Heritage’. city states declared a war for independence
against the Achaemenid rule in Greece.
Neither of the two had a conclusive victory.
7.2 Conflict between the Iranian
In 449 B.C.E. a treaty was signed between
(Achaemenid) Empire and Greece
the Greek confederacy and the Achaemenids.
Greek historians have recorded the
As the effect of this prolonged conflict
frequent battles between the Achaemenid
the Achaemenid rule grew weak. This was
empire and the city states in Greece. Among
bound to reflect in the political and financial
them the records of Herodotus are important.
spheres. An impact of this conflict was to
The Achaemenid emperors first conquered

49
Telegram Channel -: Mpsc Super Batch 112 Compiled by Sanjay Pahade 112
manifest in the way of the invasion of Persia
by Alexander III, the king of Macedonia, By the way : The Greek historian
who was later regarded as the ‘Conqueror of Herodotus was born in 484 B.C.E., in
the World’. Halicarnassus, a Greek city state. He
began to write with the intention of
Arrian was a Greek historian of the
tracing the history and causes of the
1 century C.E. He wrote a book called
st
battles between Greek city states and
‘Anabasis* of Alexander’. He has referred to
Achaemenid empire during 500-449
the correspondence between Daryush III, the B.C.E. The book he wrote was named
last Achaemenid emperor and Alexander. as ‘Historia’ (The Histories). He, at
Daryush had written to Alexander about the the outset of the book says, “Herodotus
release of his mother, wife and children. of Halicarnassus here displays his
Alexander responded by reminding Daryush inquiry, so that human achievements
of the sorrow inflicted on the Greeks by the may not become forgotten in time, and
earlier invasions of the Achaemenid emperors. great and marvellous deeds – some
Alexander wrote that he had arrived in displayed by Greeks, some by
Russia by crossing the ocean, with the barbarians – may not be without their
intention of punishing the aggressive Persians. glory.”
Next, he accused Daryush of provocating the Herodotus was the first historian
Greeks against him and he reminded Daryush who collected information seeking
that he was a defeated king. So Daryush was answers to a particular question, and
further snubbed that he should not behave as then to put it in a chronological order.
an equal to Alexander. While doing this he did not bring in
notions like god, human destiny, etc.
*
‘ Anabasis* means’ to spur into action. Later putting historical events in
chronological sequence became an
This correspondence is indicative of essential theme of history writing. This
two things: theme proved to be fundamental in the
(1) The Greeks felt that their self-esteem development of history writing as an
was challenged. independent discipline. Therefore,
Herodotus is said to be the father of
(2) The Greeks now felt confident about
historiography.
their strength to conquer the Persians.
It may be said that these two factors
kept by the Greek historians that Cyrus II
inspired Alexander’s expedition.
had conquered the region of the Kabul basin
7.3 Iranian (Achaemenid) Empire and (Gandhara). It was part of the Achaemenid
India empire. The river Vitasta (Jhelum) had
The Achaemenid emperors established formed its eastern boundary. Alexander
the Persian empire by conquering smaller crushed the Achaemenid empire and the
kingdoms in the northeast region of the region of Punjab which was the part of
Indian subcontinent and expanded their Achaemenid empire.
boundary up to Punjab. It was the same The revenue received by the Achaemenid
period when the Magadha empire was on the empire from its provinces (satrapies) in the
rise in India. Indian subcontinent was more than any other
It becomes apparent from the inscriptions provinces. According to Herodotus, this
of the Achaemenid emperors and the records revenue amounted to 360 talents* of gold

50
Telegram Channel -: Mpsc Super Batch 113 Compiled by Sanjay Pahade 113
dust**. Accepting the possibility of and proceeded further by sailing around the
exaggeration in this description, it clearly coast of the Arabian Sea and then entering
indicates that the revenue collected from the the Red Sea. He ended his journey at ‘Suez’,
Indian states formed a major share of the situated in the delta region of the Nile. It
Achaemenid treasury. took two and half years for him to complete
this journey.
*Talent used to be a unit of weight and
The logbook of the journey of Scylax is
according to the standard, set in various ancient
countries it varies from 20 to 40 kilograms per known as ‘Periplus of Scylax’. The original
talent. periplus is lost but we get to know about its
** Gold dust (placer gold) is collected from contents from the writings of the Greek
the sands on the river banks. historians. This periplus was the first source
of information of the Indian subcontinent to
7.4 Political and Cultural Impact of the western world, especially to the Greeks.
the Iranian Contact Very soon after the completion of the
The Persian supremacy prevailed in the expedition by Scylax, Daryush I conquered
Indian subcontinent for at least two centuries. the region in the lower reach of the Sindhu.
However, it had ended much before the Scylax had reported an already existing
advent of Alexander. Thinking of the impact canal that linked the Nile and the Red Sea.
left by the Persian contact in the period of It was dug by an Egyptian Pharoh. Daryush
two centuries, one thing can be noticed I revived that canal by digging it again. It
prominently. It is about the administrative opened a new water way for the transport
system set by the Persian rulers. Every between Persia and the Indian subcontinent.
conquered region was annexed to their
This newly available route facilitated the
territory as a satrapy and a satrap (governor)
trade between Achaemenid provinces in the
was appointed as its chief administrator. This
north-western region of the Indian subcontinent
method was followed by Alexander and later
and the regions of Sindh-Punjab. The ivory
by the Scythian (Saka) and Kushana rulers
and the teak wood coming from India had
as well.
great demand in the Persian markets. The
The Aramaic script came into use in inscription of Daryush I at Susa records the
the north-eastern regions of the Indian use of Indian ivory and teakwood for building
subcontinent, during the times of Achaemenid the palace.
rule. Kharoshthi, an ancient Indian script
Herodotus has described the Indian
evolved from this script. The Ashokan edicts
soldiers in the Achaemenid army. According
in this region are inscribed in the Aramaic
to it, Xerexes’ army had soldiers from
script. The method of installing royal edicts
Gandhara, Sindh and Punjab, when he
at vantage points seems to be borrowed from
invaded Greece in 480 B.C.E. The soldiers
the examples of Achaemenid inscriptions.
from Gandhara were equipped with bows
Herodotus tells us that Daryush I had made of cane and spears with short shafts.
sent many explorers to gather information They also had horses and chariots drawn by
about the course of the river Sindhu and the horses. The Indian soldiers from Sindh and
Arabian Sea. Among them was ‘Scylax of Punjab used cotton clothes and used bows
Karyanda’, a Greek sailor from Ionia, who and arrows made of cane. They were expert
was well- known. He was the first Ionian archers. When Xerexes retreated from Greece,
Greek who stepped on the Indian land. it was decided to keep a regiment posted
Scylax began his exploration from the Sindhu

51
Telegram Channel -: Mpsc Super Batch 114 Compiled by Sanjay Pahade 114
there. That regiment had Indian soldiers. Susa and Persepolis. For that purpose,
In the times before Cyrus II, the founder emperor Cyrus got Greek architects and
of Achaemenid empire, there was no coinage sculptors from the Greek settlements in Asia
in use in Persia. The barter system prevailed minor under his rule.
to a large extent. Sometimes silver ingots of
particular weight were used for exchanging
commodities. Cyrus II had conquered the city
state of Lydia, where use of coins was
already in practice. Those coins were known
as ‘stater’. Cyrus issued coins similar to
Lydian coins.
Daryush I issued coins with his own
portrait on it.
The Gold coins
issued by him
were known as Columns at Persepolis
‘Darik’ and
silver coins Persepolis was razed completely by
Persian ‘Siglos’
were known as Alexander after conquering Persia. It was the
‘Siglos’. A portrait of Daryush I, holding a end of the Persian Empire. The architects,
bow and arrow can be seen on these coins. sculptors and other artisans lost the royal
One ‘Darik’ was equivalent to 12 ‘Sigloi’* patronage. They had to move out in search
in value. of work. They migrated to India. They
received royal patronage in the court of
emperor Ashoka. Their style is reflected in
* ‘‘Sigloi’ is the plural
*
’ of ‘Siglos’. the stone pillars erected by empror Ashoka.
Thus, the sculptural art in ancient India has
Along with the Achaemenid coins, coins its roots in the Persian; and acordingly in the
from Greek satrapies also remained in Greek sculptural style.
circulation in Persia. Some satraps issued 7.5 Taxila
their own coins imitating Greek coins. These Mahabharata mentions King Takshaka
coins were cast in moulds. The obverse side as the king of ‘Naga’ people. Taxila was the
of a coin usually carried the portrait of a capital of King Takshaka. The archaeological
king and the reverse side had a symbol evidence confirm that Taxila has a hoary
punched on it. past. It was the capital of Gandhara
It is possible that the ancient Indian mahajanapada.
punch marked coins were influenced by this The archaeological remains of this city
technique of punching. are strewn over a large area that comprises
In the times that preceded emperor 18 archaeological sites. They are enlisted as
Cyrus’ reign, the practice of building huge the World Cultural Heritage. It is situated on
public monuments did not exist in Persia. the Grand Trunk Road, 30 kilometres away
Hence, there was no existing Persian from Islamabad, the capital of Pakistan.
architectural style to follow while creating Evidence of prehistoric people using
the magnificent palaces and sculptures in microliths have been found at Taxila and in

52
Telegram Channel -: Mpsc Super Batch 115 Compiled by Sanjay Pahade 115
from various regions of the Indian subcontinent
By the way : The inscription of were attracted to Taxila because of the fame
Daryush I mention the Greeks as ‘Yauna’.
of the acharyas. Thus, Taxila had become a
The roots of this word is in the Greek
great centre of education. Thereby it had
word, ‘Ionia’ The terms such as ‘Yona’
assumed the magnitude of a natural university.
(Prakrit) and ‘Yavana’ (Sanskrit)
However, the king and his administration at
mentioned in Indian literature originally
mean ‘the resident of Ionia’. These terms Taxila did not interfere in the decisions of
occur in Mahabharata, Ramayana, Puran an individual acharya as to the nature of his
texts and other literature. method of teaching, the subjects to be taught
by him or the nature of the syllabi. The
Let us see one more example of
changes happening in the meaning and duration of a course was dependent on
form of the word through the cultural individual student’s intellectual capacity to
contacts and borrowing of words from learn. There was no formal system of
other languages. Ancient Persian language examinations. Rather success in examination
was the official language of the was not perceived as the test of real
Achaemenid empire. It is used in the knowledge. It was the acharya who decided
inscriptions of the Achaemenid emperors. the level of maturity of his students.
These inscriptions mention the people in Among the subjects taught at Taxila
the Indian subcontinent as ‘Hidush’ and included, Rigveda, Yajurveda, Samaveda,
‘Hridush’. In the Ionian Greek language ancillary branches of the Vedas (Vedangas),
the consonant ‘h’ is not pronounced when Ancient Traditions and Ethics, Philosophy,
it occurs at the beginning of a word. Mathematics, Music, Medicine, Puran Texts,
Hence, Scylax the Ionian Greek explorer,
History, Weaponry, Poetry, etc.
perhaps recorded it as ‘Indos’ and its
plural as ‘Indoi’. Herodotus used these Acharya Chanakya is believed to have
words as synonymous to Hidush and taken Chandragupta Maurya to Taxila for
Hridush. The roots of the word ‘India’ education when he was young. Chanakya
is in this Greek usage. himself hailed from Taxila. After becoming
the emperor of Magadha, Chadragupta
established a regional capital at Taxila.
some caves in its vicinity. The neolithic
During the reign of Ashoka Maurya
village at Taxila was settled around 3500
Taxila became an important centre of learning
B.C.E. Its remains have been found at a site
of Buddhist tradition. Taxila did not lose its
called ‘Sarai-Khola’.
importance even when the land suffered from
Taxila was ruled by King Ambhi at the the invasions of Greeks, Shakas and
time of Alexander’s advent. He welcomed Kushanas. However, in the 5th century C.E.
Alexander and presented gifts to him. The the glory of Taxila gradually waned under
Greek historians accompanying Alexander the pressure of Huna invasions.
described Taxila as a “city full of hustle
7.6 Advent of Alexander
bustle of people, prosperous and the one that
was under a well-established administration.” Alexander invaded the Indian
subcontinent and conquered some kingdoms
Some information about Taxila can be
from Afghanistan to Sindh-Punjab. He,
gathered from the Buddhist literature and
however, could not establish his rule there
from the writings of the Greek historians.
for a long time. The information of his
There were several learned individuals
advent primarily comes from the Greek
(Acharyas) staying in Taxila. Many students

53
Telegram Channel -: Mpsc Super Batch 116 Compiled by Sanjay Pahade 116
historians like Arrian, Curtis, Diodorus, greatly influenced by his valour. The Greeks
Plutarch and Justin. were unfamiliar to the use of elephants in
At the time of Alexander’s advent the warfare. To penetrate the file of elephants
Achaemenid rule in Sindh-Punjab and proved to be a great challenge for them. The
Afghanistan had weakened and a number of Greek historians have unanimously praised
small kingdoms had come into existence. The the skill of Porus in warfare and also his
political strife stopped them from uniting to courage.
resist Alexander’s invasion. After the confrontation with Porus,
Alexander became the king of Macedonia Alexander reached Nysa, a Greek settlement.
in 334 B.C.E. He defeated emperor Daryush People of Nysa resisted Alexander in the
III in 331 B.C.E. Thereafter he marched up beginning but later they welcomed him.
to ‘Shistan’ province of Iran and from there Thereafter, Alexander proceeded in the
he turned toward Kabul. After conquering the direction of Chenab and Ravi. He conquered
region around Kabul, he reached the foothills the kingdoms on the way and reached the
of Hindukush mountains. Then he camped at banks of Beas. By that time Alexander’s
a place called Nicaea, situated on Uttarapatha. soldiers had lost spirit and they refused to
Ambhi welcomed Alexander and sought march forward. Alexander was forced to
friendship with him. Another king whose return.
name was Sisikottas (Shashigupta) surrendered Before setting on the return journey
to Alexander. However, not all Indian kings Alexander handed over the regions in Punjab
surrendered to Alexander so easily. Many to Porus and those in Sindh to Ambhi. He
fought with Alexander with great valour. handed over the kingdom of Kashmir to
Alexander conquered almost all kingdoms in Abhisara, the king of Varanavati. On his
Kabul, Sindh and Punjab. way back he defeated Shibis and Mallas, etc.
On the banks of Jhelum Alexander and In rest of the regions he appointed Greek
King Porus confronted each other and there satraps. In 325 B.C.E. Alexander died at
was a fierce battle between them. Porus was Babylon, while he was on his way back to
defeated but Alexander and his army was Greece.

For Additional Information : The trees on both sides of the road, sarais for
highway linking the regions from Bihar to overnight stay, etc. He erected pillar edicts
Afghanistan is known as ‘Grand Trunk at vantange points on this highway.
Road’ in modern days. It was in use as At a later period Shershah Sur and
a trade route even during the times of Emperor Akbar rebuilt this highway. In the
Gautama Buddha. It was known as British period a new metalled road running
‘Uttarapatha’ in those days. Its starting parallel to Grand Trunk Road was built
point was at the port city of ‘Tamralipti’ from Kolkata to Peshawar. This road is
in Bengal; known as Tamluk today. It still in use.
ended at Taxila. Emperor Chandragupta In modern times the highway starts
Maurya established this highway in a well from Bangla Desh and from Hawrah it
planned way. It was about 3200 kilometres proceeds to West Bengal, thereafter it goes
in length. to Delhi and to Amritsar, from there to
Emperor Ashoka provided many Lahore and Peshawar and in the end to
essential amenities for the merchants and Kabul.
travellers on this highway, such as planting

54
Telegram Channel -: Mpsc Super Batch 117 Compiled by Sanjay Pahade 117
Alexander’s invasion of India did not his vast empire from Bihar to Afghanistan.
have far reaching impact on the political It was the beginning of a new era in the
scenario of India. In a very short period after Indian history.
his death Chandragupta Maurya established

Exercise

Q.1 (A) Choose the correct alternative and (C) Write the names -
write the complete sentences. (1) Last emperor of Achaemenid empire-
(1) The Greek historian is said (2) Name of the coins issued by Daryush
to be the father of modern I -
historiography. Q.2 Explain the statements with reasons.
(a) Herodotus (b) Alexander (1) Alexander invaded Persia.
(c) Scylax (d) Daryush (2) Alexanders’ invasion of India did not
(2) Taxila was ruled by King have for reaching impact on the
at the time of Alexander’s advent. political scenario of India.
(a) Chandragupta (b) Ambhi Q.3 State your opinion.
(c) Porus (d) Shashigupta (1) Herodotus is said to be the father of
(B) Find the incorrect pair from set B modern history writing.
and write the correct ones. (2) Ancient Taxila was the centre of
Set ‘A’ Set ‘B’ knowledge and education.
(1) Persopolis City built by
Q.4 Answer the following questions in detail.
Daryush I
(1) Describe the political and cultural
(2) Halicarnassus Herodotus was
effects of Achemenid rule on India.
born here
(2) Describe Alexander’s invasion of
(3) Taxila Centre of knowledge
India.
and education
(4) Nysa Persian settlement Activity
Collect additional information about the
palace at Persepolis and Apadana at Susa
with the help of internet.



55
Telegram Channel -: Mpsc Super Batch 118 Compiled by Sanjay Pahade 118
8. India during Mauryan period

8.1 Rise of Magadha Empire You would like to know : The


8.2 Nanda and Mauryan Empire concept of ‘Chakravarti’ occurs in the
8.3 Emperor Ashoka literature on ancient Indian polity. The
8.4 Administrative system, trade, sovereign ruler whose chariot could roll
literature, art and architecture, in all four directions without any
social life obstruction, was known as Chakravarti.
It was expected that his rule should be
In lesson six, we studied the history of ethical and for the welfare of the State.
During his tenure the wheel of ‘Ruta’
the sixteen mahajanapadas that existed in
remains intact. His rule is on an extended
India. Among these mahajanapadas, the
territory. Chandragupta Maurya was the
power of Magadha became strong during the
first Chakravarti Emperor.
struggle of the four mahajanapadas namely,
Kasi, Kosala, Avanti and Magadha and the
dynasty ruled over Magadha.
empire of Magadha came to a rise.
The Haryanka dynasty is mentioned in
Magadha empire was the first Indian
the Mahabharata. Bimbisara was the first
empire in India. The kingdom of Magadha
well-known king. His father Mahapadma
acquired the form of an Empire. The
built the fort at Girivraja and established the
administrative system of this empire was well
first capital of Magadha. After ascending the
established and its work was carried out
throne, Bimbisara started building the
through various departments. Several factors
foundation of Magadha Empire. He attacked
are essential for the emergence of an empire.
the neighbouring kingdom of Anga and
For example, dominance over an expanded
conquered it. This victory increased the power
region, an administrative system having the
of Magadha. Bimbisara established
revenue collected in the form of taxes, king
matrimonial relations with many royal houses
as the sovereign ruler of the state and army
such as Kosala, Lichchhavi, Videh, Madra to
actively enforcing king’s authority.
support his expansionist policy. At the foot
The king’s position was strengthened of the fort of Girivraja, he established the
through ritual performances. The power and new capital of Rajgriha.
related rights are concentrated in the hands
After killing Bimbisara, his son
of the ruling family.
Ajatashatru became the king. He adopted the
8.1 Rise of Magadha Empire expansionist policy of his father. He expanded
Magadha was an important kingdom the Magadha kingdom up to the foot of
among the mahajanapadas that existed in Vindhya ranges. Ajatashatru built a small
ancient India. Fertile and rich land, perennial fort on the bank of Ganga at Pataligrama.
rivers, excellent facilities of navigation, It became a centre of trade of local
availability of commercial markets etc. were commodities. In the later period, Pataligrama
the reasons for the rise of Magadha as came to be known as Pataliputra. It became
empire. the capital of the Mauryan Empire.
In the 6th century BCE, the Haryanka Later the people deposed the last

56
Telegram Channel -: Mpsc Super Batch 119 Compiled by Sanjay Pahade 119
Mauryan King Nagdasak and elected his the Nanda dynasty. During his period, the
minister Shishunaga as the king. The state treasury was enormously rich. His
Shishunaga dynasty ruled between 430 B.C.E. military was also very huge. It consisted of
to 364 B.C.E. and then Mahapadmananda 2,00,000 infantry, 60,000 cavalry, 6000
usurped the throne and established the Nanda elephants and 2000 chariots.
dynasty. During the period of Nandas, another
factor that helped in strengthening the central
By the way : Pataliputra was an power was the importance given to enhance
important city in ancient India. The city the revenue collection. Along with the
of Pataliputra was the capital of ruling expansion of the empire came the economic
dynasties like Shishunaga, Nanda, Maurya, prosperity of the State. The state treasury
Gupta and Pala. ‘Patali’ is a species of was always full. The Nandas built canals
rice grown in this region. Hence, the city and made arrangement of irrigation facilities.
might have acquired the name. In his These facilities led to the development of
book Indica, Megasthenes has mentioned agriculture and trade. In 321 B.C.E.
the city of Pataliputra as Palibothara. Chandragupta Maurya attacked Pataliputra
and brought an end to the Nanda rule.
Political stability as well as other factors Mauryan Empire: Mauryan Empire was
were responsible in the strengthening of the first well organised and strongly controlled
Magadha as an empire. Magadha had control empire in the history of India. Religious texts
over all the prominent regions in the Ganga and other literature, inscriptions, coins,
Valley. By conquering the kingdom of Anga, sculptures etc. are the sources that help us
the regions of the east coast came under their to clearly understand the political, social,
control and Magadha succeeded in increasing economic and religious conditions of this
the trade with the distant regions. Magadha period.
was blessed with natural resources. Its land The Mauryan empire had spread on a
was fertile, especially for rice cultivation. large region of the Indian sub continent and
There was rise in revenue due to the was controlled by a central power. The
expansion of the kingdom. The abundance of Mauryan rule brought about consolidation of
sources for timber, ivory, iron and copper the political system.
gave a momentum to the local industries.
Chandragupta Maurya defeated the
The political ambitions of the rulers, combined
Nandas and established the Mauryan dynasty.
with wealth and prospering trade were the
In the text of Mahavamsa, Chandragupta is
reasons for the development of Magadha rule
referred as ‘the emperor of Jambudvipa i.e.
and establishment of a huge empire.
emperor of Bharatvarsh’. The Mauryas
8.2 Nanda and Mauryan Empire created a huge empire by conquering the big
Nanda Dynasty: The Nanda dynasty is and small states in northwest, north and
mentioned in the Puranas. The Magdhan southern regions. The war between
empire established by Ajatashatru was further Chandragupta and the Greek king Seleucus
expanded by Mahapadmananda. According was very important. As a consequence of
to some scholars, the Nanda rule had spread that war, the boundaries of Mauryan Empire
upto Nanded in the South, whereas some extended to the Hindukush in the northwest.
scholars think that it was extended upto The Mauryan Empire extended from
Mysore. On this basis it can be said that Hindukush to the Bay of Bengal in the east
Mahapadmananda was the first great emperor and Gujarat in the west, as well as Himalayas
of India. Dhanananda was the last ruler of in the north to the Krishna river in the south.

57
Telegram Channel -: Mpsc Super Batch 120 Compiled by Sanjay Pahade 120
Chandragupta Maurya died around 298 himself coronated in 268 B.C.E. He called
B.C.E. His son Bindusara became the king. himself as ‘Devanampiyo Piyadasi’ (Beloved
During his tenure he sent his son Ashoka to of the Gods) in many of his rock edicts and
subdue the revolt in Taxila. During the period pillar inscriptions.
of Bindusara the empire established by During the early period of his rule,
Chandragupta Maurya remained intact. Ashoka adopted his ancestors’ policy of
Bindusara died in 273 B.C.E. Digvijaya and expansion of kingdom. He
8.3 Emperor Ashoka attacked Kalinga and conquered it. Ashoka’s
Ashoka became the emperor of Mauryan victory at Kalinga proved to be a turning
empire after the death of Bindusara. He got point in history as well as in the life of

58
Telegram Channel -: Mpsc Super Batch 121 Compiled by Sanjay Pahade 121
For Additional Information : The You would like to know : During
rock edicts and Pillar inscriptions of the period of 321 B.C.E. to 181 B.C.E.,
Ashoka: the expansion of Mauryan empire took
The rock edicts and Pillar inscriptions place in Maharashtra. The Mauryas must
of Ashoka are found at many places in have entered Sopara from Gujarat. Sopara
the regions of Afghanistan, Nepal and is a well-known port in north konkan.
India. In 1750 C.E. Padre Tieffenthaler Sopara and Chaul (Champavati) were
rediscovered Meerut pillar inscription of prosperous trade centers and learning
Ashoka. Most of Ashoka’s inscriptions centers of Buddhism. Bhagvanlal Indraji
are inscribed in Brahmi script. In 1837 excavated the Stupa at Sopara which is
C.E. James Princep deciphered the known as ‘Baruda Rajacha kot’. In this
Brahmi script on the inscriptions. The excavation the fragments of rock edict of
inscriptions of Ashoka are very important Ashoka were found. It is the 8th rock
sources of the history of his rule. The edict among the 14 edicts of Ashoka.
boundaries of Maurya Empire can be Some of its remains are kept in the
ensured on the basis of these inscriptions. museum of Asiatic Society in Mumbai.
The description of Ashoka’s efforts for The 9th rock edict was found in the
the spread of Buddhism can be obtained Bhuigaon village. Its remains are kept in
from these inscriptions. Even though the Chhatrapati Shivaji Maharaj Vastu
Ashoka embraced Buddhism but he never Sangrahalaya in Mumbai.
tried to forcibly impose it on his subjects.
The written evidence of the spread of
Buddhism and Indian culture in distant
countries through the Buddhist monks
sent by Ashoka occurs in these
inscriptions. It included Emperor Ashoka’s
son Mahendra (Mahinda) and daughter
Sanghamitra (Sanghamitta). Stupa of Nalasopara

drinks, guiding principles regarding code of


Ashoka. There was a great human loss conduct etc. Ashoka’s policy of ‘Dhamma’
in this war. Ashoka became victorious. The meant virtuous conduct based on moral
destruction caused due to the Kalinga war values. He sent monks to various countries
brought about a great change in the mind of for the spread of Buddhism.
Ashoka. His mind got diverted towards the The first Buddhist council was held at
non- violent, peaceful Buddhist religion. The Rajgriha after the mahaparnibban of Gautama
desire of digvijaya was replaced by Buddha. King Kalashoka organised the
Dharmavijay (Dhammavijay). His career second council at Vaishali. Ashoka organized
proved to be important because of the the third Buddhist council at the capital
religious ideals propagated by him and the Pataliputra.
creation of a mechanism to until it in people’s
The kings who succeeded Ashoka were
life.
not efficient. This led to the decline of the
For the implementation of Mauryan empire. Brihadratha was the last
dhammavijay, Ashoka introduced some new king of the Mauryan dynasty. He was killed
policies, for example, appointment of by his General Pushyamitra who then came
‘Dharmamahamatras’, ban on intoxicating to power.

59
Telegram Channel -: Mpsc Super Batch 122 Compiled by Sanjay Pahade 122
Important information : In the rock Antiochus II Theos of Syria (260 B.C.E.
edicts of Ashoka there are references about -246 B.C.E.), Ptolemy II Philadelphus of
the contemporary kings from the western Egypt (285 B.C.E. - 247 B.C.E.), Antigonus
world. The identification of Greek king Gonatus of Macedonia (276 B.C.E. -239
Antiouchus and the four greek kings beyond B.C.E.), Magas of Cyrene and Alexander
the kingdom of Antiouchus namely, of Epirus. This paragraph is an important
Turmaya, Amtekina, Maga and link in the process of connecting the
Alikasundara is done through a paragraph chronological sequence of ancient Indian
from one of the inscriptions. That is history.

8.4 Administrative system, trade, the head to the lowermost office of the
literature, art and architecture, social life Gramini. At the local level the people were
Administrative system given autonomous rights to a great extent in
The nature of administrative system the internal matters. In this way the
during the Mauryan period is clearly administration of Mauryan kingdom was
understood with the help of Kautilya’s decentralised. The most important feature of
Arthashastra, Megasthenes’ Indica and the Mauryan administration was the effort made
inscriptions of Ashoka. Chandragupta Maurya for the overall material and moral development
was a valiant king and an efficient of the people. The Mauryan administration is
administrator. He set up a defined mechanism appreciated due to various factors such as
of state administration, which suited the keeping the civil and military areas separate
economic conditions and the needs of the from each other, creating an independent
Mauryan empire. The Mauryan emperors judicial system, implementing a specific
created a unified empire. At the same time, salary system to the officers, control over the
they devised a decentralised administrative trade in the kingdom etc.
system for efficient functioning of the vast Trade
empire. In the administrative system of the There was an increase in revenue of the
empire, there was a Mantri Parishad (council Maurya empire due to well organised taxation
of ministers) made up of wise, experienced, system. ‘Bali’ was a tax to be given in the
virtuous and selfless ministers who gave scale of the land under cultivation. ‘Bhag’
advice to the king. There was a committee was a share from the tax on production.
of some of these ministers, who were known Agricultural land and the tax acquired from
as ‘Mantrana’ to look after the day to day the agricultural production was the basic
affairs of the state. Mauryas had created a foundation of the administrative system.
stable framework of administration by Industries gained momentum during this
creating various departments of administration period. Cloth production was the most
and appointing the experienced and expert important industry. Apart from that metallurgy,
officers over it. These officers were known carpentry, ivory art, spinning, weaving and
as Amatya. Arthashastra mentions total 18 many such professions began. The prosperous
Amatyas such as Pradhan, Samaharta agriculture and flourishing industries led to a
(revenue), Sannidhata (finance), Senapati, flourishing internal and foreign trade. Internal
Yuvraj etc. Kautilya has given information trade was carried through land route and
about 30 administrative departments. The water ways. Many royal ways were built on
hierarchical order of the officers in the the trade routes during this period. Many
Mauryan empire started from the Samrat at roads such as Pataliputra to Takshashila,

60
Telegram Channel -: Mpsc Super Batch 123 Compiled by Sanjay Pahade 123
Pataliputa to Kashi-Ujjaini, Pataliputra to in Arthashastra.
Tamralipti came into existence. During the Mauryan period, along with
The increasing use of iron and also the Sanskrit literature, many texts were written
different types of iron implements, painted in Prakrit language. Especially the edicts of
pottery and its distribution upto southern Ashoka are inscribed on the rocks and pillars
parts of India indicates expansion of trade. in Prakrit language. The famous texts of
Bharuch, Roruk (Rodi), Sopara, Tamralipti Buddhist literature, Tipitaka, were edited
and many such ports on the coastal region during this period. Among the texts of
of India were famous for trade purpose. Tipitakas, Abhidhammapitaka was composed
Exports were carried out on a large scale after the third Buddhist council.
from India to countries like Greece, Rome, This period was very rich from the point
Egypt, Syria, Bactria, Sri Lanka etc. The of view of Jain literature. The texts like
commodities such as cotton and silk cloth, ‘Dashavaikalika’, ‘Upasakadashanga’,
linen, jari clothes, spices, diamonds, ivory, ‘Acharanga Sutra’, ‘Bhagvati Sutra’ etc. were
perfumes etc. were exported from India to composed during this period.
these countries. Glass articles, dyes etc. were Art and architecture
imported. The government levied taxes on the
After the establishment of Mauryan rule,
production of goods as well as on its import
there was happiness, peace, prosperity and
and export. Vartani (transport tax) and shulk
good governance. Mauryan art is the
(octroi) are found to be mentioned in the
beginning of ancient Indian architecture and
contemporary sources. In return the
the sculptural art.
government took up responsibility of security
of the goods. For the protection of the trading It appears that, during the Mauryan
community the government had appointed period, the artists had acquired the skill of
officers like Chorarajjuka and Seemaswami. cutting and carving the stone. In both, the
skill of sculpting and the technique of mirror
Literature
polish of the stone is seen. This is known as
Literature reflects the thought process of ‘Mauryan Polish’. During this period stone
the people and the political, social, economic statues of Yaksha and Yakshi are found.
and religious conditions. During the Mauryan Especially the statues of
period, along with the literature in Sanskrit Yakshi are beautiful and
language, the use of Pali and Ardhamagadhi finely carved. The famous
literature in Prakrit language is also seen. Chauri bearer of Didarganj
For the spread of Jainism and Buddhism, is supposed to be one of
literature was created in Prakrit language. such statues. The Yaksha
The Prakrit languages mainly included Pali, statues in Parkham and
Ardhamagadhi, Shaurseni, Maharashtri etc. the female images at
The world famous work of Panini’s Besnagar and Patna of
Ashtadhyayi was composed in this period. this period are exhibited
Bhasa was one of the great dramatists in in the Mathura museum.
Sanskrit literature. The thirteen dramas written
The pillars erected by
by him includes ‘Swapnavasavadattam’. The
Emperor Ashoka and the
most important work of this period is
sculptures on them are
Kautilya’s Arthashastra. This text is divided
excellent examples of
into 15 sections including total 180 topics. Chauri bearer Mauryan sculpture.
The topics from King to Ganikas and politics of Didarganj
to war strategies are found to be discussed

61
Telegram Channel -: Mpsc Super Batch 124 Compiled by Sanjay Pahade 124
Rampurva and Lauria Nandangarh. The Social Life
Sankissa pillar has an elephant capital. At The remains found in the excavations
Sarnath the pillar capital has four lions reflect the rich lifestyle of the people.
carved on it. This sculpture of the four lions Megasthenes mentions the seven classes of
is the ‘National emblem’ of India. Indian society based on their profession;
The beginning of rock-cut caves is seen Priest, Cultivator, Shephard and Hunter,
in the Mauryan architecture. The rock cut Trader and Labours, Soldiers, Spies and
caves on Barabar and Nagarjuni hills are the Government officials. The life of the people
first datable rock cut caves of India. was prosperous and happy. The class of
entertainers is mentioned which includes
Do You Know? actors, dancers, singers, musicians etc. Chariot
and horse racing, wrestling as well as dance
During this period the concept of
‘Vishti’ is mentioned in context of craft and singing competition were the popular
production. ‘Vishti’ was a tax paid by the means of entertainment. Gambling was also
people in form of physical labour. Through in practice but the state had control over it.
vishti the artisans provided a stipulated The system of education during vedic
amount of free labour to the state. period continued during the Mauryan period.
The cities of Taxila, Kashi etc. became great
There was a huge palace of Chandragupta centres of higher education and art during the
at Pataliputra. Megasthenes compares it with Mauryan period.
the Palace of Susa, the capital of Iran. The Female education began to be neglected
high brick fortification during this period. According to Kautilya’s
wall was built for the Arthashastra, it seems that during Mauryan
protection of the palace period women were given certain rights.
and inside the palace Women had complete rights over the
there were many buildings. Stridhana. The Government took care of the
These buildings were built orphan and disabled women. Many women
of stone. Wood was also spies were a part of the intelligence system.
used in the buildings. The The Mauryan period has multiple
Chinese traveller Fa-hien dimensions and hence it has obtained a
has described this palace. historical significance. After the death of
One of the unique Ashoka, the centralised administration
contribution in the field continued to grow weak and the empire
of art are the huge Capital of an disintegrated into small kingdoms. In the
standing pillars erected Ashokan Pillar Post-Mauryan period Shunga, Kanva and
during the period of Satavahana rulers rebuilt their empires. Vedic
Ashoka for the spread of religion. We know religion, varnashrama system, and vedic
them as Ashokan Pillars. These pillars are lifestyle once again gained importance. We
erected at thirty places. They are erected at are going to study about these dynasties in
places related to important events in the life the next chapter.
of Gautama Buddha and on important royal
paths. During Ashoka’s period, stupas were
built on a large scale. It is said that during
his rule, Ashoka built 84,000 stupas.

62
Telegram Channel -: Mpsc Super Batch 125 Compiled by Sanjay Pahade 125
Exercise

Q.1 (A) Choose the correct alternative and (a) Spread of Art
write the complete sentences. (b) Spread of ‘Dhamma’
(1) The first well-known king of the (c) Growth of trade
Haryank dynasty is . (d) For recognition of his work
(a) Chandragupta Maurya Q.3 Explain the following statements with
(b) Bimbisara (c) Ajatashatru reasons.
(d) Mahapadma (1) Ashoka’s victory at Kalinga proved
(2) The Nanda dynasty was established to be a turning point in history as
by . well as the life of Ashoka.
(a) Dhanananda (b) Shishunaga (2) The war between Chandragupta and
(c) Mahapadma Nanda the Greek king Seleucus was very
(d) Emperor Ashoka important.
(3) The great drama in Sankrit literature, (3) The Mauryan administration acquired
Swapnavasavadatta, was written by a definite form.
. Q.4 State your opinion.
(a) Kautilya (b) Bharat (1) Chandragupta Maurya is the first
(c) Kalidasa (d) Bhasa Chakravarti emperor in the history of
(B) Find out the incorrect pair from India.
set B correct it and rewrite. (2) The rock edicts of Ashoka are
Set ‘A’ Set ‘B’ important sources of history.
(1) Built the foundation Bimbisara (3) Various trade guilds developed during
of Magadha Empire the Mauryan period.
(2) Brought an end to Chandragupta Q.5 Observe the map on Page number 58
rule of the Nandas Maurya and answer the questions based on it.
(3) Chinese traveler who Megasthenes (1) Regions included in Ashoka’s Empire
visited India during (2) Locations of Ashoka’s rock and
the period of pillars edicts.
Chandragupta Maurya
(4) Changed his mind Emperor Activity
due to the destruction Ashoka Collect information about the Stupas
in Kalinga war during the period of Ashoka.
Q.2 Choose the correct reason and complete 
the sentence.
Emperor Ashoka erected many pillars.
Reason .

63
Telegram Channel -: Mpsc Super Batch 126 Compiled by Sanjay Pahade 126
9. Post Mauryan India

Heliodorus at Besnagar (Vidisha) indicates


9.1 Shunga Empire
that there were some Greek followers of
9.2 Rise of Satavahana Empire Vaishnavism. Literature in Sanskrit language
9.3 Administrative system, literature, was encouraged during the Shunga period.
art and social life To review the changes in Sanskrit language,
Patanjali wrote the ‘Mahabhashya’ based on
Magadha was the first empire of India. Panini’s ‘Ashtadhyayi’. According to some
A stable and systematic administration was scholars, there were many new additions
set up during the Mauryan period. We studied made in the original text of Mahabharata
this in the previous lesson. After Emperor during this period. The Manusmruti was
Ashoka, due to the internal conflicts, this composed during this period.
empire became weak. After Brihadratha, the Along with literature, the Shunga’s
last Mauryan emperor, Pushyamitra Shunga contribution in the field of art is also
came on the throne. important. The stupas at Sanchi and Bharhut,
9.1 Shunga Empire Garuda pillar at Besnagar are excellent
The Shunga empire was spread from examples of art during this period. The
Magadha in the east to Siyalkot (Punjab) in sculptural art in the Shunga period widened
the west, as well as Himalayas in the North its horizons and depicted the life of common
to Vidarbha in the south. Although Pataliputra people on a large scale.
was the main capital of this empire, The last king of Shunga dynasty,
Pushyamitra established a second capital at Devbhuti, was not an efficient king. His
Vidisha. Pushyamitra Shunga was valiant minister, Vasudeva, killed him and established
and ambitious. He strengthened his rule in the Kanva dynasty. This incident is described
the provinces of Kosala, Vatsa, Avanti etc. in the Harshacharita by Banabhatta.
Later he conquered the lost provinces of 9.2 Rise of Satavahana Empire:
Magadhan empire upto Siyalkot. He After the Mauryas, the Shunga dynasty
courageously repelled the attack of the Greek established itself in the north whereas in the
King Demetrius. His victory over Demetrius south the Satavahana dynasty rose to power.
is mentioned in ‘Malavikagnimitram’ a play The disintegration of Mauryan empire created
written by Kalidasa. a favourable atmosphere for the rise of new
Try this out : Collect information powers. Satavahanas are mentioned as the
about the plays written by Kalidasa and earliest ruling dynasty in Maharashtra.
make a note based on it. Initially the Satavahana rule came to a rise
in Nashik, Pune, Aurangabad and later
After establishing his rule over North spread in the vast region of Maharashtra,
India and some parts of South India. Andhra and Karnataka. Paithan, in
Pushyamitra twice performed the Ashwamedha Maharashtra, was the capital of the
sacrifice to proclaim himself as a powerful Satavahanas. The Puranas mention the
king. Pushyamitra revived the vedic sacrificial ‘Andhras’ or ‘Andhrabhrutyas’. According to
system which had lost its significance during some scholars, these terms refer to
the Mauryan period. The Garuda pillar of Satavahanas.

64
Telegram Channel -: Mpsc Super Batch 127 Compiled by Sanjay Pahade 127
For the map of Satavahana empire, visit the became weak due to the constant struggle
website given below: between Shakas and Satavahanas.
https://en.m.wikipedia.org/wiki/File:Satvahana.svg 9.3 Administrative system, literature,
Many Satavahana inscriptions are found art and social life
in North Maharashtra. Simuka, the first Administrative System :
Satavahana king, is mentioned in the The state under the Satavahanas was
Naneghat inscription. Shri Satakarni, Hala, divided into small provinces and on every
Gautamiputra Satakarni, Yajna Satakarni province civil and military officials were
were among the important kings of the appointed to administer individual provinces
Satavahana dynasty. over it. These included civil officials, such
Before Gautamiputra Satakarni, some of as ‘Amatya’, ‘Mahabhoj’ and military
the Satavahaha kings were not capable. officials, such as ‘Mahasenapati’ and
Taking advantage of this situation, the Shaka ‘Maharathi’. Grama (village) was the smallest
Satrapas established their rule in the western unit of administration. Grama was a source
regions of India. Gautamiputra Satakarni, the of revenue and also for the recruitment of
Satavahana king, conquered the Shakas. soldiers during warlike situations. Thus,
Satavahana dynasty regained its strength Grama was an integral part of the central
with this victory. machinery.
Gautamiputra Satakarni conquered Agriculture was the main source of
Central India and the entire regions of means of subsistence. Along with it many
Dakshinapatha. He defeated the southern industries and trade flourished under the
kingdoms of Avanti, Surashtra (Saurashtra) Satavahanas. Various shrenis (Guilds) also
and the Shaka kings in Maharashtra. Similarly emerged during this period. The trade and
he established his dominance over the industries were controlled with the help of
Republic states in Central India and Rajasthan. these shrenis. The shrenis also provided
A hoard of coins is found at Jogaltembi in loans. Indo-Roman trade also flourished
Nashik district. These coins belong to Shaka during the Satavahana period. Trade centres
Kshatrapa Nahapana on which Gautamiputra like Pratishthan (Paithan), Tagar (Ter), Nasik
restruck his own emblem. This clearly (Nashik), Karhatak (Karhad) developed
indicates that Gautamiputra proved his during the Satavahana rule.
strength by gaining victory over Nahapana. Tagar (Ter) and Pratishthan (Paithan)
After Gautamiputra, Vashishthiputra are mentioned in the ‘Periplus of the
Pulumavi and Yajnashree Satakarni were the Erythraean Sea’. According to it, “Among
important kings. But after their rule, the market towns of Dakshinapatha, there are
Satavahana rule began to decline. They two cities of special importance; Paethana

For Additional Information : The reign and achievements of Satavahana Emperor


Gautamiputra Satakarni are described in the Nashik inscription of his mother Gautami
Balashri. Gautamiputra Satakarni is mentioned as ‘Shakapahlavyavannisudan’ i.e. the
destroyer of Shakas, Pallavas and Greeks, ‘Satavahanakulayashahpratishthapankar’ i.e.
the one who reinstalled the success of the Satavahana family, ‘Trisamudratoyapitavahana’
i.e. the one whose horses drank the water of the three seas. Such type of mentioning
indicates that many kings in the southern region must have accepted feudatory status and
ruled under the suzerainty of Gautamiputra Satakarni. The Satavahana kings used
Matronomics, i.e. using the names of their mothers as prefix, for example, Gautamiputra
Satakarni, Vasishthiputra Pulumavi etc.

65
Telegram Channel -: Mpsc Super Batch 128 Compiled by Sanjay Pahade 128
(Pratishthana or Paithan of today) at a Various artisons such as Kularika*,
distance of about twenty day’s journey to the Tilpishaka**, Kolika*** are mentioned in the
south of Barygaza (Bhadoch); beyond which inscriptions at Nashik and Junnar. Pliny
about ten days to journey towards the east, mentions thirty fortified cities that existed in
there is another great city, Tagar (Ter). Satavahana kingdom. The coins, found during
Carnelian is transported in great quantity the excavations at the archaeological sites of
from Paithan to Bhadoch. There are no these cities are indicators of the well
permanent roads built for these purposes, but developed trade during this period. Sopara,
only untreaded tracts. Cotton cloth, muslins Kalyan, Bhadoch were important trade centres
and sack cloth (rough jute fabric) and other of this period.
merchandise from coastal regions is *
‘अॅनाबेसिस’
*Potter **Oil म्हणजे
miller ***‘कूच
Weaver
transported from Tagar to Barygaza.”

You would like to know : Naneghat for the convenience of travellers. There is a
is a mountain pass on the ancient trade stone jar excavated for collection of toll in
route in Maharashtra. This route connects the pass. Sopara and Kalyan were the most
the ancient town ‘Jirnanagar’ (Junnar) and important trade centres in ancient
the konkan province. The route proceeds Maharashtra. Internal as well as foreign
in the direction of Junnar and descends trade was carried out from these towns.
into Kokan at Murbad (District Thane). Especially, the goods imported from Rome
Satavahanas excavated a cave on this were unloaded at Sopara and from there it
route; the inscriptions in this cave mention was carried over to Paithan and Kolhapur
the Satavahana Queen Naganika, the via Kalyan, Naneghat, Junnar, and Nevasa.
achievements of the Satavahana kings and Similarly the same route was used to
the donations given by them. These transport the goods to be exported to Rome.
inscriptions are in Brahmi script. There Another important feature is that the
were statues of the Satavahana kings. At inscriptions in the cave include numeric
the foothills of the pass there is a village symbols for “2, 4, 6, 7, and 9’’ that resemble
named Vaishakhede. A number of inns modern numerals.
(Dharmashalas) were probably built here

king of the Satavahana dynasty,


By the Way : This is compiled ‘Gathasaptashati’.
a picture of a ship on the Gunadhya, a minister in his
coin belonging to King court, wrote an incomparable
Pulumavi. This image on text named ‘Brihatkatha’ in a
the coin is an indication of Prakrit language called Paishachi.
flourishing sea trade during Sarvavarma wrote a treatise on
Satavahana period. Sanskrit Grammer, named
‘Katantra’.

Literature : Art and Architecture : The influence of


During the Satavahana period, learning Persian and Greek sculptural styles during
and art received royal patronage. Prakrit the Mauryan period seems to have decreased
language and literature flourished during during the Shunga and Satavahana period. A
Satavahana period. Hala, the seventeenth new native Indian style emerged in this

66
Telegram Channel -: Mpsc Super Batch 129 Compiled by Sanjay Pahade 129
For Additional information : The
world famous Ajanta caves, cave no. 8,
9, 10, 12 and 13 belong to the
Satavahana period. It is interesting that
the paintings in cave no. 9 and 10 are
the earliest in India.

Society : The society of Satavahana


period was organised into four varnas.
Karle Chaitya
Similarly during this period the caste system
also became deeply rooted. The intermixture
of varnas and castes (Varnasankara), closed
period. The four gateways (Toranas) of the nature of different guilds, need of including
Sanchi Stupa No.1 built during this period foreigners in the social structure were some
are excellent examples of this style. Many of the reasons behind it.
incidents from the life of Gautama Buddha
are carved on these gateways. A number of Apart from that, there were four classes
rock-cut caves (Viharas and Chaityas) at in the society. The first class consisted of
Bhaje, Nashik, Karle etc. belong to officials such as ‘Maharathi’, ‘Mahabhoja’
Satavahana period. Especially the Chaitya at and ‘Mahasenapati’. They were appointed on
Karle, its magnificient facade entrance and the various ‘Rashtrakas’ (Subhas).
sculptures are the finest examples of the ‘Mahabhoja’ was appointed on the Konkan
sculptural art of this period. province and Maharathi on the plateau area.
The second class consisted of ‘Amatya’,
‘Mahamatra’ and ‘Bhandagarika’, ‘Naigam’
Gathasaptashati : The Satavahana (traders), ‘Sarthavaha’ (chief of caravan
king, Hala, compiled several poems merchant), and ‘Shreshthi’ (Head of trade
written in Prakrit. He selected 700 guilds). ‘Lekhanika’ (scribes), ‘Vaidya’
verses, and compiled it which was (physicians), ‘Halakiya’ (cultivators),
named as ‘Gaha Sattasai’ (Gatha ‘Suvarnakar’ (goldsmith), ‘Gandhika’ (traders
Saptashati). This is the first text of perfumes) were included in the third social
composed in Maharashtri Prakrit class. Whereas the fourth class included
language. These poems beautifully ‘Vardhaki’ (carpenter), ‘Malakara’
portray human nature and transactions. (gardeners), ‘Lohavanija’ (blacksmith), and
Various religious observance, ethical ‘Dasaka’ (fisherman).
norms and contemporary festivals are The invasions of foreigners like Greeks,
described in it. Gatha Saptashati reflects Pallavas, Shakas and Kushanas took place
the social life in Maharashtra of during the Satavahana period. But these
Satavahana period. Some words in foreigners were soon absorbed in the social
Gatha Saptashati are used in today’s structure. This means that the contemporary
Marathi language as well. The poems society, was flexible enough to accommodate
in Gatha saptashati clearly indicate the the foreigners. However, these political
relation between Maharashtri Prakrit invasions also brought about some social,
and the modern Marathi language in economic and cultural changes which we will
Maharashtra. study in the next lesson.

67
Telegram Channel -: Mpsc Super Batch 130 Compiled by Sanjay Pahade 130
Exercise

Q.1 (A) Choose the correct alternative and Q.2 Explain the following statements with
write the complete sentences. reasons.
(1) The last emperor of the Mauryan (1) During the Satavahana period there
dynasty was . was rise of many trade centres.
(a) Ajatashatru (2) The caste system became rigid during
(b) Chandragupta Maurya the Satavahana period.
(c) Brihadratha Q.3 Give your opinion.
(d) Emperor Ashoka Prakrit language was encouraged during
(2) The Nashik inscription mentions king the Satavahana period.
as ‘Trisamudratoyapitavahana’. Q.4 Write short notes:
(a) Gautamiputra Satakarni (1) Nashik inscription of Gautami
(b) Hala Balashri
(c) Yadnashree Satakarni (2) Naneghat – ancient trade route in
(d) Simuka Maharashtra
(3) Satavahana king, Hala, compiled (3) Gatha Saptashati
. Q.5 Write information about the Satavahana
(a) Brihatkatha dynasty with the help of the given
(b) Gatha Saptashati points.
(c) Katantra (a) Rise and expansion
(d) Meghdoota (b) Administrative system
(B) Find incorrect pair from Set B and (c) Trade and industries
write the correct ones. (d) Literature and art
Set ‘A’ Set ‘B’ Activity
(1) Kalidasa Malavikagnimitram
Collect information about historical
(2) Gunadhya Gatha Saptashati
remains in your surrounding area and
(3) Sarvavarma Katantra
prepare an information booklet based on
(4) Patanjali Mahabhashya
the gathered information.



68
Telegram Channel -: Mpsc Super Batch 131 Compiled by Sanjay Pahade 131
10. Changing Times

10.2 Indo-Greeks, Shakas, Kushanas


10.1 Nomadic tribes of Central Asia
Indo-Greeks : The Greek Satrapas of
10.2 Indo-Greeks, Shakas, Kushanas
North-west are known as ‘Indo-Greeks’. In
10.3 Kushana Empire Indian tradition, they are mentioned as
10.4 Gupta Empire Yavana. Their objectives were to establish
10.5 Vardhan Empire their dominance in the region of Mediterranean
10.6 Karkotaka Empire Sea and to strengthen the hold over trade in
west and central Asia. Seleucus Nicator was
10.7 Trade, coinage, art, iconography
Indo-Greek king from the province of Bactria
10.8 Indo-Roman trade (Trade centres in the North-west. In the conflicts between
in Maharashtra) the Indo-Greek kings, the kingdom of Bactria
proved to be powerful. Bactrian king
Demetrius attacked India in 180 B.C.E. He
After the disintegration of Mauryan
won over Takshashila. His capital was at
empire, the ambition of local rulers got
Sakal (Siyalkot). During this period, the
strengthened. The regional kingdoms came to
Indo-Greek king Eucratides established an
a rise. Among them, we studied the Shunga
independent kingdom. In this way, two
and Satavahana kingdoms in the last lesson.
independent Indo-Greek kingdoms were
During the same period there began foreign
established in the northwest region. There
invasions, such as invasions by Greek,
were 40 Indo-Greek kings in total including
Shakas, Kushanas, on India. This period of
the two branches established by Demetrius
the rise of regional kingdoms was also the
and Eucratides. The only source to know the
period of political and social transition.
history of Indo-Greeks is their coins. Things
10.1 Nomadic tribes of Central Asia like emblems, script, portraits on their coins
After the death of Alexander, the of the kings, images of deities are their gifts
Satrapas appointed by him declared their to the numismatic tradition of India.
independent status and they came to be
known as Indo-Greek kings. During the To know the extent of Indo-Greek kingdom see :
https://en.wikipedia.org/wiki/File:Indo-
declining period of Indo-Greek kings, nomadic Greeks_100bc.jpg
tribes in Central Asia attacked Bactria. The
tribes of Central Asia migrated to India in Shakas : Shakas came from Central
this period. These migrations proved to be Asia. They established their kingdom by
important in the political scene in North driving out the Greeks of Bactria. Their
India. In the latter half of 2nd century B.C.E., colony acquired the name ‘Shakasthan’
the Pahalavas (Parthians) and Shakas (Shistan). ‘Maues’ alias ‘Moga’ was the first
(Scythians) attacked northwestern regions. Shaka king in India. He established the
The Yuezi tribes from China pushed the kingdom by conqureing the provinces of
Shakas out from Central Asia. Yuezis were Gandhara and Punjab. Due to the weakness
pastoralists. With the strength of their war of the succeeding Shaka kings, Pahalava
skills, they vanquished the local kings and king Gondophernes defeated them and
there they established their own kingdoms. established his rule in India.

69
Telegram Channel -: Mpsc Super Batch 132 Compiled by Sanjay Pahade 132
For additional information : The rule of Shaka king
Rudradaman proves to be noteworthy due to his conflict
with the Satavahanas as well as the cultural transition in
that period. The Junagarh inscription in Saurashtra is an
important Sanskrit inscription. It is inscribed below the
Ashokan edict. The Ashokan edict is in Prakrit while
Rudradaman’s inscription is in Sanskrit. It seems that the
Shaka kings had adopted Sanskrit language. The
Rudradaman inscription records repairs of Sudarshan lake
that was built during the Mauryan era. It shows that the
lake was used over a prolonged period. It mentions
Rudradaman’s victory in the Narmada valley, expedition
against Satavahana kingdom and praises him for his
Junagarh inscription victory over Yaudheya Republics in Rajasthan.

Although the Shaka rule in Gandhara Kushanas : After coming to Bactria from
and Punjab came to an end, some Shaka central Asia, the Kushanas captured the Indo-
rulers continued to rule in Western India. The Greek kingdoms. As an effect of their contact
Shaka administrative system was structured with the Greek is reflected in some traits of
after the Achaemenid and Seleucid models. Kushana culture. After pushing the Shakas out
The state was divided into various Satrapis to the South, under the leadership of Kujula
(provinces) and ‘Mahakshatrapa’, a military Kadphises, the Yuezi groups came together
official, was appointed for each satrapy. and Kujula Kadphises brought them to
These Satrapies were further divided into northwest India from the Hindukush mountains.
sub-sections and a Satrapa was appointed on He declared himself as the king of Bactria.
each of it. These Satrapas were permitted to With this the Kushana rule was established
engrave their own inscriptions and also issue upto Kabul, Kashmir. We get this information
coins. It seems that they had considerable from Chinese records, Greek, and Roman
autonomy. literature. In Indian literature, Kushanas are
referred as ‘Tukhar’ or ‘Tushar’.
10.3 Kushana Empire
You would like to know : Shakas
After winning over the small kingdoms,
were nomadic pastoralists. There are
Kushanas followed the system of Shakas and
no architectural remains, which can be
established their Satrapis there. A military
ascribed to Shakas. However, their
officer called ‘Satrapa (Kshatrapa) was
characteristic tombs have been
appointed on the satrapies. The King was the
discovered. These tombs contain
chief of all the Satrapas and adopted the
remains of an individual, who could
titles like ‘Rajadhiraj’, ‘Maharaj’ etc. The
be their chief, along with his personal
concept of divinity of the King is seen for
belongings including remains of his
the first time in the inscriptions on the
horse and other objects. Horse was an
Kushana coins. They called themselves as
important part of the culture of Shakas.
‘Devputra’(Son of God).
Their skills in mounted archery enabled
them to make swift attacks. Saddle, By defeating many Shaka kings,
reins, composite bow and arrow made Kanishka obtained the status of an emperor.
their war technique more effective. He built the Kushana empire from Kabul to
Pataliputra and from Kashmir to Malwa. He

70
Telegram Channel -: Mpsc Super Batch 133 Compiled by Sanjay Pahade 133
is said to have attacked China twice. From
Central Asia it was difficult to administer You would like to know : The detailed
over the far extended empire. Hence an Prashasti (eulogy) of Samudragupta is
inscribed on the Ashokan pillar at Allahabad.
intermediate administrative centre was the
This prashasti is a written source giving
need of the moment. Hence, Kanishka
information of his political career. This
established two capitals, one at Purushpur
inscription is known as ‘Prayaga Prashasti’.
(Peshawar) and the other at Mathura.
It is paradoxical right below Ashokan’s
Kanishka had organised the fourth Buddhist edict declaring the importance of
Council in the vihara at Kundalvan in ‘Dhammavijay’ is inscribed the detailed
Kashmir. account of Samudragupta’s ‘Digvijaya’.
The Kushanas had adopted the Indian
culture. This is obvious from the name of means, ‘the annihilator of all kings’ proves
last Kushana king ‘Vasudeva’. During his this. Samudragupta first conquered neighboring
period the decline of Kushanas began. The kingdoms. After the conquest in the North,
Kushana empire disintegrated into pieces and he carried out expedition in the South to
the various Satrapas became independent. conquer the Southern kingdoms. During that
The Kushana rule existed in the province of period, the rule of Vakatakas was strong in
Punjab and Gandhar till 4th century C.E. southern region. Samudragupta could include
the southern part upto the region of Kanchi
Visit the below website to see the expansion of the under his dominance, except the Vakatakas.
Kushana empire : https://en.m.wikipedia.org/wiki/
The rulers of the regions beyond his conquered
Kushan_Empire#/media/File:Kushanmap.jpg
area became his feudatories. By obtaining
10.4 Gupta Empire tribute from these kings, he made them accept
The history of Gupta empire indicates the his sovereignty.
process of transformation of a small state into After this great conquest (Digvijay),
a mighty empire. Srigupta was the founder of Samudragupta performed Ashwamedha
Gupta dynasty. His title ‘Maharaj’ indicates sacrifice and proclaimed himself as
that he was a feudatory. His son Ghatotkach Chakravarti king. Shakas, Kushanas, and
also uses the same title, which means during kings in Sri Lanka accepted the suzerainty
his period also the Gupta kings were of the Guptas. The image of Samudragupta
feudatories. Chandragupta I is given the credit playing a lyre (Veena) on his coins indicates
of expanding the Gupta rule and transforming that he was a patron of art. Chandragupta
it into an empire. The title added to his name, II defeated Shakas and ascended the throne.
titled ‘Maharajadhiraj’, gives an idea of the He took up the title ‘Vikramaditya’. He
increasing status of the Gupta kings. established his rule in the western regions of
Chandragupta I had married Kumaradevi who Malwa, Gujarat, and Kathewar. Due to this
belonged to the Lichchhavi clan. This the areas on west coast also came under the
matrimonial alliance proved to be politically dominance of the Guptas and they entered
beneficial for the Gupta dynasty. His portrait into the trade carried out in the west. Later
with Kumaradevi is seen on a coin issued by Chandragupta crossed Hindukush and brought
him. His rule was spread over in the regions the north-western provinces under his control.
of Magadha, Saket (Ayodhya) and Prayag. In this manner he established a sovereign
King Samudragupta was the most brilliant, state encompassing the entire north India. He
valiant and benevolent ruler of this dynasty. gave his daughter, Prabhavati, in marriage to
Samudragupta’s ambition was to rule the Vakataka king Rudrasena II and created
entire India as an unified empire. His title congenial relations with the Vakatakas in the
‘Sarvarajochchheta’ on his coins, which south.

71
Telegram Channel -: Mpsc Super Batch 134 Compiled by Sanjay Pahade 134
During the period of Kumargupta, son of kings proved to be incapable of protecting
Chandragupta II, the Huna invasions in India their kingdom. Finally their kingdom was
began. Kumaragupta withheld the attacks of disintegrated into many small kingdoms.
Hunas. After Kumaragupta, the succeeding

You would like to know : Chandragupta II gave patronage to many scholars, just
like his father and had deep interest in knowledge and art. It is said that there were
‘Navratnas’ (nine gems) in his court. They were : Dhanvantari (physician), Kshapanak
(astrologer), Amarsinha (lexicographer), Sanku (architect), Vatal Bhatt (magician),
Ghatkarpura (architect/author), Kalidasa (great poet), Varahamihira (astronomer) and
Vararuchi (grammarian).

72
Telegram Channel -: Mpsc Super Batch 135 Compiled by Sanjay Pahade 135
State administration and social life: excellent examples of their skills. Beautiful
The Gupta period is considered as the and realistic portrayal of various aspects of
Classical Age of Indian history. the king is characteristic of Gupta coins.
‘Nigam’, ‘Shreni’ and ‘Gana’ like
During the Gupta period, king was the
Organisations were established by traders
centre of administration. The Prince, Amatya
and industrialists.
(ministers) and advisors helped him. Many
times, the Prince was appointed as Viceroy Till the Gupta period, many foreign
who looked after the provincial administration. communities, came and settled in India.
Every province was subdivided. These They were accepted in the social system.
subdivisions were called ‘Vishaya’. There Smriti texts such as ‘Narada Smriti’,
was an independent administrator appointed ‘Yajnavalakya Smriti’ were written during
on these Vishayas. They were known as this period. These texts provide a glimpse of
‘Vishayapati’. ‘Kumaramatya’ were the the social changes in this period.
provincial officers and ‘Ayuktak’ were the
For additional information : Textile
officials who looked after the affairs of the
industry flourished during Gupta period.
districts. Gupta administration was
A variety of textiles were produced on
decentralized. Many decisions were taken at
large scale. These varieties include
the local level.
Kshaum (linen), Chitrapatta (printed silk),
The Gupta kings changed the economic Dukul (silk), Pulakbandh (coloured cotton),
structure of agricultural system. Their Pushpapatta (floral designs), Amshuka
preference was on giving the land to local (muslin). These textiles had great demand
farmers for tilling. Land donations were also in internal and distant market.
made for the cause of religion and learning.
These lands were tax free. They were called The prosperity is reflected in
as ‘Agraharas’. Similarly, in this period the contemporary sculptural art and literature.
civil and military officers were granted land Political stability in the Gupta period,
grants instead of salaries. The roots of feudal prosperity and royal patronage gave impetus
system, an important feature of the mediaeval to the creation of Sanskrit literature. One of
period lie in this system. It is believed that the best creations in Indian literature of this
this feudal economy brought about the decline period is Kalidasa’s ‘Shakuntalam’.
of urban centres. Because of revenue tax Human figures gained prime importance
exemption on the donated land, the revenue in the sculptural art of Gupta period. Human
decreased and royal treasures depleted. It sculptures and images of deities were created
was concentrated in the hands of local during this period. Sculptures belonging to
landlords. Because of this the king no longer this period are found at many places such as
remained in the centre of power. Sarnath, Devgadh, Ajanta. During this period,
According to the Chinese traveller Yuan Hindu, Buddhist and Jain religious icons
Chwang, sugarcane and wheat were cultivated were made of stone, metals and terracotta.
in the north-western province of the Gupta During this period, the foundation of
empire, while rice was cultivated in Magadha Indian temple architecture was laid down.
and eastern regions. Use of dressed stones became common during
Minerals, animals and herbs were the this period. The earliest remains of temples
main commodities of trade that brought of Gupta period are found in places like
prosperity during Gupta period. Goldsmiths Sanchi, Bhumra, and Deogadh. These are a
enjoyed a high social status in this period. few examples of the Gupta style of
The gold coins during Gupta period are architecture.

73
Telegram Channel -: Mpsc Super Batch 136 Compiled by Sanjay Pahade 136
Tibet, Korea, Japan, Shri Lanka etc. came
You would like to know : In the Gupta
here. Harshavardhan’s empire was the last
period, there was remarkable progress in
powerful empire in ancient India.
making alloys and casting of metals. The
10.6 Karkotaka Empire
iron pillar at
Among the dynasties that ruled in
Delhi stands as a
Kashmir, the Karkotaka dynasty (7-9th century
witness for this.
Since many C.E.) is mentioned by Yuan Chwang. He had
centuries the iron been to Kashmir when he had visited India.
pillar stands A detailed information regarding this dynasty
without rusting. is found in Kalhan's Rajatarangini.
From this we Durlabhavardhan was the founder of
understand that Karkotaka dynasty. His empire extended
during this period from Narmada to Tibet.
the science of The reign of Lalitaditya (724-760 C.E.),
metallurgy was the grandson of Durlabhavardhan, is
very advanced. noteworthy. He was also known by the name
‘Muktapeed’. He is credited with two victorious
expeditions (Digvijaya). He drove away many
Iron Pillar : Mehrauli foreign tribes on the northern borders of
Kashmir, especially the region in the Amudarya
10.5 Vardhan Empire basin. It included the Tukhar (Turks) as well.
During the declining period of the He conquered the regions from Avanti in the
Guptas, in Nothern India there was rise of north upto Pragiyotishpur (Assam). He
dynasties such as Vardhans at Sthaneshwar conquered Tibet with the help of Yashovarma,
(Thanesar), Maukharis in Ganga Yamuna the king of Avanti (Kanauj). After conquering
Doab, Maitrakas in Saurashtra etc. the regions upto Kaveri river in the south, he
Pushyabhuti was the founder of the Vardhan crossed the seas and reached the land beyond
dynasty. After the accession of (Shri Lanks). This is mentioned by Kalhan,
Prabhakarvardhan, the Vardhan dynasty which suggests that Lalitaditya had extended
became powerful. He took up the sovereign his empire from Tibet in the north to the river
title of ‘Parambhattarak Maharajadhiraj’. Kaveri in the south. Lalitaditya's empire is
The most prominent ruler of this dynasty was comparable to the Gupta empire. Lalitaditya
Harshavardhan. His empire extended from was a devotee of Vishnu. He built the Martand
Nepal to Narmada and Saurashtra to Bengal. temple. He set up the city of Lalitpur. Latpur,
After Harshavardhan, there was no heir to situated on the banks of river Jhelum, might
the throne under the situation the empire be the ancient Lalitpur. He built Buddhist
disintegrated and many feudatories declared Viharas at Hushkapur (Ushkur).
independent status in North India. 10.7 Trade, coinage, art, iconography
We get a lot of information about this In Indian history, the period of six
period from ‘Harshacharita’ written by centuries from 2nd century B.C.E. to 4th
Banabhatta who was the court poet and also century C.E., was the period of arrival of the
from the travel accounts of Chinese traveller Central Asian tribes and the kingdoms that
Yuan Chwang. The universities of Nalanda they established. These people adopted
and Vallabhi became the great learning various cultural traits as they came to India.
centres during Harshavardhan’s reign. The introduction of these traits brought about
Hundreds of students, not only from India a transformation in the cultural life of India
but also from other countries like China, as well.

74
Telegram Channel -: Mpsc Super Batch 137 Compiled by Sanjay Pahade 137
Telegram Channel -: Mpsc Super Batch 138 Compiled by Sanjay Pahade 138
and Chinese rulers. The legends on the
For Additional Information : Some
Kushana coins found in Central Asia are in
Bactrian coins resemble the Greek coins in
Prakrit language, written in Kharoshthi script.
terms of their shape and weight. On some
of these coins the image of owl is seen. This evidence indicates that people speaking
Owl is the symbol of Goddess Athena. Prakrit language were also staying outside
Athena was the main deity of Athens. the Indian subcontinent.
Indo-Greek coins are found in Bactria. The The Gandhara region had become a
Greek legends are written on the obverse melting pot of Indian, Greek, Roman, Persian
side and the legends are written in Prakrit. and Shaka cultures. This province acquired
Kharoshthi script on the reverse side. importance from cultural point of view. A new
style of iconography developed in the cities of
In contemporary India, agriculture and Pushkalavati, Taxila,
animal husbandry were the main sources of Purushpur etc. It is known as
livelihood. Along with it various industries ‘Gandhara style’. The theme
and trade developed during this period. of the sculpture was Indian
India’s contact with the foreigners proved to but the style was Greek. The
be beneficial for this. During this period also Mahayana sect of Buddhism
merchant and artisans guilds (shrenis) were gave an impetus for the
in existence. During this period, India’s sea development of this style of
trade began to increase on a large scale. art. Along with the images
Indian goods were sent upto the Red Sea and of Gautama Buddha and
from there via Egypt to Rome. From India, Bodhisatva statues of the
animals such as tiger, lion and monkey; birds donors are carved in stone.
such as parrot and peacock; wool, silk, But this style remained
muslin, cotton cloth, ivory, pearls, spices, Gandhara Art
limited to parts of North-
sandalwood, medicinal herbs, diamonds, west India. In the icons of Gandhara style,
precious stones, and other luxury items were there was more emphasis on physical beauty.
exported and from other countries goods such During the same period, Mathura and Varanasi
as lead, copper, glass, silver, gold, wines etc. style of art developed. The images of Gautama
were imported. Due to this trade the flow of Buddha were completely Indian in nature.
gold coins was constantly coming in India Mathura was an important centre of Indian art
and hence India became prosperous. of sculpture. Portrait sculpture is the feature of
The portraits of kings and images of Mathura style of art. The statues of Kushana
deities were inscribed on the gold coins of kings, Vima Takshama and Kanishka are
the Bactrian Greeks. There was a great examples of this portrait sculpture. Similarly
impact of the Greek and Roman coins on the images of Saraswati, Vishnu, Surya, Shiva
coins of Shaka and Kushana rulers. The use and Kartikeya were created for the first time
of Kharoshthi script is seen on the Shaka during this period. The sculptural art during
coins. Kushana period was an indication of innovation
The coins of this period indicate the in the history of Indian sculpture. During this
mixed impact of Greeks, Indian and Persian period there was an effort of imprinting the
cultures. The Kushana kings depicted Indian reflection of the then society.
deities on coins. They brought in circulation 10.8 Indo-Roman trade (Trade centres
the gold and copper coins with the image of in Maharashtra)
Shiva. Kushana coins are also found in the The Periplus of Erythrean Sea written in
border regions of India and China. There Greek language in the mid 1st century C.E.,
were political relations between Kushanas mentions about the Red Sea and the sea

75
Telegram Channel -: Mpsc Super Batch 139 Compiled by Sanjay Pahade 139
routes connecting coastal line, ports, provinces, idea of the extensive scale of this trade. The
goods exchanged in trade. Red Sea was flourishing Indo-Roman trade gave rise to
important for trade relations between India several markets and cities. There was rise of
and Rome. Among the trading commodities important ports in western India as well. The
textile, black pepper, precious stones, ivory merchandise transported to Maharashtra were
and animals such as monkey, parrot and stored at Ter, Nevasa, Bhokardan, Kondapur,
peacock for the entertainment of the Roman and Sannati. The ports of Sopara and Kalyan
elites came to be exported from India. Gold were important centres of Indo-Roman trade.
coins were paid in exchange of these goods. Excavations at the archaeological sites have
Coral and liquor were also imported. The brought forth the evidence of trade in the
remains of Amphorae are found in the form of Roman pottery (amphorae) and red
excavations. Residues at the base of the ware. The replicas of Roman coins are also
amphora, revealed remains of wines. found in the excavations. In South India,
Amphoras were also used for storing olive along with increase in trade the Buddhist
oil and garom (pickled fish). The price of centres were established at places such as
the goods exported to Roman market was Dharnikot, Amaravati and Nagarjunikonda.
paid in gold coins. The large amounts of In the next lesson we are going to study
these coins found in South India gives an about the ruling powers in South India.

Exercise
Q.1 (A) Choose the correct alternative and was concentrated in the hands of
write the complete sentences. local landlords.
(1) The history of Indo-Greeks is known (2) Every tide of Huna invasion made
through their . the Gupta rule weaker.
(a) literature (b) inscription Q.3 State your opinion.
(c) coins (d) pottery Samudragupta performed Asvamedha
(2) Gupta dynasty was founded by sacrifice after Digvijaya.
.
(a) Sri Gupta (b) Ghatotkach Q.4 Write Short notes.
(c) Samudragupta (d) Ramgupta (1) Gupta Sculpture
(2) Indo-Roman Trade
(3) After accession by defeating the
Shakas, took up the tile of Q.5 Answer the following question with the
‘Vikramaditya’. help of given points.
(a) Chandragupta II How did the standardisation in various
(b) Ramgupta (c) Chandragupta I sections during the Gupta period take
(d) Kumaragupta place?
a. division of Gupta administration
(B) Find the incorrect pair from set B b. changes in economic struture economy
and write the correct ones. of agriculture
Set ‘A’ Set ‘B’ c. Development of Gupta coinage
(1) Kshaum Linen cloth
(2) Pushpapatta Cotton cloth Activity
(3) Dukul Silk cloth Collect more information about
(4) Amshuka Muslin cloth ‘Harshacharita’ written by Banabhatta,
Q.2 Explain the following statements with with the help of internet.
reasons.
(1) During the Gupta period the power 

76
Telegram Channel -: Mpsc Super Batch 140 Compiled by Sanjay Pahade 140
11. Kingdoms in South India

11.1 Important kingdoms in South India established their rule from Pudukkottai to
11.2 Administrative system, trade, social Kanyakumari. Towards their west, the Cheras
life had created their kingdom in Kerala. They
11.3 Literature, art, architecture are mentioned as ‘Kedalaputra’ (Keralaputra/
Sons of Kerala) in contemporary literature.
In this lesson we will be introduced to These kingdoms were in constant conflict
the kingdoms in South India, their with each other to establish political suprem
administrative system, trade etc. acy.
11.1 Important Kingdoms in South In many stories of Sangham literature,
India we find mention of the first Chola king
Chola, Pandya and Chera dynasties : ‘Karikala’. He brought together eleven small
Chola, Pandya and Chera were the ancient kingdoms and built a strong army and by
ruling powers in South India. The kingdoms defeating the Chera and Pandya kings, he
in South are mentioned in Megasthenes’ established the Tamil dominance.
‘Indica’, Panini’s grammar and inscriptions Some kingdoms arose to the north of
of Ashoka. Ancient Tamil literature, known Krishna-Tungabhadra rivers. Among them
as ‘Sangham’ literature, is believed to be one Satavahanas were most powerful.
of the main sources of history of South Vakataka dynasty : The power of
Indian rulers. Satavahanas started growing weaker from
the 3rd century C.E. Vakatakas took advantage
For Additional information : of this situation and established independent
South India has a great contribution in rule. Vindhyashakti was the founder of
the history of India and its culture. Due Vakataka kingdom. After Vindhyashakti ,
to the cultural exchange between North King Pravarasena I ascended the throne. He
India and South India, the Indian culture expanded the Vakataka Empire to Malwa in
became diverse and still remained
the North and from Gujarat to South upto
united. The people living in South India
Kolhapur, Kurnool (Andhra Pradesh).
spoke Dravidian languages. Tamil,
Kolhapur was knwon as ‘Kuntala’ at that
Kannada, Malyalam and Telugu are the
time. Pravarasena I performed four
four main languages in the Dravidian
Ashwamedha sacrifices and took up the title
group of languages. ‘Brauhi’ language
‘Samrat’.
spoken in Baluchistan is a part of the
Dravidian group of languages. After Pravarasena I, the Vakataka
kingdom got divided and two main branches
Chola Dynasty : The Chola Kingdom were created. Among them, Nandivardhan
was established in first century C.E. Chola (Nagardhan-Ramtek, Nagpur district) was the
dynasty originated in Thanjavur and capital of one branch. The capital of the
Tiruchirappalli (Tamilnadu). This province second branch was at Vatsagulm, i.e. modern
was known as ‘Cholamandala’ (Coromandel Washim (Washim district).
is an anglicised form of ‘Cholamandala’). To Prabhavatigupta, daughter of Gupta
the south of Cholas, the Pandyas had emperor Chandragupta II, was married to

77
Telegram Channel -: Mpsc Super Batch 141 Compiled by Sanjay Pahade 141
Vakataka king Rudrasen II. This is already he established his capital at Vatapi (Badami).
mentioned earlier. Varahadev, minister of His grandson Pulakeshi I was the first
Harishena, was a follower of Buddhism. important king of the Chalukyas. He built the
Cave no.16 of Ajanta was excavated at his fort of Badami. He performed the
instance. The excavation of other caves at Ashwamedha sacrifice and took up the title
Ajanta and the murals were done during the of ‘Maharaja’. He also took titles like
reign of Harishena. ‘Prithvivallabha’ and ‘Satyashraya’. After
Vakataka king Pravarasena II composed Pulakeshi I, his son Kirtivarman I came to
‘Setubandha’ in Maharashtri Prakrit. Among power. During his reign, he conquered the
many of Kalidasa’s classical works, Kadambas of Vanvasi (Karwar) and Mauryas
‘Meghadoota’ deserves special mention. of Aparanta (North Konkan) and expanded
References to historical geography of his rule. Kirtivarman was a patron of art.
Vakataka period occuring in this poetic work The beautiful caves at Badami were created
are important. by him.
Chalukyas : During the rule of Pulakeshi II was the greatest king of the
Harshavardhan in North India, the Chalukya Chalukyan dynasty. He performed the
dynasty ruled in the south for approximately Digvijaya and strengthened the power of
200 years. Jaising was the founder of this Chalukyas. His achievements are described in
dynasty. In the beginning of 6th century C.E. the eulogy (prashasti) composed by poet

78
Telegram Channel -: Mpsc Super Batch 142 Compiled by Sanjay Pahade 142
Ravikirti. He defeated the Kadamba were built at Tiruchirapalli, Chingalput and in
kings, Maurya, Nala, Kalchuri, Rashtrakuta, east and west Arkat district. The war between
Lat, Malav and Gurjara dynasties and Pallavas and Chalukyas took place during his
established the Chalukyan rule over Vidarbha period. His son Narsimhavarman defeated the
and other parts of Maharashtra and Karnataka. Chalukyan Emperor Pulakeshi II. During
The expedition of Harshavardhan to conquer Narsimhavarman’s region, many temples were
the southern states was resisted by Pulakeshi built at Mahabalipuram. It includes the
II. He defeated Harshavardhan thereby monolithic temples (Rath Mandiras) as well.
acquiring great fame. After his victory he Yuan Chwang, the well-known Chinese
took up the the title ‘Parameshwara’. With traveller, was in his court for a certain period
the conquest in south, his empire extended of time. The literary works by Narsimharvarman
from Narmada to Kaveri and from east coast are important as major source of Pallava
to the west coast. His fame had reached history. The Pallava rule ended in 9th century
beyond India as well. During this period C.E. The Chola king, Aditya, defeated the
Badshah Khosrow Parvez ruled in Iran. It is Pallavas and destroyed it.
said that he had sent his ambassador to the
court of Emperor Pulakeshi. The Pallavas of
Kanchi defeated Pulkeshi II. Vikramaditya,
the son of Pulakeshi II, in turn defeated the
Pallavas. The struggle between Chalukyas
and Pallavas continued over prolonged period
of time. The last king of Chalukyas,
Kirtivarman, was defeated by the Rashtrakuta
king Dantidurg.
Pallavas : Around 6th century C.E. to 9th
century C.E., the Pallavas were known as the
most powerful rulers in South India. Historians
vary in their views regarding the origin of the
Pallavas. Some copperplates of the Pallavas Yudhisthira Rath Temple at Mahabalipuram
are found. There is a mention of Sinhavarman Rashtrakutas : Dantidurga was the first
and Shivaskandavarman Pallava who ruled powerful king of the Rashtrakuta dynasty.
over the regions of eastern coast. Kanchi was The rule of Rashtrakutas was spread from
the capital of the Pallavas. We get a detailed Vindhyan ranges to Kanyakumari in the
information of the Pallava dynasty beginning south. After Dantidurga, his uncle Krishna I
from the reign of Simhavishnu. He conquered became the king. He uprooted Chalukyas
the province of Cholas and extended his rule rule. The famous Kailas temple at Ellora is
from Krishna to Kaveri. After Simhavishnu, ascribed to Krishna I. The succeeding
his son Mahendravarman ascended the throne. Rashtrakuta rulers became influential in
He was a great scholar. He wrote the Sanskrit North India as well. Amoghvarsh was a
play ‘Mattavilasa’. He wrote books on subjects capable king of the Rashtrakuta dynasty.
like music, dance, sculpture, painting etc. Amoghvarsh composed the texts entitled
These art forms received patronage under him. ‘Ratnamalika’ and ‘Kavirajmarg’. He
Many monelithic temples were carved during established a new city at Manyakheta
his tenure, such temple complex at (Malkhed) near Solapur. The Rashtrakuta
Mahabalipuram is recognised as ‘Ratha dynasty declined because of the attacks by
temples’. During his reign, several temples Parmars and Chalukyas of Kalyani.

79
Telegram Channel -: Mpsc Super Batch 143 Compiled by Sanjay Pahade 143
of Kolhapur and Kadambas
captured some of the regions
in North Konkan. Mummuni,
brother of Chhittaraj, built the
huge temple of ‘Amreshwar
Mahadev’ at Ambarnath.
This is considered as the
beginning of the Bhoomija
style of temple architecture
in Maharashtra.
3. Shilaharas of
Kolhapur - The modern
districts of Satara, Sangli,
Ratnagiri, and Belgaum were
included in this kingdom.
Kailas Temple at Ellora Jatiga established the Kolhapur branch of
Shilaharas. Bhoj II was the important king
Shilaharas : There were three branches
of this dynasty. The cities of Kolhapur,
of this dynasty – Shilaharas of south Konkan,
Valivade and Panhala were the capitals of
north Konkan and Kolhapur. They called
this kingdom. The Shilaharas of Kolhapur
themselves as ‘Tagarapuradhishwar’ (Tagar-
are credited with the creation of the
Ter, Osmanabad district). Jimutvahan is
magnificent ‘Koppeshwar Mahadev’ temple
considered as the founder of all the three
at Khidrapur.
branches. They ruled for almost three hundred
years as the feudatories of Rashtrakutas and Gonds : The Gond dynasty was
later the Chalukyas. established at Chanda (Chandrapur) during
the Yadava period. Kol Bheel was the
1. Shilaharas of South Konkan -
founder of this dynasty. He brought the
‘Sanafulla’ established the south Kokan
people of Gond tribe together and encouraged
branch of Shilaharas. His son, Dhammiyar
them to rebel against the Naga dynasty. He
set up the village of Vallipattana and built
established the capital at Sirpur. In the later
a fort there. Later King Adityavarma
period Khandkya Ballal Singh built a fort at
expanded his kingdom from Thane to Goa.
Ballarpur and shifted the capital from Sirpur
Rattaraj was the last ruler of this dynasty.
to Ballarpur. The temple at Achaleshwar was
The history of this dynasty is known with
constructed during his period. Nilkanth Shah
the help of one of the copper plates found
was defeated by
at Kharepatan.
Raghuji Bhosale of
2. Shilaharas of North Konkan - Nagpur and the Gond
Kapardi established the north Kokan branch territories were merged
of Shilaharas. Rulers of this branch were in his kingdom. Sixty
initially the feudatories of the Rashtrakutas. two Gond kings ruled
Their capital was at Sthanak (Thane). for many centuries in
Aparajita was another important king of this Devgad, Nagpur and
branch. He ruled for around thirty five years. Chandrapur in
Chittaraj followed Aparajita as king. His Maharashtra.
brothers fought for their claim to the throne.
Queen Durgavati
The struggle of
Taking advantage of this, the Shilaharas
Queen Durgavati of

80
Telegram Channel -: Mpsc Super Batch 144 Compiled by Sanjay Pahade 144
the Gond dynasty, against the Mughals is ‘Lilacharitra’ written by Mhamimbhatta
considered important. After the death of of the Mahanubhava sect, ‘Viveksindhu’ by
her husband, Duragavati fought against in Mukundaraj, ‘Bhavarthdipika’ by
Mughal emperor Akbar. She was not ready Dnyaneshwar belong to this period. Similarly
to surrender to him. Instead she preferred the composition of saint poets like Namadeva,
death. Janabai, Chokhoba of the Varakari sect
composed poems, which are known as
Yadavas : Yadava dynasty is one of the
‘Abhangas’. Many temples were built in this
important dynasties of mediaeval Maharashtra.
period. They are generally referred to as
Bhillam V (1185-93 C.E.) was the important
‘Hemadpanti’ temples. The Gondeshwar
king of the Yadava dynasty. He expanded
temple at Sinnar and the temple complex at
the kingdom by defeating the Kalachuris. He Anjaneri are the finest example among the
established his capital at Devgiri and got temples built during this period. An important
himself coronated. In the later period Singhan feature of the Hemadpanti temples is that no
was an important ruler of this dynasty. He mortar is used. Instead the stones are
defeated the Hoysalas, Shilaharas and interlocked with the help of mortise and
expanded the rule of Yadavas. tenon. During this period, the crucial forts of
In 1294 C.E. Alauddin Khalji attacked Ankai and Tankai were constructed.
south India. Ramadeva, the Yadava king was
defeated by Khalji. From here onwards the
Yadava rule began to decline. In 1307 C.E.
Alauddin sent his commander Malik Kafur
to carry out a campaign over Devgiri. He
defeated the Yadavas. Yadavas accepted a
feudatory status to Delhi. In 1310 C.E.
Yadava king Shankardev was killed by
Malik Kafur. In 1318 C.E. the rule of Gondeshwar Temple at Sinnar
Yadavas was brought to an end by the Delhi 11.2 Administrative system, trade,
Sultanate. social life
The Yadava period has a special In the administrative systems of South
importance in the history of Maharashtra. Indian dynasties there were officials such as
During this period there was notable ‘Mahadandanayaka’, ‘Rashtrika’,
development in the cultural sphere of ‘Deshadhikruta’, ‘Amatya’, ‘Ayukta’, etc.
Maharashtra. The Mahanubhav and Warkari The council of officers in the Chola kingdom
sects had a beginning in the Yadava period. was called ‘Udankuttam’. The kingdom was
During this period, Patan in Khandesh, divided into many provinces. These provinces
Solotagi in Karnataka and Paithan in were known as ‘Mandalam’. A member of
Maharashtra become important learning the royal family was the chief of the
Mandalam. There were many officers under
centres. During the same period, Sanskrit
him such as ‘Vishayapati’, ‘Deshadhipati’,
texts were written on subjects like
‘Deshadhikrut’, ‘Rashtrika’, etc. The
Dharmashastra, ‘Purva Mimansa’
administration of the Southern kingdoms was
(philosophical text), Nyaya and Vedanta. The
very efficient. Every order issued related to
critical work on Yajnyavalkya by Aparark, state affairs recorded down and no action
‘Chaturvargachintamani’ are among the was taken unless authenticated by the
important Sanskrit texts. ‘Sangitaratnakara’, concerned officer. Village autonomy was an
text based on Indian music, by Sharangdev important feature of the state administration
is considered relevant even today. in South India. The ‘Gramasabha’ (village

81
Telegram Channel -: Mpsc Super Batch 145 Compiled by Sanjay Pahade 145
council) looked after the entire administration industries in South India flourished during
of the village. The head of the Gramasabha this period. Cities like Paithan, Tagar (Ter),
was known by different names such as Nashik etc. flourished because of it.
‘Gramabhojaka’, ‘Gramakuta’ etc. head of The goods produced in rural areas were
the Gramasabha was elected by the villagers brought for sale in the cities. The market
or sometimes appointed by the king. A place used to be located in the centre of the
council similar to Gramsabha functioned at city. The merchant guilds (Shreni) in South
district and provincial level. Land revenue India played an important role in trade and
was the main source of income for the state. the social system.
Apart from it the excise tax, professional tax,
11.3 Literature, art, architecture
pilgrimage tax were the sources of income.
The tradition of writing poetry and
In the kingdom of Cholas, Pandyas,
grammar was prevalent in south India from
Cheras, Vakatakas, Chalukyas, Rashtrakutas,
ancient times. The ‘Sangham literature’ is
Pallavas, Shilaharas and Yadavas many
supposed to be most ancient in the Tamil
professions requiring specialised skills were
tradition. This literature is one of the main
practiced. Ivory was obtained from the
sources of political history of South India. It
Malayagiri jungle. The Cholamandalam
is believed that three ‘Sanghams’ (Council)
provinces were well-known for excellent
were held.
quality textiles. Plain and coloured cotton
cloth as well as silk clothes was produced in This period was important from the point
this province. In the Chera kingdom was of view of Sanskrit literature. Kalidasa
known the fine cloth produced there. It was composed the ‘Meghadoot’ at Ramtek near
one of the important commodities in the Nagpur. During the Vakataka rule,
Indo-Roman trade. It is mentioned in the compositions in Prakrit language were created.
travel accounts of Marco Polo. The trade and Pravarsena II, the Vakataka king, wrote the
well-known composition named ‘Setubandha’.
For Additional information : Coins ‘Harivijaya’ was composed by Sarvasena,
of South India- During the Mauryan the founder of Vatsagulm branch of
period the Maurya coins were in circulation Vakatakas.
in South India. After the decline of Two styles of architecture are seen in
Mauryas the Pandya kings issued their South India, one is the ‘Dravida’ style and
own punch marked coins. The images like other is the ‘Vesara’ style of architecture.
sun, horse, stupa, tree, fish were carved The Dravida style of architecture emerged
on it. On the Chera coins, there used to and developed in the regions from Krishna
be bow and arrow on one side and image river to Kanyakumari. The important feature
of an elephant on the other. On the Chola of Dravida architecture identified by
coins the symbol of tiger as the royal
construction of Shikhara. In this type the
emblem of Cholas is depicted. The Chola
storeys reduce in size as the Shikhara rises.
coins were made of gold, silver and the
Kailasnath and Vaikuntha Perumal temples
legends on it was inscribed in Devnagari
at Kanchi, Brihadeshwara temple at Tanjore
script. The coins belonging to Chalukya
king Rajaraja are issued in three metals- are some of the excellent examples of
gold, silver and copper, with the image of Dravida style of architecture.
Rajaraja and a tiger. Due to the trade The Chalukyas built temples at places
with Rome, many Roman coins are found like Aihole, Badami and Pattadakal etc.
in this area. It is seen that the seals of During the Rashtrakuta period the world
Indian rulers are reprinted on these coins famous ‘Kailasa’ temple was created.
and again brought into circulation.
The metal statues of the Chola period

82
Telegram Channel -: Mpsc Super Batch 146 Compiled by Sanjay Pahade 146
are the best among Indian metal sculptures. Among
them the most famous is the bronze statue of Nataraj
Shiva.
During the Vakataka period the sculptural art
and paintings reached its zenith. The Ajanta cave no.
1, 2, 16, 17, and 19 were carved out during this
period. It seems that the artists had good knowledge
about the details of physiology and nature.
Till now we studied the cultural and political
history of India. Ancient India had established cultural
relations with far away countries.
In the next two lessons we will study about the
cultural relations between India and other countries.
Bronze statue of Nataraj Shiva

Exercise

Q.1 (A) Choose the correct alternative and Q.2 Explain the following statements with
write the complete sentences. reasons.
(1) The name of the well-known king of (1) A strong rule of the Chalukyas was
Vakataka dynasty was . created in south India
(a) Sarvasena (b) Pravarasena (2) Yadava period has a special
(c) Jaising (d) Chandragupta importance in the history of
(2) After the conquest in South, Maharashtra.
Pulakeshin II took up the title of Q.3 Write short notes.
. (1) Administrative system in South India
(a) Parameshwara (b) Vishayapati (2) Coins in South Indian dynasties
(c) Deshadhipati (d) Satyajay Q.4 Answer the following question in brief
(3) Kalidasa composed at with the help of points given below.
Ramtek in Vidarbha. Describe the rule of Shilahara dynasty.
(a) Shakuntal (b) Meghdoot (a) Founder
(c) Malavikagnimitra (d) Harivijay (b) Shilahara of south Kokan
(4) The world famous Kailasa temple (c) Shilaharas of north Kokan
was built during the period. (d) Shilaharas of Kolhapur
(a) Chalukya (b) Pallava Activity
(c) Cher (d) Rashtrakuta
Visit the Kailasa temple at Ellora and
(B) Find the incorrect pair from set B prepare a detailed note.
and write the correct ones.
Set ‘A’ Set ‘B’ 
(1) Setubandh Panini
(2) Harivijay Sarvasena
(3) Indica Megasthenes
(4) Sangit Ratnakar Sharangdev

83
Telegram Channel -: Mpsc Super Batch 147 Compiled by Sanjay Pahade 147
12. India, Nations in the northwest of the Indian
Subcontinent and China

12.1 Trade and Cultural Relations in


never tried to impose their culture, religions
the Ancient times
and political rule on the local people.
12.2 India and Gandhara (Afghanistan
Wherever they went, they had cultural contact
and Pakistan)
and exchange with the native people and it
12.3 India and China
resulted into the enrichment of the native
cultures.
12.1 Trade and Cultural Relations in The spread of Indian culture in the
the Ancient times nations lying beyond the Hindukush mountains
So far, we studied about the transitions was mainly a result of the spread of Buddhist
in the various fields such as historical, social, religion.
financial, cultural etc., over a prolonged The Indian trade with distant lands is
period of about 4000 years. In this lesson we mentioned in the texts like ‘Kathasaritsagara’,
are going to study about the impact of the Jataka Stories, ‘Deepvamsa’, ‘Mahavamsa’,
Indian culture on the other countries. etc. These texts narrate many stories of the
It is very significant that the Indians sea travels and adventures of the Indian
travelling or migrating to other countries merchants. The Sangham literature mentions

84
Telegram Channel -: Mpsc Super Batch 148 Compiled by Sanjay Pahade 148
the boats of the yavanas who brought gold group of merchants, who paid a visit to the
and carried back black pepper in exchange. court of the Roman Emperor Augustus.
There are a number of inscriptions reported The coin hoards found in Tamilnadu had
from the western parts of Maharashtra, that Roman gold coins in it. Many of those coins
mention the donations given by yavanas. show cuts taken for the purpose of assaying
‘Ophir’, a port city is mentioned in the (for checking the grade of gold) the gold. It
‘Old Testament’ (Bible), which is supposed means those coins were not in circulation as
to be referring to ‘Sopara’ near Mumbai. The coinage but they were valued for its gold. It
Indian teakwood and the Himalayan cedar is said that Nero, the Roman emperor had
timber was in demand in Babylon. Beside paid one million gold coins for a single
the timber, commodities like sandalwood, chalice of emerald of Indian make. Pliny the
ivory, tortoise shells, monkeys, peacocks, elder had expressed concern about that the
pearls, precious and semi-precious stones, as Roman gold being drained into India. He
well as black pepper, cinnamon and incense compared India to an enormous sink draining
were also exported from India. A sailor’s all gold of the world. Strabo tells us that
handbook of his sea travels is known as animals such as snakes, hunting dogs, tigers,
‘Periplus of Erythrean Sea’ ( Latin name : elephants, birds like parrots and peacocks,
Periplus Maris Erythraei). The ancient port things like hides and horns of rhinoceros,
cities like Bharuch, Sopara, Kalyan, etc. are expensive textiles, pearls, ivory and spices,
mentioned in this periplus. Also the important etc. from India were in great demand among
trade centres like Ujjain are described in it. the Roman elites.
Beside the periplus there are other texts like Apart from the Roman gold coins, many
‘Geographia’ written by the Greek historian other things like lead, zinc, corals, wines,
Strabo, ‘Naturalis Historia’ by Pliny the olive oil were the Roman imports coming to
elder, ‘Geographia’ written by the Greek India. The archaeological survey conducted
mathematician-geographer Claudius Ptolemy, in the sea near ‘Bet Dwaraka’ in Gujarat, in
‘Indica’* by the Greek historian Arrian, etc., the year 2000-2001 has yielded amphorae of
which are the sources of information about various sizes, anchors, potsherds and a lead
the Indo-Roman trade. ball, etc. The amphorae were used to transport
olive oil and wines, which were imported
*Arrian was the historian of the 2nd century from Rome. There are many archaeological
C.E. He never visited India. Most of the
information in his work is derived from ‘Indica’ sites, which have yielded evidence of the
written by Megasthenes. Indo-Roman trade. It confirms that the cities
in Maharashtra like Paithan, Ter, Kolhapur*,
‘Periplus of Erythrean Sea’ mentions Bhokardan (Bhogvardhan) in the Jalana
that Hippalus, the Greek navigator had district were important trading centres.
accurately traced all the ports on the
Erythrean Sea. *Ptolemy has mentioned Kolhapur as ‘Hippokura’.
In the 1st century C.E. the Indo-Roman
trade had begun to grow. The silk route and 12.2 India and Gandhara (Afghanistan
the ports on the south and west coasts of and Pakistan)
India played vital role in this trade. The It is apparent that geographically
records of the Greek historians mention of a Afghanistan (Gandhara) was favourably

85
Telegram Channel -: Mpsc Super Batch 149 Compiled by Sanjay Pahade 149
By the way: The crows were known also clearly indicates that the ancient
to ancient sailors as the birds who begin Indian merchants used to sail to distant
to fly in the direction of land, the moment lands.
it is in their view. The sailors used to ‘Baveru’ is identified with Babylon.
follow the crows in order to reach the Babylon was a kingdom that existed in
coast safely. Crows trained for these Mesopotamia during 1800-600 B.C.E.
purpose were known as ‘Dishakak’. A Hammurabi was famous king of Babylon.
Harappan terracotta tablet shows a boat The Achaemenid emperor Cyrus II
and dishakaks flying above it. conquered Babylon in 539 B.C.E.
The story of ‘Baveru Jataka’ offers a This story confirms that ancient Indian
testimony to the use of dishakak by ancient merchants used to carry birds like crow
Indian sailors to reach the coast. This story and peacock for selling.

situated on the trade route that linked India Kushana Emperor Kanishka and Post-
and Central Asia. It had close cultural Kushana Period : Kanishka’s empire had
connections with India from the ancient spread from Pataliputra in the east to Kashmir
period of janapadas till the introduction of in the north and further to Central Asia.
Islam. Whether it were the invaders from the Purushspur (Peshawar) and Mathura were the
Central Asia, or the Buddhist monks sent out two capitals of his empire. Ancient Kapisha
by Emperor Ashoka , or the Chinese monks (Begram) was an additional capital of the
travelling to India, all had to pass through Kushanas, which was located on the silk
Afghanistan. route at a vantage point. The trade route
Emperor Ashoka’s Period : We have from Afghanistan to China was under the
seen that the 13th edict of Emperor Ashoka rule of Kushanas. This route starting from
mentions the names of Greek kings, who Taxila extended further, crossing through
were his contemporaries. It also mentions Khyber pass to Bamiyan in Afghanistan and
that people in their kingdoms were following from there to China through the plateau of
the path of morality as outlined to his Pamir.
message of Dhammavijaya. Among these In the Kushana period, Buddhism reached
kingdoms was included the kingdom of China by this trade route. Some of the coins
Kamboja in Afghanistan. of emperor Kanishka carry the image of
The Ashokan edict at Kandahar is in Gautama
Greek language and the script used for it is Buddha. It also
Aramaic. This inscription shows that has a legend
Afghanistan was the integral part of Ashoka’s which reads,
empire. ‘Boddo’. This
Ashoka had sent ‘Thera Mahyantika’ is the oldest
Gold coin of Kanishka representation
(Majjhantika) to Kashmir and Afghanistan
and ‘Thera Maharakkhita’ to the Greek of Gautama
(Yona) kingdoms, for spreading the teachings Buddha in an image form on a coin.
of Gautama Buddha.

86
Telegram Channel -: Mpsc Super Batch 150 Compiled by Sanjay Pahade 150
The Chinese monks like Fa-Hien, Yuan The building of the vihara complex at
Chwang, who travelled through India have Takht-i-Bahi had begun in the 1st century
described the Buddhist viharas and stupas in C.E. The work was continued till 7th century
India. There are numerous remains of viharas C.E. Among the remains discovered there are
and stupas in Afghanistan. Among them a three stupas and other buildings.
place called ‘Shahji-ki-Dheri’ near Peshawar Apart from these remains in Afghanistan,
in Pakistan was excavated. The importance the famous ‘Bamiyan statues of Buddha’ are
of the stupa discovered at this place is declared as the World Cultural Heritage.
underlined by the fact that it was built during Bamiyan is 250 kilometres away from Kabul,
Kanishka’s period. It is also known as to the west of it. There are about 750 caves
‘Kanishaka’s Stupa’. According to prevailing hewn out in one of the cliffs of the mountain
tradition, the sacred remains in the box ranges known as ‘Kuh-e-Baba’. There were
(karandaka) found there are of Gautama very beautiful Buddhist murals painted inside
Buddha. The box has an inscription. It also these caves. The same cliff has two huge
mentions the name ‘Agnishala’, the main niches dug in it. In these niches were built
supervisor of the building work of ‘Kanishka two standing Buddha statues in stucco. One
Vihara’, in the precints of ‘Mahasena of it was 53 metres tall the other was 38
Sangharama’*. This box is in the Peshawar metres. The core of these statues was carved
museum at present. out in the original sandstone of the cliff. The
core was plastered with several layers of
*Sangharama means the residential complex built mud mixed with straw. The folds of Gautama
for the bhikkhus. Buddha’s robes were modelled in this clay
plaster. His forearms were supported with the
The ancient ‘Nagarhar’, modern ‘Hadda’
help of wooden armatures and fixed with
near Jalalabad in Afghanistan was another
wooden pegs. According to Yuan Chwang
important centre of Buddhism. There are
the statues were surrounded by colourful
several remains of stupas and viharas
murals and even the statues were painted and
scattered over the place. The sculptures found
decorated with gold coating and precious
around the stupas are excellent specimens of
stones.
Gandhara style. ‘Takht-i-Bahi’ is another
important place, which is enlisted as ‘World These statues were destroyed by Taliban,
Cultural Heritage’. It is in the Pakhtunkhwa a fundamentalist organisation in 2001 C.E.
province of Gandhara region. It is a part of The government of Afghanistan, with the
Pakistan now. help of UNESCO and various associations
from countries like Japan, France, Switzerland,
etc. is working to restore this World Cultural
Heritage. Some of the murals inside the
Bamiyan caves were rediscovered, when the
work of restoration began. Also, remains of
a 19 metres long ‘Mahaparinibban’ image of
Gautama Buddha were found.
According to Yuan Chwang’s description
there was a library of rare Buddhist
manuscripts at Bamiyan. Archaelogists have
A Sculpture from a stupa at Hadda indeed found birch bark and palm leaf
manuscripts in a Bamiyan vihara.

Telegram Channel -: Mpsc Super Batch 151


87 Compiled by Sanjay Pahade 151
further to Central Asia, linking one oasis
to the other in the desert. It was easier for
the merchants and travellers to find
facilities of accommodation and food in
the oasis cities. They also found ready
market for their merchandise. Another line
crossed through the steppes to the north of
this main line. Journey on this line was
relatively shorter. However, it was rarely
used by merchants as it was infested by
troublesome pastoral tribes and it hardly
have any accommodation and food facilities
Bamiyan Buddha images en route.
The Silk route that continued from the
Some images of Hindu gods and Xinjiang province of China bifurcated into
goddesses have also been found in two lines and both reached Taxila. One of
Afghanistan. Among them is an image of it reached the Gandhara region from the city
Ganesha found near the city of Kabul. This of Kashgar in Xinjiang and the other reached
image is dated to 4th century C.E. and it is Kashmir via Leh from Yarkand in the same
the earliest image among the extant Ganesha province. The Chinese monks used the same
images. It may be noted that even in India route to reach India. An internal line to the
an image of Ganesha, as ancient as this one north of the Xinjiang line started from the
is not found. There is one temple called province of Gansu (Kansu) and reached
‘Khair Khana’ near Kabul. The excavations Kashgar. The famous ‘Great Wall of China’
at this site have yielded an image of God is in this province.
Surya mounted on a chariot.
A detailed review of all the Buddhist By the way : Some Chinese texts
and Hindu remains in Afghanistan is not mention Kashmir by the name of ‘Ki-
possible in this book. However, the above Pin’. Some texts have mentioned the
examples are enough to confirm the close ancient city of Kapisha (Begram) in
cultural association between the Gandhara Afghanistan by the same name.
Chinese literature mentions India
region and India before the advent of Islam.
variously as ‘Shen-tu’, ‘Tien-chu’,
12.3 India and China ‘Tien-tu’, ‘Xien-tu’, ‘Yuan-tu’, ‘Xuan-
The route that links Asia and Europe tu’, etc. Among all these names the
was referred for the first time as the ‘Silk name Yin-tu has prevailed in China
Route’ by Ferdinand von Richthofen, German till today.
geographer. The silk route runs more than In China the kings of ‘Han’
6000 kilometres. One may have an impression dynasty were ruling, when Kanishka
that the silk route is an unbroken highway. ruled in India.
However, in reality this route is an intricate Sir Aurel Stein, the British
web of major and minor lines bifurcating and archaeologist was the first to conduct
crossing each other. an archaeological survey on the silk
One of the major lines of this route route traversing through India, China
and Central Asia.
linked China and India and then extended

88
Telegram Channel -: Mpsc Super Batch 152 Compiled by Sanjay Pahade 152
It was in the 1st century C.E. that Buddha and Bodhisattvas. This art style that
Buddhism began to spread from India to originated in the Xinjiang province is known
China. The ‘Han’ dynasty that arose in China as ‘Serendian Art Style’ (Seres meaning
in this period had expanded its empire up to China + India). Serendian Art Style was
Central Asia. They dominated the silk route. influenced by Gandhara art. It presents a
Keeping in with the tradition ‘Ming-ti’ the combination of Greek, Persian and Chinese
second king of the Han dynasty had sent his art forms. During the exploratory survey
representatives to India. They returned to conducted by Sir Aurel Stein many terracotta
China accompanied by two Buddhist monks, sculptures of Serendian style were brought to
‘Kashyapa Matang’ and ‘Dharmaraksha’ in light.
67 C.E. They carried many Buddhist texts In the 4th- 6th century C.E. many Buddhist
with them. The texts were loaded on white temples and viharas were built in China.
horses. After reaching China the monks However, stupas built in this period were
translated the texts in Chinese language. The very few. Instead of a stupa every temple
Chinese emperor built a temple in their had a pagoda of Chinese style. These pagoda
honour, which is known as ‘White Horse being wooden, hardly any of them exists
Temple’. This is the first Buddhist temple today. These pagodas used to have multiple
built in China. floors. The upper floors of the pagodas
In the 3rd century C.E. the rule of the successively reduced in size. The uppermost
Han dynasty ended and the period of floor held a metal staff (yashti), which was
disintegration and unrest began with it, fixed with metal rings that became
pushing common people to a state of successively smaller in size. Arrangement of
desperation. This period lasted till 6th century staff and the rings seems to be the only
C.E. People got attracted to Buddhism in reminder of the Umberella in the Indian
this period in large numbers. Many Chinese stupa architecture. Later, pagodas were built
pilgrims began to visit Buddhist centres in by using bricks and stones.
Central Asia and India. In the 4th century The influence of Buddhist art tradition
C.E., Kumarjeeva the famous and highly from India still exists in the form of rock-cut
learned monk translated many Buddhist texts caves. Three of them have received the status
in Chinese language. In the 6th century the of World Cultural Heritage. Among them the
popularity of Buddhism reached its peak in caves of ‘Dunhuang’ are located on the
China. By this time both sects of Buddhism, ancient silk route. These caves are known as
Theravada (Hinayana) and Mahayana, and ‘Mogao Caves’. Creation of these caves
their sub-sects were well established in China. continued till 13th-14th century C.E.
In 7th century C.E. Islam and Christianity The merchants coming from the
were gradually introduced. Kubalai Khan, the interior regions of China used to gather at
grandson of Genghis Khan had great interest Dunhuang. It was an important meeting point
in Buddhism. for the Chinese merchants and those coming
The spread of Buddhism in China in the from other countries. Nearly 500 caves of
1 century C.E. also introduced a new trend
st
Dunhuang are enriched with numerous
in the Chinese art. Many of the Mahayana sculptures and murals. Thousands of
Buddhist monks who arrived in China, hailed manuscripts were discovered from these
from Central Asia. Their presence in China caves.
inspired the making of images of Gautama

89
Telegram Channel -: Mpsc Super Batch 153 Compiled by Sanjay Pahade 153
In the next lesson we will briefly its history and the extant traces of Indian
review the spread of Indian culture in Sri culture in those countries.
Lanka and the countries in Southeast Asia,

Exercise

Q.1 (A) Choose the correct alternative and Q.2 Complete the concept map.
write the complete sentences.
(1) The coin hoards found in Jataka Stories
had Roman gold coins in it.
(a) Tamilnadu (b) Maharashtra
(c) Karnataka (d) Kerala Texts describing the
(2) The famous king Hammurabi had Indian trade with
ruled the kingdom of . distant places
(a) Syria (b) Babylon
(c) China (d) Greece
(3) The route that links Asia and
is referred as the ‘Silk Route’.
(a) Europe (b) Africa Q.3 Explain the following statements with
(c) America (d) Russia reason.
(4) ‘White Horse Temple’ is the first (1) Buddhism reached China during
Buddhist temple built in . Kushana period .
(a) India (b) Japan (2) The merchants rarely used the shorter
(c) China (d) Egypt line of the Silk Route.
(B) Find the incorrect pair from set B Q.4 Give your opinion.
and write the correct ones. There were close cultural association
Set ‘A’ Set ‘B’ between the Gandhara region and India
(1) Strabo Geographia before the advent of Islam.
(2) Pliny the Elder Naturalis Historia
Q.5 Write short notes.
(3) Hippalus Hippokura
(1) Shahji-ki-Dheri
(4) Arrian Indica
(2) The Bamiyan Buddhas
(C) Write the names.
Q.6 Answer the following question with the
(1) A place in the Gandhara region
help of given points.
enlisted as a world Cultural heritage.
Write about the ties between ancient India
(2) Buddhist monks in the 4th century
and China.
B.C.E. who translated many Buddhist
(a) Trade relations
texts in Chinese language
(b) The spread of Buddhism in China
(c) Serendian art style
Activity
Collect more information about the ‘Silk
Route’ with the help of internet.


90
Telegram Channel -: Mpsc Super Batch 154 Compiled by Sanjay Pahade 154
13. India, Sri Lanka and Southeast Asia

13.1 India and Sri Lanka sermon from Thera


Mahinda, the King and
13.2 India and Southeast Asia
all the subjects
accompanying him
13.1 India and Sri Lanka became the followers
The history of Sri Lanka and India has of Buddhism.Anula,
remained close-knit from ancient times. the wife of king’s
‘Deepvamsa’, ‘Mahavamsa’, ‘Chullavamsa’ younger brother wished
are the three texts that tell us about the to become a Bhikkhuni.
Indian and Sri Lankan dynasties, their mutual On hearing of her
relations and the historical events, in the wish, Thera Mahinda
times before and after Gautama Buddha. suggested that his sister
These texts are known as ‘Vamsagranthas’. Theri Sanghamitta
According to the Vamsagranthas, the (Sanghamitra) may be
first kingdom of Sri Lanka was established invited from India, for
in the 6th century C.E. and was known as the purpose.
‘Tambapanni (Tamrapanni)’. Another name Statue of Accordingly, Theri
Devanampiya Tissa Sanghamitta arrived in
of the kingdom was ‘Rajrat’. Greek historians at Mihinthale
have mentioned it as ‘Taprobane’. According Sri Lanka. She brought a
to the tradition, King Vijaya, the founder of branch of the ‘Bodhi’
this kingdom was a prince from the Vang- tree along with her. King Devanampiya Tissa
Kalinga kingdom in India. He first went to welcomed her personally. Theri Sanghamitta
Supparaka (Sopara) on the west coast of initiated Anula into the Buddhist Sangha.
India from Vang-Kalinga and from there Anula was the first woman to became a
reached Sri Lanka. Buddhist nun. With Anula’s initiation, Theri
Sanghmitta established the first Bhikkhuni
For further information of the ancient Shasan (Bhikkhuni Sangha) of Sri Lanka.
kingdoms and important ports in Sri Lanka,
visit the following web site.
The festival known as ‘Unduvapa Poya’*
https://en.wikipedia.org/wiki/Anuradhapura_ is celebrated every year in Sri Lanka, on the
Kingdom#/media File:Important_ locations_of_ full moon in the month of December, in the
Anuradhapura_Kingdom.png memory of Theri Sanghamitta’s arrival.

Thera Mahinda (Mahendra), son of * ‘Unduvapa Poya’ means full moon in the month of December.
Emperor Ashoka arrived at Mihinthale, near
Anuradhpur, the capital of Sri Lanka. He The important cultural places in Sri
initiated (pabbajja/pravrajjya) King Lanka
Devanampiya Tissa of Sri Lanka to Buddhism. Anuradhpur  -  Mihinthale : Thera
This event has been described in great details Mahinda and Theri Sanghamitta stayed at
in the vamsagranthas. After listening to the Mihinthale near Anuradhpaur. It facilitated

91
Telegram Channel -: Mpsc Super Batch 155 Compiled by Sanjay Pahade 155
the establishment and spread of Buddhism in as a memorial to his queen. These temples
Sri Lanka. are the most ancient Hindu temples in Sri
Important stupas at Anuradhpur- Lanka.
Mihinthale : ‘Kantakchetiya’ is one of the The supremacy of the Cholas in Sri
earliest stupas at Mihinthale. An inscription Lanka was ended by Vijayabahu.
near the stupa mentions that the revenue Parakramabahu, a descendant of Vijayabahu,
collected from a nearby tank and the who ruled in 12th century C.E. is supposed
surrounding land was reserved as a gift for to be historically an important king. The
the maintainance of this stupa. Buddhist sanghas in Sri Lanka had become
The stupa erected on the remains disrupted by the time of Parakramabahu. He,
(Shareerik Dhatu/asthi) of Thera Mahinda at under the guidance of Mahathera Kassap,
Mihinthale, is known as ‘Ambasthal Thupa’. focused on reorganising them.
Parakramabahu had routed the
kingdom of Ruhuna in Sri Lanka. This
kingdom had a tooth of Gautam
Buddha (dantdhatu) in their custody. It
was reclaimed by King Nissanka
Malla. He built a temple on it in
Polannaruwa.
There is a stupa at the centre of the
temple. At the foot of the stupa is a
semicircular step, which is characteristic
of the stupa architecture of Sri Lanka. It
Thuparama
is called, ‘Moonstone’ (Chandrashila). It
King Devanampiya Tissa erected a stupa is carved with the figures of swans,
on the relics of Gautama Buddha in elephants, horses and creepers.
Anuradhpur. It is known as ‘Thuparama’.
Thuparama is the earilest stupa among those
that are extant in Sri Lanka.
Buddhaghosha was an Indian philosopher.
He stayed at ‘Mahavihara’ in Anuradhpur.
‘Vishuddhimagga’ a text written by him is
well-known. ‘Vishuddhimagga’ is the text,
which is honoured as an equivalent of
Tipitaka texts. ‘Moonstone’ (Chandrashila)
Pulatthinagar (Polannaruwa) : The city Galapotha (Book in stone) is a unique
of Polannaruwa is mentioned in Chullvamasa inscription recording the reign and
by the name, ‘Pulatthinagar’. In the 10th achievements of Nissanka Malla. It is
century C.E. the Chola emperor Rajraja I inscribed on a stone slab that is 8.17 metre
attacked Sri Lanka and razed Anuradhpur long and 1.39 metre wide. On one side of
completely. Then he established his capital the galapotha an image of Gajalakshmi is
at Polannaruwa. He renamed Polannaruwa as carved between two borders of a row of
‘Jananathmangalam’ and built a Shiva temple swans.
there. Later he built one more Shiva temple
The temple of dantdhatu at present is in

92
Telegram Channel -: Mpsc Super Batch 156 Compiled by Sanjay Pahade 156
the city of ‘Kandy’. It is known as ‘Sri
Dalad Maligava’. This temple is enlisted as
the World Cultural Heritage.

Galapotha

Mural of Sigiriya
By the way : The relics of (corporeal
remains) Gautama Buddha were handed By the way : The inscriptions
over to various sanghas in India and found in Sri Lanka, dated to the 3rd
other counries after his mahaparinibbana. century B.C.E. to the 1st century C.E.
These relics are known as ‘dhatu’. are inscribed in the Ashokan Brahmi
According to ‘Deegha Nikaya’ a script. Researchers opine that the modern
‘dantdhatu’ was given in the custody of Sinhala script gradually developed from
the king of Kalinga. Later this dantdhatu the Ashokan Brahmi.
reached Sri Lanka.
‘Lalitvistar’, the Buddhist text
A deep belief was rooted in Sri enlists 64 Indian scripts. The Brahmi
Lanka that a royal house holding the script is one of it. The scripts used in
custody of the dantdhatu had the divine Sri Lanka and many other countries in
right to rule. As a result the ruling kings Southeast Asia were developed from it.
of Sri Lanka strived to see that it
remains in the precincts of their palace.
Consequently the seat of the dantdhatu 13.2 India and Southeast Asia
kept changing from time to time. There are very few references in the
Indian literature to the Indian settlements and
Dambulla and Sigiriya : The caves at kingdoms in Southeast Asia. However, the
Dambulla in Sri Lanka are declared as court records of Chinese emperors provide
World Cultural Heritage. There are images considerable information in this regard. The
of Gautama Buddha and the Bodhisattvas ancient Indian literature refers to the land of
inside the caves. The roofs of five caves at Southeast Asia as ‘Suvarabhumi’ (the Land
Dambulla are decorated with paintings. of Gold).
There is an enormous rock in the The trading relations between India and
mountains near the city of Dambulla. A fort Southeast Asia began in the 1st century
and a palace was built on this rock. At its B.C.E. and continued through the 1st century
entrance a huge image of a lion was carved C.E. The strait of Malacca was convenient
in the rock. The place was named ‘Sigiriya’ for the Indian merchants to enter the South
after this lion. Sigiriya murals are compared China Sea. They unloaded their merchandise
with the murals at Ajanta. on the western coast of Malya Peninsula,

93
Telegram Channel -: Mpsc Super Batch 157 Compiled by Sanjay Pahade 157
from there transported it to the east coast by new lands they visited. Traces of Indian
land route and again loaded it on the boats. culture can be seen there even today.
This was easier than proceeding along the Myanmar : ‘Myanmar’ is a neighbouring
coast all the way. This trade by the sea and country of India, adjacent to its northeast
land route grew to a great extent under the border. It was earlier known as ‘Brahmdesh’.
rule of Chola kings at the end of the 10th In the 2nd century B.C.E. there were city
century C.E. states known as ‘Pyu’ in the north and
The term ‘Southeast Asia’ was coined in central regions of Myanmar. Some new Pyu
the times of the second world war. The cities came into existence at a later period.
Buddhist texts mention a ‘Suvarnabhumi’. Among them the cities of ‘Halin’ and
The scholars divide Southeast Asia into two ‘Shrikshetra’ were important.
parts based on its geographic features : Shrikshetra (near the city which was
1. The Mainland - This region is also known as ‘Prome’ during the British period
known as Indo-China. It comprises the and presently known as ‘Pyay’) was the
countries of Myanmar, Thailand, Cambodia, largest among the ‘Pyu’ cities. According to
Laos, Vietnam and also the western region the prevailing tradition the two brothers, who
of Malaysia. 2. The Maritime region - It were the founders of Shrikshetra hailed from
comprises the Malaya peninsula, the eastern the Shakya clan of Gautam Buddha. The
region of Malaysia as well as Indonesia. kingdom of Pagan (Bagan) arose in Myanmar
Although all these regions are included under in the 1st century C.E. and by the 11th century
a single umbrella name as Southeast Asia, it it grew into a great empire. All the Pyu city
is necessary to keep in mind the local states including Shrikshetra were merged into
diversity of cultural and historical constitution the empire of Pagan.
of each region. King Anawrahta the founder of ‘Pagan
empire’ is known as the greatest ruler in the
Southeast Asia – for additional information history of Myanmar. He is credited with the
visit the following web sites : unification of the north and the south
(1) http://www.world-maps.co.uk/continent-map- Myanmar. This unification is supposed to
ofsouth-east-asia.htm have given Myanmar its national identity.
(2) https://en.wikipedia.org/wiki/Khmer_Empire# Anawrahta put a check to the increasing
/media/File:Map-ofSoutheast-asia_900_CE.png) power of the Khmer empire of Cambodia.
During his reign the ‘Thervada Buddhism’,
The contact of Indians with various which had grown weaker, was revived.
regions in Southeast Asia was on the increase The remains of the fortifications and
because of trade during the 2nd century B.C.E. moats around the three Pyu cities of Halin,
to 2nd century C.E. The merchants who had Beikthano and Shrikshetra can be seen even
to travel for months together were naturally today. The archaeological excavations at the
accompanied by a large retinue of priests, sites of these cities have brought into light
monks, also travellers who were out to test many remains of buildings of those days,
their luck, ambitious members of royal stupas, cemeteries and the structures built for
families etc. These were the people who water management. The three sites have been
proved instrumental in the spread of Indian declared as the World Cultural Heritage.
culture in Southeast Asia. Some of them The ‘Shwedagon’ pagoda built between
even established independent kingdoms in the 6 - 10th century C.E. at Yongan (Rangoon)
th

94
Telegram Channel -: Mpsc Super Batch 158 Compiled by Sanjay Pahade 158
is looked at as the finest example of the Thailand : The ancient Thai people
stupa architecture in Myanmar. It is said that referred to their country as ‘Mueng Thai’.
two merchant brothers had visited India and However, it was known in the world as
they had the fortune of meeting Gautama ‘Siam’. In the 20th century its name was
Buddha in person. At that time they had changed to ‘Thailand’. Thailand was ruled
received 8 hair of the Buddha from himself. from the 6th to the 11th century by ‘Mon’
After returning to their motherland they people. At that time it was known as
handed over the hair to the king. The king ‘Dvaravati’. Indian culture was introduced
erected a stupa on the hair and it came to and spread in Thailand in the ‘Dvaravati’
be known as ‘Shwedagon’ pagoda. This period. The Indian traditions of sculpture,
pagoda is covered with sheets of gold. literature, ethics, judicial science, etc. had a
great role in shaping up the Mon culture.
Compared to other kingdoms in Southeast
Asia the kingdom of Dvaravati was smaller
and weaker. However, it contributed greatly
to the development of writing, arts,
administration, religion and science, etc. in
the other kingdoms. The remains of sculptures
and architecture of the Dvaravati period have
been found in the vicinity of the cities like
Lop Buri (Lao Puri) and Ayuttha (Ayodhya).

Shwedagon Pagoda

The ‘Anand Temple’ is another important


monument built during the reign of Kyanzittha,
the emperor of Pagan in the 11th century C.E.
It is the finest example of the combined
architectural style of India and Pagan.

Dvaravati style of temple architecture

The Dvaravati Sculptures show a great


influence of Indian sculptural style. Primarily
it includes Buddha images, but a few
Shivalingas and images of Vishnu are also
found. The sculptural art of Cambodia is
supposed to have originated from the
Anand mandir Dvaravati art style.

95
Telegram Channel -: Mpsc Super Batch 159 Compiled by Sanjay Pahade 159
In the 14th century C.E. a new kingdom skilled artisans. Arieal photography has
of Ayuttha was established in Thailand. In confirmed this description. The excavations
the 18th century it was completly razed by conducted by the French archaeologist Louis
the rulers of Myanmar. It was burnt by the Malleret have brought to light remains of
conquerors. All of its sculptures, libraries, temples built in brick masonry, workshops of
temples were completely burnt. making jewellery, residential houses, etc.
The kings of Ayuttha bore names with Roman coins of 2nd century C.E. were also
the prefix ‘Ram’. The popularity of Ramayana found.
may be the reason behind it. Thai Ramayana
has been developed into an independent * Mekong river originates on the plateau of Tibet
and flows through the Yunnan province of China,
tradition. It is known as ‘Ramakien’ (Ram Myanmar, Laos, Thailand, Cambodia and in the
Akhyana). The stories of Ramakien have end Vietnam, where it merges into South China Sea.
been preserved in all Thai art traditions
including sculpture, folk music, dance and 2. Champa : ‘Champa’ was an ancient
theatre. kingdom in the coastal region of Vietnam.
Vietnam, Laos, Cambodia : In the Several Sanskrit inscriptions in Brahmi script
colonial period Vietnam, Laos and Cambodia, have been found there. Champa was named
these three countries together were known as after the ‘Cham’ tribe. The names of the
‘Indo-China’. cities in Champa* were ‘Indrapur’,
During the 8th - 12th century C.E. the ‘Amaravati’, ‘Vijay’, ‘Kauthara’ and
Mon and Khmer people ruled over Cambodia. ‘Panduranga’. The city of Vijay was the
Khmer empire originated in Cambodia. capital of Champa kingdom. The inscriptions
of Champa mention the names of it kings
and queens and the temples of Hindu deities
For the reference of map visit the following
web site: https://commons.wikimedia.org/wiki/
built by them, especially Shiva temples.
File:Bandovietnam-final-fill-scale.svg Wooden images of Gautama Buddha have
also been found. This evidence indicates the
1. ‘Funan’ : This was a kingdom in existence of more city states like Funan in
Vietnam which existed in the Delta region of Vietnam, which were trade centres from
the river Mekong*. Funan is known mainly where various types of merchandise were
through the Chinese records. The ‘Han’ imported and exported.
dynasty ruled in China in the 3rd century C.E.
When the rule of Han dynasty was ended, To see the names and the map of Champa visit the
following web site: https://upload.wikimedia.org/
China disintegrated into three parts. Because
wikipedia/commons/4/45/VietnamChampa1.gif
of it the southern kingodom of China was
not left with any alternative path to reach During the span of 4th - 14th century C.E.
the silk route. Hence, the king of the southern Shaiva temples were built in the kingdom of
kingdom sent some people to explore the sea Champa. These temples are built in a valley
route. They found a kingdom in the delta known as ‘My Son Valley’. The temple of
region of the Mekong. They named it ‘Funan’. ‘Bhadreshvara’ is supposed to be important
According to their description, Funan was a in this group. Once there were more than 70
city with fortification, a place, well-established temples at My Son. In the walled courtyards
revenue system, laws, system of record of these temples many stone tablets with
keeping in written form, and a class of inscriptions are erected. These inscriptions

96
Telegram Channel -: Mpsc Super Batch 160 Compiled by Sanjay Pahade 160
are in Sanskrit and Cham languages. In the Buddhism is the main religion in Laos. The
same precincts are found burials of the cultural and literary expressions of people are
members of the royal families. It seems that influenced by the stories of Gautama Buddha’s
My son was the ceremonial complex, reserved life and Ramayana. Especially, this influence
for the royal families. The site of My Son is conspicuous in the sculptural and
is declared as the World Cultural Heritage. presentations of performing arts, which are
The characteristic aspect of the frequently based on the themes from these
architectural style of My Son temples is that sources. ‘San Sinxay’ is an epic composed
it is imagined in the form of ‘Meru Parvata’. in the 16th century in Laos. It is popular, not
only in Laos but also in Thailand. Its main
plot is quite akin to the story of Ramayana.
4. Cambodia : Cambodia was known
as ‘Kambujadesha’ in ancient times. Its
history is known from the inscriptions
installed in the precincts of its temples. These
inscriptions are in Sanskrit and Khmer
languages. The first kingdom, established in
Cambodia was known by the name of
‘Chenla’. The people who established Chenla,
were known as ‘Khmer’. The influence of
Indian culture in Cambodia dates back to the
Chenla period. The kingdom of Chenla was
established by Jayavarman II. He was
coronated in 802 C.E. His capital’s name
was ‘Hariharalaya’.
In the span of next 500 years the kings
Sculptural model of My Son Temple
of Chenla expanded their empire from
Vietnam to Myanmar and to China in the
The war that was continued for 20 years north. It came to be known as the Khmer
in Vietnam, Laos and Cambodia is known empire. After King ‘Jayavarman VII’ the
as ‘Vietnam War’. The vicinity of My Son Khmer empire began to decline. In the 15th
temples got damaged to a great extent century C.E. the Khmer empire was ultimately
because of this war. rooted out completely.
3. Laos : Laos is a landlocked country. The reign of Suryavarman II in the 11th
The population of Laos is mainly composed century and that of Jayavarman VII was
of the ‘Lao’ people who came to Laos from proved beneficial for temple architecture.
southern China. The name of the kingdom of The world famous temple of ‘Angkorwat’
Laos was ‘Lao Sang’. This kingdom was in was built in Yashodharpura, the capital city
existence during 14th-18th century. Lao Sang built by Suryavarman II. The temple area is
was attacked by Thailand in the 19th century. about 500 acres, that is about 2 Sq.Kilometers.
Lao Sang could not survive the attack. In The main entrance is at the west and the
the latter half of the 19th century the French temple is surrounded by a moat, which is
established their administrative centre in 200 metres in depth. Among the sculptural
Vientiane, the capital of Laos. panels on the southeast walls of the temple
Like other countries in Southeast Asia, the panel of ‘Samudramanthan’ is notable.

97
Telegram Channel -: Mpsc Super Batch 161 Compiled by Sanjay Pahade 161
Angkorwat Temple

After the death of Suryavarman II, Heritage by UNESCO.


Angkorwat was attacked by the king of Malaysia and Indonesia : Before the
Champa. He caused considerable damage to arrival of the Europeans, there have been
the temple. Later, it was turned into a three kingdoms in Malaysia. The ‘Vayu
Buddhist temple. Purana’ mentions Malay peninsula as
Jayavarman VII built a new capital, ‘Malaydvipa’. The Chinese Bhikkhu I-Tsing/
‘Angkorthom’ near Yashodharpura, the first Yi-Jing (7th century C.E.) had visited the
capital of the Khmer empire. He was a ‘Malayu’ kingdom. Ptolemy has mentioned
Budhhist. The planning of Angkorthom city, Malayu as ‘Maleu Kolon’ and ‘Golden
its water management and architecture are Chersonese’ (golden peninsula). An inscription
the indicators of the advanced Khmer style. in the Brihadishvara temple in Tanjore
The temple of ‘Bayon’ was built at the centre mentions it as ‘Malaiyur’. Malayu was one
of Angakorthom. Bayon temple represented of the kingdoms conquered by the Chola king
‘Mount Meru’. The planning of the temple Rajendra. The Chinese court records also
and the city represents the mythological story mention ‘Malayu’.
‘Samudramanthana’. The temple at the centre 1. Srivijaya : This kingdom proved to
is supposed to be the churner. There are be more powerful among all kingdoms who
statues of gods and daityas, the rivals of competed with each other. This kingdom
gods, standing at both sides of the southern originated in Sumatra. Malayu and other
gate, churning the ocean with Mount Meru neighbouring kingdoms who were weaker
as the churner. The moats around the city than Srivijaya were gradually merged into it.
are linked to two lakes called ‘East Barray’ In the 11th century C.E. Srivijya became
and ‘West Barray ’. These two lakes are the weaker while facing the Chola invasion. In
source of water for the moats and also for the 14th century C.E. ‘Parameswaran’ alias
Angkorthom as well as for Angkorwat and ‘Eskandar Shah’, the last king of Malayu
other temples in the vicinity. City gates of established the first sultanate of Malaya.
Angkorthom are known for their Shikharas, 2. Majapahita : In the 13th century C.E.,
which are in the form of a smiling human in Eastern Java, arose a kingdom known as
face of enormous size. The faces are built ‘Majapahita’. This was the last kingdom,
by assembling stones that are cut as different with Indian cultural traits. The name of the
parts of the face. founder king of Majapahita was, ‘Vijaya’.
Angkorwat, Angkorthom and its vicinity King Vijaya was successful in sacking Kublai
have been declared as the World Cultural Khan from Java. He established his

98
Telegram Channel -: Mpsc Super Batch 162 Compiled by Sanjay Pahade 162
A group of Hindu temples on the Dieng
For additional information : plateau in Central Java was built in the
I-Tsing/Yi-Jing had stayed in Sumatra Shailendra period.
before coming to India and also on his
4. Matram : There was a kingdom
return journey to China. He studied
called Mataram contemporary to Shailendra.
Sanskrit grammer during his stay in
Its founder king’s name was Sanjaya. During
Sumatra. He has mentioned the
the period of Matram kingdom, ‘Mahabharata’
kingdom of Sumatra by the name of
‘Shili Foshi’ (Srivijaya). The king of and ‘Harivamsha’ were translated in Javanese
Srivijaya had sent him to Malayu. He language. Poems in ancient Javanese language
stayed in Malayu for two months. are composed in Sanskrit metres like
Before returning to China, he again ‘Shardulvikridita’. Poems composed in metres
stayed in Malayu for two months. He are known as ‘Kakvin’.
mentions that in the intervening period The shadow puppetry show known as
of twenty years between his first ‘Wayang’ is a famous performing art of
arrival and return journey the name of Indonesia. This show is presented by using
Malayu had changed to Srivijaya. cut-outs from leather or wood. Stories of
According to his description there were Mahabharata and Ramayana are presented
more than a thousand Buddhist on the stage by the artists in this show.
Bhikkhus in Srivijaya studying Sanskrit. There are also some forms of ‘Wayang’
played by human characters.

For additional information : A


number of inscriptions have been
discovered in Sumatra. They are written
in the ancient Malayu language and the
script used for it is Pallava Brahmi (a
form of Tamil Brahmi script).

supremacy in some of the islands like Java,


Bali and also some other islands and extended
the kingdom of Java into an empire. Its
Wayang Puppets
existence came to an end with the rise of
Islamic states in the 15th and 16th centuries.
The Shaivaite temples, and Jaina temples
3. Shailendra : According to some were akso built in Indonesia. The group of
Indian historians the Shailendra kings hailed temples in Prambanan is important. This
from India. However, this opinion is not group has been accorded the status of World
accepted by all historians. The Shailendras Cultural Heritage. The main temple in this
reached the peak of their political power in group is known as ‘Candi* Prambanan’ or
the 8th - 9th centuries. The shailendra kings ‘Candi Lara/Rara Jonggrang’. It was built by
were followers of Buddhism. They built many King Daksha. This is a temple dedicated to
Buddhist temples and stupas. Among them Siva and it has a beautiful image of Goddess
the stupa of ‘Borobudur’ is unique for its Durga in it. Local people call her Lara/Rara
architecture, sculptural art and its expression Jonggrang.
of Buddhist philosophy. It has been enlisted
as World Cultural Heritage. * Candi (चंडी) means temple.

99
Telegram Channel -: Mpsc Super Batch 163 Compiled by Sanjay Pahade 163
Thus, so far, we have studied the history The history from the beginning of the
of cultural impact of India in several other Christian era in the ancient period to the
countries. Spread of Buddhism and Indian medieval age is the history of cultural
trade facilitated the spread of Indian culture. exchange between India and other countries.

For additional information : The base of every platform is decorated


According to Buddhist philosophy the with sculptures all around. There are
universe exists at three planes : (1) niches with images of Gauthama Buddha
Kamdhatu ( Bondage in the form of in it. On the third plane, there are three
desire) (2) Roopdhatu (Bondage in the circular platforms, which have stupas
form of physical appearance and with latticed structure along the edges of
appelations) (3) Aroopdhatu (State of the platforms. There are images of
being beyond any bondage). The Gautama Buddha inside the latticed
architectural design of Borobudur Stupa stupas. The last platform has a stupa in
is based on the concept of these three its centre. This stupa is built in solid
planes. The first two planes have masonry. The grand stupa of Borobudur
platforms that successively reduce in size. was built around 800 C.E.

Borobudur Stupa

100
Telegram Channel -: Mpsc Super Batch 164 Compiled by Sanjay Pahade 164
Exercise

Q.1 (A) Choose the correct alternative and (C) Write the names.
write the complete sentences. (1) Son of Emperor Ashoka -
(1) Buddhaghosh was an Indian (2) Ancient kingdom in Laos -
in Sri Lanka. (3) Kingdom of ‘Cham’ people -
(a) Thinker (b) Philosopher (4) Last king of Malayu -
(c) King (d) Priest Q.2 Write short notes on -
(2) The founder of Pagan empire was (1) Chenla kingdom
. (2) Visnhu temple at Angkorwat
(a) Kyanzittha (b) Anwrahta (3) Majapahit kingdom
(c) Ayuttha (d) Jayavarman (4) Champa kingdom
(3) The ancient name of Cambodia was Q.3 Answer the following questions in detail.
. (1) Explain the spread of Indian culture
(a) Kambuja Desh (b) Laos in Thailand.
(c) Angkorwat (d) Sumatra (2) Write about the cultural
(B) Find the incorrect pair from set B interrelationship between India and
and write the correct ones. Myanmar.
Set ‘A’ Set ‘B’ Activity
(1) Finest example of Shwedagon Find out the names of World Cultural
stupa architecture pagoda Heritage in this lesson. Collect their
in Myanmar pictures with the help of internet. Prepare
(2) Saivaite temples in My Son a chart by using the following points :
Champa kingdom Name of the heritage, place and country.
(3) The world famous
temple in Cambodia Angkorwat 
(4) The centre of Dieng
Angkorthom Temples

101
Telegram Channel -: Mpsc Super Batch 165 Compiled by Sanjay Pahade 165
Telegram Channel -: Mpsc Super Batch 166 Compiled by Sanjay Pahade 166

You might also like